You are on page 1of 74

APOSTILA DE INGLS NDICE

Substantivos .................................................................................................................................................. pg. 2 Pronomes ...................................................................................................................................................... pg. 4 Adjetivos ....................................................................................................................................................... pg. 5 Verbos ........................................................................................................................................................... pg. 7 Condicionais ................................................................................................................................................ pg. 13 A voz passiva ............................................................................................................................................... pg. 13 Discurso direto e indireto ............................................................................................................................ pg. 15 Phrasal Verbs ............................................................................................................................................... pg. 16 Subject Question ......................................................................................................................................... pg. 20 Advrbios ..................................................................................................................................................... pg. 20 Numerais ....................................................................................................................................................... pg. 23 Preposies .................................................................................................................................................. pg. 25 Prefixos e Sufixos .......................................................................................................................................... pg. 26 Artigos ........................................................................................................................................................... pg. 27 Conjunes .................................................................................................................................................. pg. 27 Question Tag ................................................................................................................................................ pg. 28 Conectivos ................................................................................................................................................... pg. 29 Vocabulrio ................................................................................................................................................. pg. 29 Exerccios ...................................................................................................................................................... pg. 30 Gabarito ....................................................................................................................................................... pg. 55 Prova anterior com resoluo (AFA) ........................................................................................................ pg. 55 Prova anterior com resoluo (ITA) .......................................................................................................... pg. 63 Prova anterior com resoluo (IME) ......................................................................................................... pg. 70

1. SUBSTANTIVOS
Dividem-se em prprios e comuns. Os comuns podem ser contveis e no-contveis e ambos podem ser concretos ou abstratos. CONTVEIS Podem ser empregues no singular, precedidos de: - A(N). Ex.: a book (um livro), a dog (um co), an elephant (um elefante), etc. THE: Ex.: the book (o livro), the dog (o co), the elephant (o elefante), etc. Podem ser empregues no plural, precedidos de: - artigo zero : Ex.: books (os livros), dogs (os ces), elephants (os elefantes), etc. - THE: Ex.: the books (os livros), the dogs (os ces), the elepants (os elefantes), etc. - SOME: Ex.: some books (alguns livros), some dogs (alguns ces), some elephants (alguns elefantes), etc. - HOW MANY (nas interrogativas): Ex.: How many books? (Quantos livros?), How many dogs? (Quantos ces?), How many elephants? (Quantos elefantes?), etc. - empregues com nmeros: Ex.: one book (um livro), five dogs (cinco ces), fifteen elephants (quinze elefantes), etc. - Concretos: a cat (um gato), an ashtray (um cinzeiro) - Abstratos: a conclusion (uma concluso), an idea (uma idia) NO-CONTVEIS Esses substantivos no podem ser empregues no plural, no podem ser precedidos de A(N), nem a seguir a um nmero. No singular, podem ser precedidos de: - artigo zero : Ex.: milk (o leite), aluminium (o alumnio), etc. - THE: Ex.: the milk (o leite), the aluminium (o alumnio), etc. - SOME: Ex.: some milk (algum leite), some aluminiun (algum alumnio), etc. - HOW MUCH: Ex.: How much mik? (Quanto leite?), How much aluminium? (Quanto alumnio?) - Concretos: butter (manteiga), soap (sabo),etc. - Abstratos: advice (conselho(s)), courage (coragem), etc. Existem substantivos no-contveis que no possuem a forma plural em ingls. Veja alguns deles: advice conselho(s) baggage bagagem, bagagens damage dano(s), prejuzo(s) evidence prova(s) homework - trabalho(s) de casa informatin informao, informaes laughter riso(s) news notcias research pesquisa(s) transport transporte(s) Esses substantivos no podem ser quantificados por si s, para isso necessrio recorrer a uma construo: PARTITIVE + OF + (substantivo no-contvel). Existem 3 tipos dessas construes: measure partitives (quantificadores de medidas), typical partitives (quantificadores especficos) e general partitives (quantificadores gerais ou neutros). Veja alguns exemplos: MEASURE PARTITIVES an acre of (land) um acre de (terra) an kilo of (meat, sugar) um quilo de (carne, acar) a pound of (butter, cheese) uma libra de (manteiga, queijo) a yard of (cloth) uma jarda de (tecido) TIPYCAL PARTITIVES an amount of (work) uma quantidade de (trabalho) a block of (ice) um cubo de (gelo) a glass of (water, wine) um copo de (gua, vinho) a sack of (coal, rice) um saco de (carvo, arroz) GENERAL PARTITIVES a bit of (advice) um conselho a bit of (interest) um pouco de (interesse) a piece of (furniture) uma pea de (moblia) a piece of (news) uma notcia PLURAL DOS SUBSTANTIVOS

Alguns substantivos apresentam-se somente na forma plural, referem-se a objetos constitudos por duas partes iguais. Ex.: bellows (fole), braces (suspensrios), goggles (culos de proteo), pliers (alicate), shorts (cales), suspenders (suspensrios), trunks (cales de banho), etc. Os verbos que os acompanham conjugam-se no plural quando concordam com o prprio substantivo ou quando concordam com a expresso two, three, etc., pairs of. Ex.: These trousers are torn. (Estas calas esto rasgadas.) My glasses are broken. (Os meus culos esto partidos.) There are three pairs of glasses on the table. (H trs pares de culos em cima da mesa.) There two pairs of trousers are dirty. (Estas duas calas esto sujas.) Os verbos que os acompanham conjugam-se no singular quando concordam com a expresso a pair of: Ex.: A pair of glasses costs a lot of money these days. (Um par de culos custa muito dinheiro hoje em dia.) There is a pairo of glasses on the table. (H um par de culos em cima da mesa.) J outros substantivos tambm se apresentam somente na forma plural, mas so acompanhados somente de verbos no singular. Esses geralmente referem-se a certos tipos de jogos, doenas, disciplinas, etc. Ex; athletics (atletismo), bowls (jogo com bolas de madeira), draughts (jogo de damas), gymnastics (ginstica), phonetics (fontica), shingles (herpes zoster) Physics is my favourite subject. (A fsica a minha disciplina favorita.) The news on television is more interesting. (As notcias na televiso so mais interessantes.) GNERO Quanto ao gnero os substantivos podem ser classificados em masculino, feminino, neutro ou comum. Feminino - Em ingls, como regra geral, o feminino formado a partir da adio do sufixo ess ao substantivo masculino. Ex.: author authoress, count countess, heir heiress Em alguns casos, existem mudanas na ortografia. Ex.: ambassador ambassadress, prince princess, duke, duchess Masculinos/femininos irregulares Alguns substantivos masculinos apresentam formas irregulares de feminino. Ex.: bachelor spinter, dog, bitch, fox - vixen, gentleman - lady, king queen, son daughter, wizard witch Invariveis Alguns substantivos apresentam uma forma nica, tanto para o masculino quanto para o feminino. O que os diferencia a aplicao dos pronomes pessoais, possessivos e reflexivos. Veja alguns deles: artist, baby, child, cook, cousin, dancer, driver, journalist, neighbor, painter, parent, person, prisioner, reprter, singer, teacher. Ex.: Our teacher didnt correct our test because she was so tired. (feminino) My cousin told me that he needed to take care of his dog. (masculino) Distino para animais Para difereciar o sexo dos animais, utilizam-se as formas male (macho) e female (fmea) antes deles. Ex.: male elephant (elefante macho), female elephant (elefante fmea). Essas formas tambm so usadas com referncia a pessoas, quando o substantivo do tipo uniforme. Ex.: male nurse/female nurse Substantivos femininos No h forma de masculino para alguns substantivos. Ex.: baby-sitter, miss, nanny. Casos especiais Alguns substantivos so diferenciados pela anteposio ou posposio de termos. Ex.: boyfriend girlfriend, man doctor woman doctor, he cat she cat Neutros Substantivos que designam seres inanimados e abstratos pertencem ao gnero neutro. Neste caso, os pronomes a serem utilizados, no singular, so it e suas formas associadas its e itself. Ex.: book, milk, fear, pride, etc. FORMA POSSESSIVA O caso possessivo (apstrofo) empregue para indicar posse de algo, ou relao entre pessoas, ou de pessoas e algo, num sentido genrico. Aplica-se a pessoas, grupo de pessoas, pases e animais, mas normalmente no empregada em seres inanimados. Adiciona-se o apstrofo nos seguintes casos: - substantivos singulares, mesmo os que terminam em s: Ex.: This is my sisters house. (Esta e a casa da minha irm). The actresss eyes are blue. (Os olhos da atriz so azuis.)

- substantivos plurais regulares: Ex.: This is my parents car. (Este o carro dos meus pais.) - substantivos plurais irregulars: Ex.: Childrens toys are usually expensive. (Os brinquedos para crianas so nomalmente caros.) - nomes de pessoas no singular, mesmo os que terminam em s: Ex.: Pauls ambition is to become scientist. (A ambio do Paulo se tornar um cientista.) - perodos de tempo no singular: Ex.: Yesterdays meeting was very tiring. (A reunio de ontem foi muito cansativa.) - em caso de lojas ou casas, emprega-se normalmente os substantivos singulares isoladamente com a adio dos: Ex.: Where have you been all this more? To the doctors. (Onde estiveste este tempo todo? No mdico.) - pode-se aplicar o apstrofo mais de uma vez na mesma frase, em certas situaes: Ex.: Johns daughters teacher is sick. (O/A professor(a) da filha do Joo est doente.) - nome de pessoas no plural: Ex.: The Smiths house is big. (A casa dos Smiths grande.) - nomes gregos antigos no singular, com mais de uma slaba: Ex.: Socrates ideas (as idias de Scrates) - perodos de tempo no plural: Ex.: Ill see you in two days time. (Ver-te-ei dentro de dois dias.)

2. PRONOMES
Pronomes pessoais (Personal pronouns) Os pronomes pessoais substituem na frase um substantivo ou um grupo nominal e indicam as pessoas gramaticais. Se dividem em pronomes do caso reto e pronomes do caso oblquo. Os subject pronouns (caso reto) so utilizados como sujeito, portanto, vm antes do verbo. I sempre escrito com letra mauscula. Quando o sujeito for composto, ele vir sempre em ltimo lugar, ex.: You and I are excellent players. He e she geralmente so usados para pessoas, mas so usados para animais ou coisas quando queremos personific-las, ex.: My dog is a poodle. He is very small. She tambm usado para substituir os substantivos ship (navio), nation/country (nao/pas) e car (carro). Ex.: Tha is a new ship. She is very modern. It neutro, usado para designar animais ou coisas no singular. Ex.: The aplle is red. It is red. Tambm pode ser usado para refir s palavras child (criana) e baby (beb) quando no se sabe o sexo. Ex.: There is a baby on the sofa. It is crying. Os objects pronouns (caso oblquo) tm a funo de objeto na orao e so sempre precedidos por um verbo ou uma preposio. Ex.: Mary and Jssyca are sisters. I know them well. Pronomes possessivos (possessive pronouns) Possessive adjectives my your his her its our your their Possessive Pronouns mine yours his hers its (esse raramente usado) ours yours theirs

Os possessives adjectives so usados antes de substantivos masculinos ou femininos, tanto no singular, como no plural, ou seja, so invariveis. Ex.: To his surprise, he couldnt win! Her blue eyes are beautiful. OBS: Com os possuidores everydoby/everyone, anybody/anyone, somebody/someone, nobody/no one, usa-se o possessivo his. Ex.: Everybody has to do his homework. Quando o possuidor for one, impossvel indicar sua posse atravs dos possessives adjectives, empregando-se a forma possessiva ones. Ex.: One has do ones homework. Os possessives pronouns, assim como os possessives adjectives, tambm concordam com o possuidor, mas nunca so usados antes de um substantivo, pois sua funo substituir a construo possessive adjective + substantivo. Ex.: Is that car still your car? Is that car still yours?

Outra forma de indicar posse pode ser feita por one of + possessive pronoun. Ex.: A teacher of mine (one of my teachers) is Italian. Pronomes Reflexivos (Reflexive pronouns) myself yourself himself herself itself ourselves yourselves themselves Reflexivo: indica que o sujeito pratica e recebe os efeitos da ao do verbo. O pronome vem logo aps o verbo e concorda com o sujeito. Ex.: I hurt myself in the soccer game. Enftico: enfatiza o sujeito ou o objeto da orao. Sua posio pode variar e no parte essencial da orao. Ex.: She talked to the king herself/She talked to the king himself. Idiomtico: vem precedido de by, formando uma expresso que significa sozinho, sem ajuda. Ex.: He lives by himself. Pronomes indefinidos (Indefinite pronouns) Embora sejam invariveis, mudam de funo de acordo com a forma em que forem usadas. SOME (algum, alguns, alguma, alguma), SOMEBOBY/SOMEONE (algum), SOMEWHERE (em algum lugar), SOMETHING (algo, alguma coisa) So empregados em frases afirmativas, antes de substantivos; em frases interrogativas, expressando oferecimento, pedido ou sugesto. Ex.: She ask me some questions./Would you like some candies? ANY (algum, alguns, alguma, algumas, nenhum, nenhuma, qualquer, quaisquer), ANYBODY/ANYONE (algum, ningum, qualquer pessoa), ANYWAY/ANYHOW (de alguma forma, de nenhuma forma, de qualquer forma), ANYWHERE (em algum lugar, em nenhum lugar, em qualquer lugar), ANYTHING (algo, nada, qualquer coisa) So empregados em frases interrogativas, em frases negativas e em frases afirmativas, com o significado de qualquer. Ex.: Do you need any help?/I dont need any help./Any student here speaks English well. NO (nenhum, nenhuma), NOBODY/NO ONE (ningum), NOTHING (nada), NOWHERE (em nenhum lugar). So empregados em frases afirmativas, cujo sentido expresse uma negao. Ex.: I have no money. Oraes Relativas (Relatives clauses) So oraes que identificam ou qualificam os elementos que as precedem em uma frase. Elas so iniciadas por um pronome relativo. Ex.: The boy who sits next to me is very handsome. So classificadas em Identificadoras (identifying) e no identificadoras (non-identifying). Identifying so as que identificam ou classificam o substantivo a que se referem, elas dizem a que coisa ou pessoa estamos nos referindo. Ex.: Was it your car which was towed by the police? Non-identifying estas apenas acrescentam informaes sobre o substantivo que j foi identificado. Ex.: This is my friend Perry, who works at Contry Hospital.

3. ADJETIVOS
So empregues para caracterizar os seres, objetos ou o estado das coisas. Em ingls eles so invariveis, isto , no fazem concordncia com a palavra qual esto associados. Ex.: a strong boy (um rapaz forte), strong boys (rapazes fortes) Os adjetivos podem ser empregues: - substantivo + adjetivo = an old man - verbo + adjetivo = He became angry. - verbo + complemento + adjetivo = She makes me happy. - advrbio + adjetivo = very hot - comparativo = Mary is younger than Emily. - superlativo = This is the cheapest. A maioria dos adjetivos mais comuns no tem uma terminao especfica. Veja como so formados alguns deles: - substantivo + able (tendo a qualidade de) = pleasure pleasurable (agradvel) - verbo + able (que pode ser) = imagine imaginable (imaginvel)

- substantivo + al (relativo a) = education educational (educacional) - substantivo prprio + an (habitante/lngua de) = Korea Korean (coreano) - substantivo + ed (tendo a caracterstica de) = disease diseased (doente, doentio) - verbo + ed (afetado pela ao do verbo) = interest interested (interessado) - verbo + ent (que , que faz) = differ different (diferente) - substantivo prprio + ese (habitante/lngua de) = Nepal Nepalese (nepals) - substantivo + ful (cheio de, tendo a qualidade de) = fear fearful (temeroso, medroso) - verbo + ible (que pode ser) = deduce = deducible (deduzvel) - raiz de origem no-inglesa + ible (que pode ser) = legible (legvel) - substantivo + ic (de, sobre, a respeito de) = poet poetic (potico) - verbo + ing (indicando a ao do verbo) = disgust disgunting (repugnante, nojento) - raiz de origem no-inglesa + ing (indicando a ao do verbo) = impending (iminente, prximo) - substantivo + ish (comportando-se como) = child childish (acrianado, infantil) - adjetivo + ish (um tanto, um pouco)= old oldish (um tanto velho) - substantivo prprio + ish (habitante/lngua de) = England English (ingls) - verbo + ive (tendo tendncia para) = deduce deducive (dedutivo) - raiz de origem no-inglesa + ive (tendo tendncia para) = lucrative (lucrativo) - substantivo + less (no tendo a qualidade de) = meaning meaningless (sem sentido, sem significado) - substantivo + like (semelhante a, caracterstica de) = child childlike (infantil) - substantivo + ly (tendo a qualidade de) = cost costly (caro, dispendioso) - adjetivo + ly (tendo a qualidade de) = lone lonely (solitrio) - substantivo + ous (tendo a caracterstica de) = peril perilous (perigoso) - raiz de origem no-inglesa + ous (tendo a caracterstica de) = previous (prvio) - substantivo + y (cheio de, tendo a qualidade de) = wind windy (ventoso, com muito vento) Existem tambm os adjetivos compostos, que so constitudos de duas ou mais palavras conjugadas e so empregados normalmente antes de um substantivo e com hfen. Veja a formao de alguns deles: - substantivo + particpio passado ed = a hand-made wallet (uma carteira feita mo) - substantivo + particpio presente ing = a fact-finding mission (uma misso de pesquisa) - substantivo + adjetivo = a waterproof watch (um relgio prova de gua) - advrbio + particpio passado ed = a well-known writer (um escritor bem conhecido) - advrbio + particpio presente ing = a long-playing record (um disco de longa durao) - advrbio + adjetivo = a fiercely-competitive sport (um esporte altamente competitivo) -adjetivo + parcitpio passado ed = a white-washed cottage (uma casa de campo caiada) - adjetivo + particpio presente ing = a free-standing sculpture (uma escultura sem nenhum suporte) - adjetivo + substantivo = a high-speed train (um comboio de alta velocidade) - adjetivo + substantivo + ed = a good-humoured person (uma pessoa alegre) - adjetivo de trs ou mais palavras conjugadas = an out-of-date dress (um vestido fora de moda)

COMPARATIVOS e SUPERLATIVOS
Adjetivos de uma slaba Comparativo: adjetivo + er (than): older, higher, bigger than Superlativo: The + adjetivo + est: the oldest, the highest, the biggest Adjetivos de duas slabas terminados em y: easy, heavy, funny, happy, dirty etc. O y muda para ier ou iest no final: easier/the easiest, heavier/the heaviest, funnier/the funniest, happier/the happiest, dirtier/the dirtiest. Muito menos comum so os adjeivos que terminam em ow narrow, shaloow, mellow narrower/the narrowest, shallower/the shallowest. Adjetivos de mais de uma slaba Comparativo: More + adj. (than): more tiring, more expensive, more famous, more beautiful, more sincere, more interesting etc. Superlativo: The most + adj.: the most tiring, the most expensive, the most famous, the most beautiful, the most sincere, the most interesting etc. Excees: Good better than The best Bad worse than The worst Far further than The furthest

Menos que Comparativo: It was less expensive/fun than last year. (Foi menos caro/divertido que no ano passado.) Superlativo: Shes the least generous/stupid person I know. (Ela a pessoa menos generosa/estpida que conheo.) Igualdade = AS AS Shes (not) as tall as me. Its just as hot as yesterday. I dont have as much money as I used to. Are there as many people as you expected? Graus de comparao Much/Far large, smaller, better etc. Considerably more difficult, popular, interested etc. A little more/less milk (substantivos incontveis) Slightly more/fewer people (substantivos contveis) Not quite as old/expensive as (Not) nearly as many/much as

4. VERBOS
RESUMO DE TEMPOS VERBAIS PRINCIPAIS Simple Continuous (To be+ ing) Present I live I am (Im) living Past I lived I was living Future I will (Ill) live I will (Ill) be living PRESENT PERFECT Simple Continuous Perfect (To have + pp*) I have (Ive) lived I had (Id) lived I will (Ill) have lived Perfect Continuous (To have + been + .ing) I have (Ive) been living I had (Id) been living I will (Ill) have been living

Have/Has + particpio passado (* a 3 forma do verbo) Have/Has + been + verbo com -ing

1. Passado no terminado = aes que comearam no passado mas no terminaram ainda: I have lived (Ive lived) in Rio since 2002. (Moro no Rio desde 2002.) She has been (Shes been) waiting for 20 minutes. (Ela est esperando h 20 minutos.) Se a ao j terminou, usa-se o Past: I lived in Rio for 2 years. (Morei no Rio durante 2 anos.) She waited for 20 minutes. (Ela esperou 20 minutos.) 2. Passado no-determinado = aes no passado, mas sem um tempo especfco, porque: a) Aconteceu no passado geral, na sua experincia/vida, e quando aconteceu no for considerado importante: Have you ever been to Paris? Yes, I have. (Voc j esteve em Paris? Sim, j estive l.) Ive seen City of God times. Have you seen it? (Eu j vi Cidade de Deus 3 vezes. Voc j viu?) No primeiro exemplo, a palavra ever usada para enfatizar a primeira vez em qualquer momento da sua vida Por outro lado, se voc est fazendo sobre um tempo mais especfico, usa-se o Past: Did you go to Paris when you were in France? (Voc j foi para Paris quando estava na Frana?) I saw Cit of God last year. (Eu vi Cidade de Deus no ano passado.) b) Aconteceu no passado recente; mais importante o que aconteceu h pouco tempo atrs do que exatamente quando aconteceu. Este tipo menos usado no ingles Americano. Weve bought a new car. (Compramos um carro novo.) My parents have just arrived. (Os meus pais acabaram de chegar.) Hes been playing football. (Ele estava jogando futebol.) No segundo exemplo, a palavra just serve para enfatizar que algo acabou de acontecer, que aconteceu recentemente, mas sem o tempo especfico. Quando o tempo determinado, por outro lado, usa-se o Past:

We bought a new car two weeks ago. (Compramos um carro novo h duas semanas atrs.) My parents arrived on Thursday. (Os meus pais chegaram na quinta-feira.) Outras palavras muitas vezes usadas com Present Perfect: Yet: Have you met her parents yet? (Voc j conheceu os pais dela?) I havent met them yet. (No conheci eles ainda.) Still: We still havent moved house. (Ainda no mudamos de casa.) Already: Shes already sent the cheque. (Ela j mandou o cheque.) O FUTURO As formas mais comuns 1. WILL INTENES a) Decidido no momento; reaes; ofertas, pedidos, promessas, ameaas etc. a) J decidido antes de falar planos pessoais. Organizado com outra(s) pessoa(s) PREVISES b) Baseado na sua opinio ou conhecimento de comportamento tpico b) Com evidncia no presente, quase acontecendo.

2. GOING TO 3. PRESENT CONTINUOUS (Opcional)

1. a) A: This is so heavy! B: Ill help you (A: to pesado! B: Vou te ajudar.) b) I think shell probably be late. (Acho que provavelmente ela vai atrasar.) 2. a) Im going to study hard. (Vou trabalhar bastante.) b) Be careful, youre going to fall! (Cuidado, voc vai cair!) 3. Im having lunch with Xuxa. Were meeting at Bobs. (Vou almoar com a Xuxa. Vamos nos encontrar no Bob.) As formas menos comuns 4. PRESENT SIMPLE 5. FUTURE CONTINUOUS (Will be + .-ing) 6. FUTURE PERFECT (Will have + pp/Will have + been + .-ing)

Horrios ou aes repetidas com freqncia. Atividade em progresso em um tempo especfico no futuro. Ao que ser completada (ou ter uma certa durao) at um tempo especfico no futuro.

4. What time does the train leave? (A que horas sai o trem?) 5. Tomorrow Ill lyving on the beach. (Amanh vou estar deitado na praia.) 6. Well have finished by nex week. (Vamos terminar at a semana que vem.) FORMAS INTERROGATIVAS Perguntas diretas Present Simple: Where DO you work? (Onde voc trabalha?) Why DOES she have a dog? (Por que ela tem um co?) (Excees: to be e to have got: How is she? (Como ela est?) - Has he got a car? (Ele tem um carro?) Past Simple: DID you buy the CD? (Voc comprou o CD?) How DID she get home? (Como ela chega em casa?) (Exceo: to be: How old were you? (Quantos anos voc tinha?) Why was he o angry? (Por que ele foi irritado?) Present/Past Continuous: What are/were you doing? (O que voc est/estava fazendo?) Is/was he playing? (Ele est/estava jogando?) Present/Past Perfect: Have you finished? (J acabou?) Has he paid? (Ser que ele paga?) Had he been waiting long? (Ele teve que esperar muito tempo?) Verbos modais: Can/could you help me? (Pode/Poderia me ajudar?) Should I call her? (Devo cham-la?) Would you believe it? (Voc acreditaria nisso?) Sujeito da frase: Who lives upstairs? (Quem mora no andar de cima?) Who ate my candy? (Quem comeu meu doce?)

Perguntas indiretas (+ a forma positiva): Do you know where the bathroom is Can/could you tell me how much it costs Id like to know why she didnt come I wish knew how many rooms there are I dont know/understand who broke the stereo I have no idea when shes coming Im not sure what the looked like I cant remember how often she goes home Ive forgotten what time the film starts

Voc sabe onde o banheiro? Pode/Poderia me dizer quanto custa? Eu gostaria de saber porque ela no veio. Desejo saber quantos quartos h.
Eu no sei/entendo quem quebrou o aparelho de som. Eu no tenho nenhuma idia de quando ela est vindo.

Eu no tenho certeza do que parecia.


No me lembro quantas vezes ela vai para casa.

Esqueci a que horas o filme comea.

Perguntas relatadas (quando voc relata o que outra pessoa j perguntou) I Mudana de tempo verbal: Present Past, Past/Present Perfect Past Perfect Will Would, Can Could II Forma positiva (como perguntas indiretas) What time are you going? He asked what time I was going. (Que horas voc vai? Ele perguntou que horas eu estava indo.) Did you buy some milk? She asked if I had bought some milk. (Voc quis comprar leite? Ela perguntou se eu tinha comprado um pouco de leite.) Outras perguntas difceis Whats he like? (Como ele ? = sua personalidade) What does he look like? (Como ele fisicamente?) How old/tall is he? (Quantos anos ele tem/Qual sua altura?) How did you get there? ((Como voc chegou l?) Can you tell me the way to ..? (Pode me dizer como chegar ?) How long does it take to ? (Quanto tempo leva para chegar em ?) How/What time did you get there? (Como/Que horas voc chegou l?) How come youre doing this exam? (Por que est fazendo esta prova?) What did you say that for? (Por que voc falou isso?) Whats the point in going? (Qual o motivo para ir?) MODO IMPERATIVO usado para fazer um pedido, convite ou para dar uma ordem. formado pelo infinitivo sem o to. Nas formas negativas usa-se dont ou do not. Ex.: Wait for me (Espera por mim.) forma afirmativa Have a good day! (Tenha um bom dia!) forma afirmativa Dont move, please! (No se mova, por favor.) forma negativa Dont smoke. (No fume.) forma negativa Let him try! (Deixe ele tentar!) forma afirmativa com o uso de let Dont let them stop. (No deixe eles pararem.) forma negativa com o uso de let

GERNDIOS e INFINITIVOS
Gerndio (o verbo + -ing)* Verbos depois de preposies: por exemplo, Im thinking of studying, Shes good at writing, I apologised for being late, We talked about not moving, Theyre interested in coming etc. (Estou pensando em estudar, Ela boa em escrever, Me pediu desculpas por chegar atrasado, Ns falamos sobre no se mover, Eles esto interessados em vir, etc.) Verbos usados como substantivo: Having children is hard work, Its profitable being politician, Working late is part of the job, Its hard not having a ar in Floripa. (Ter filhos um trabalho rduo, Ser poltico lucrativo, Trabalhar at tarde parte do trabalho, difcil no ter um carro em Floripa.) Verbos usados como adjetivo: A growing economy, falling prices, screaming children. (Uma economia crescente, preos em queda, crianas gritando.) Depois de alguns verbos, principalmente: admit, appreciate, consider, delay, deny, detest, dislike, enjoy, escape, face, feel, like, finish, give up, imagine, involve, mention, mind, miss, postpone/put off, practise, prefer, recommend, resent, risk, suggest, understand.

s vezes o verbo faz parte de uma expresso fixa: have difficulty/trouble/problems doing, cant stand/bear doing, cant help doing, spend time doing, Its (not) worth doing, Theres no point (in) doing, Its no use doing, Dont mind doing, be/get used to doing etc. Infinitivo (to + verbo) Depois de alguns adjetivos: easy to (remember), hard to (say), happy to (go), glad to (hear), sad to see), wrong to (steal), exciting to (be), funny to (think). Depois de alguns substantivos: (to make) a decision to, a promise to, an agreement to, an arrangement to, an offer to, a wish to etc. Depois de alguns verbos, principalmente: afford, agree, appear, arrange, ask, attempt, begin, care, choose, consent, decide, determine, expect, fail, forget, happen, hate, help, hesistate, hope, intend, learn, like, love, manage, mean, offer, prefer, prepare, pretend, promise, propose, refuse, seem, start, swear, want, wish. Outros verbos sao seguidos por uma pessoa/um objeto + o infinitivo, por exemplo: want someone to do (querer que algum faa). Outros verbos: advise, allow, ask, cause, command, encourage, expect, forbid, force, get, hate, help, instruct, invite, leave, like, mean, need, oblige, order, permit, persuade, prefer, recommend, remind, request, teach, tell, tempt, trouble, warn, wish, would like. Some/any/no + one/body/thing/where: somewhere to sit, nobody to help, something to eat, anyone to go with, nowhere to buy, nothing to do, anywhere to sleep etc. Gerndio ou infinitivo, com diferena de sentido Verbos demonstrando preferncias like, love, hate, prefer, cant bear/stand Gerndio (mais comum) = preferncias mais gerais: She likes reading. Infinitivo = preferncias mais especficas: I like to read in bed. Try to do (fazer um esforo): I tried to call you. (Eu tentei ligar para voc.) Try doing (experimentar): He tried hiding the evidence. (Ele tentou esconder as provas.) Need to do (precisa fazer): You need to lose weight. (Voc precisa perder peso.) Need doing (precisa ser feito): My hair needs cutting. (Meu cabelo precisa de corte.) Remember to do (esquecer/lembrar fazer antes da ao): Did you remember to feed the dog? (Voc se lembra de alimentar o co?) Remember doing (ter lembranas de ter feito algo): I remember putting the keys on the table. (Lembrome de colocar as chaves sobre a mesa.) Regret to do (se arrepender de algo no presente): I regret to tell you that (Lamento dizer-lhe que) Regret doing (se arrepender de algo no passado): She regrets not having kids. (Ela lamenta no ter filhos.) Verbos seguidos por gerndio e infinitivo, ou nenhum dos dois (Be/get) used to e (Be/become) accustomed to (estar/ficando acostumado): Im not used to living alone. (No estou acostumado a morar sozinho). Look forward to (esperar ansiosamente): I look forward to hearing from you, We look forward to seeing you again. (Eu espero ter notcias de voc, Ns esperamos o ver novamente.) Object to: He objects to waiting in line. Let (deixar): He let me use his car. (Ele me deixou usar o seu carro.) Make (fazer/obrigar): They made me retake the exam. (Eles me obrigaram a refazer a prova.) Help (ajudar): We helpd her move. (Ajudamos ela a fazer a mudana.) See/Hear (ver/ouvir): I saw you eat it. (Eu vi voc comer.) She heard me come in. (Ele me ouviu entrando.) Had better (deveria): Youd better see a doctor. (Voc devia ir ao mdico.) Would rather (preferia): Shed rather take a taxi. (Ela preferia pegar um txi.)

10

VERBOS AUXILIARES So verbos que ajudam a simplificar a lngua inglesa, usados, na maioria das vezes, em perguntas ou frases interrogativas. Em perguntas voc pode mudar o tempo verbal de uma frase simplesmente mudando o verbo auxiliar, em vez de memorizar as conjugaes dos verbos. Ex.: Do you play soccer? (Voc joga futebol?) Did you play soccer? = (Voc jogou futebol?) Will you play soccer? = (Voc jogar futebol?) Would you play soccer? = (Voc jogaria futebol?) OBS: Note que eles no tem nenhum significado na frase. Ele tambm facilita as coisas de uma outra maneira: na resposta a uma pergunta, ele substitui o verbo e todos os seus complementos Ento, se algum pergunta: Do you always go to work by car on week days? (Voc sempre vai para o trabalho de carro nos dias da semana?), a resposta pode ser, simplesmente, Yes, I do. (Do= always go to work by car on week days) Nas frases negativas acrescentado o not depois do verbo auxiliar. Veja: I do not like pizza / I don't like pizza (Eu no gosto de pizza). VERBOS MODAIS Habilidade Verbo (modal) Can Could Was/were able to

Funo Habilidade no presente e no futuro Habilidade geral no passado ou em um condicional Habilidade em uma situao especfica no passado Habilidade no passado, no realizada Conhecimento ou habilidade especfica Conseguir com alguma dificuldade

Could have done Know how to Manage to/succeed in

Exemplos I can sing, but I cand dance. Can you come tomorrow? He could speak at 18 months. If I had a car, I could take you. They were able to save him. Wash he able to get another job? I couldve gone to university. Couldnt you have called me? I know how to change a wheel. Do you know how to open it? We managed to get tickets. He secceeded in selling the house.

Pedidos e permisso Can I have the menu please? Yes, of course you can. (Posso ter o menu por favor? Sim, claro que pode. Can you tell me the way to ? (Pode dizer-me o caminho para ?) Could I send It next week? (Poderia me enviar na prxima semana?) Could you just wait a moment? (Voc poderia esperar apenas um momento?) May I take this chair, please? (Posso tirar esta cadeira, por favor?) Would you mind closing the door? (Voc se importaria de fechar a porta?) Would you mind if I closed the door? (Voc se importaria se eu fechasse a porta?) Would it be possible (for you) to close ? (Seria possvel (para voc) fechar ? Possibilidade Presente e Futuro Might/Could/May It could be him, or it might be someone else. ( possvel ser ele, ou possvel ser outra pessoa.) We might go to France, but we may just stay here. (Talvez iremos para a Frana, mas de repente a gente vai ficar aqui.) Passado Migth/Could/May + have + past participle He might have taken the car. ( possvel ser ele que levou o carro.)

11

She couldve been at the party. (Talvez ela estava na festa.) Certeza (deduo) Positivo Must (have + past participle) She must earn a good salary. (Ela deve ganhar um bom salrio.) They must have left. (Eles devem ter sado). Negativo Cant (have + past participle) That cant be easy. (No deve ser fcil.) He cant have got lost. (Ele no pode ter se perdido.) Obrigao forte e necessidade Positivo Must, Have to/Have got to I must get a new TV. (Eu devo comprar uma televiso nova.) You have to leave the room by 12h. (Voc precisa sair do quarto at 12h.) Hes got to study. (Ele tem de estudar.) Negativo Mustnt V Dont have to/Havent got to You mustnt park here. (No deve estacionar aqui.) I dont have to work today, its a holiday! (No preciso trabalhar hoje, feriado!) She hasnt got to come. (Ela no precisa vir.) Obrigao fraca e conselhos Should/Ought to (have + past participle). Should mais comum: You should/ought to stay in bed. (Voc deveria ficar na cama.) He should/ought to have paid the bill. (Ele devia ter pago a conta.) To be supposed to = regras ou obrigaes (geralmente no seguidas) Im supposed to start at 9h. (Eu devia comear s 9h.) Youre not supposed to wear jewelry. (No devia usar jias.) Neednt have + past participle = algo feito desnecessariamente. The shop was closed so we neednt have gone. (A loja estava fechada, ento no precisava ter ido.)

STATIVE and DINAMIC VERBS


Verbos que tem conotao de movimento so Dynamic Verbs. Verbos com conotao esttica (geralmente implicam em estado, condio inerte, sentimento, sensao) so Stative Verbs e no aceitam a forma progressiva ("to be loving", por exemplo, que uma forma incorreta). Veja abaixo uma pequena lista deles: Stative Verbs Dynamic Verbs adore abandon believe beg doubt die forgive fall guess grow hate help hear hip impress jump know knock love land mean look at perceive mature prefer nod presuppose play recall rain regard read see say smell sit

12

taste understand want wish

tap whisper work write

Devemos lembrar que existem muitos outros verbos, alm dessa lista e a vai uma dica pra quando voc tiver dvida de como escrever ou falar: pergunte-se se o verbo tem conotao de movimento ou de inrcia. Se for um verbo de movimento, pode flexionar (to be + ing), caso contrrio, s se lembrar de usar o gerndio.

5. CONDICIONAIS
Usamos os conditionals quando queremos dizer que uma coisa ir acontecer em funo de outra, ou seja, quando se estabelece uma condio para que certas situaes ocorram. Tipo FIRST SECOND THIRD Forma If + present + future If + past + would/could If + Past Perfect + Would/could have + past participle Uso Situaes bastante provveis SItuaes no muito provveis ou hipotticas Situaes no passado

If you read this book, youll pass the exam. (Se voc ler este livro, vai passar no exame.) Im going to scream if you touch me. (Vou gritar se voc tocar em mim.) If Brazil lost to Peru, the coach would be sacked. (Se Brasil perdesse contra Peru, o tcnico seria demitido.) I could buy a car if I had more money. (Eu poderia comprar um carro se tivesse mais dinheiro.) If had know, I wouldve called you. (Se eu soubesse, ia te ligar.) She couldve won if shed played better. (Ela podia ter ganho se tivesse jogado melhor.) Outras palavras usadas com os mesmos tempos verbais When they arrive (Quando eles chegam As soon as the film starts (Assim que o filme comea Before there are new laws + FUTURO (Antes de existirem leis novas After we sign the contract (Depois que assinar o contrato Until my boss come back (At o meu chefe voltar Even if = mesmo se: Even if the comes, he wont have any money. Whether .. or no Whether he comes or not, he wont have (Just) In case = caso /se: Take an umbrella in case it rains. Unless = a menos que: Unless he passes the exam, hell repeat the year. (A menos que ele passe no exame, vai repetir o ano.) As long as = desde que: As long as/provided you pay me back. (f) Provided (that) (Desde que voc me devolve o dinheiro.) On condtion (that) If only = se ao menos, quem me dera (expressa desejos e sonhos): (Wish) If only/I wish I could speak French. (Se ao menos eu pudesse falar Francs.)

6. A VOZ PASSIVA
To be + particpio passado (3 forma) Usada para dar mais nfase no objeto de uma frase, geralmente porque o sujeito (quem faz/fez/vai fazer) no importante, no conhecido ou porque bvio. Apenas o verbo to be muda de acordo com o tempo verbal, enquanto o particpio passado constante.

13

Present Simple: Many cars are made in Brazil. (Muitos carros so feitos no Brasil.) Continuous: Plans are being made. (Os planos esto sendo feitos.) Past Simple: I was told to call later. (Disseram-me para ligar mais tarde.) Continuous: My car was being repaired. (Meu carro estava sendo consertado.) Future: Will: You will be given a key. (Uma chave ser dada a voc.) Going to: The film is going to be shown. (O filme vai ser mostrado.) Present Continuous: The event is being held. (O evento est sendo realizado.) Perfect: A decision will have been taken. (A deciso ter sido tomada.) Present Perfect: Many books have been written. (Muitos livros foram escritos.) He has been awarded a prize. (Ele foi premiado.) Past Perfect: We realized the car had been stolen. (Ns percebemos que o carro tinha sido roubado.) Verbos modais: Your project must be handed in by Friday. (Seu projeto deve ser entregue at sexta-feira.) The TV must have been switched on. (A tv deve ser ligada.) Gerndio: He hates being told what to do. (Ele detesta que lhe digam o que fazer.) I got used to being fired. (Eu me acostumei a ser despedido.) Infinitives: Present: A decision has to be made. (A deciso tem que ser tomada.) Past: She was pleased to have been selected. (Ela estava feliz por ter sido selecionada.) Verbos com dois objetos como tell, give, show, offer e seel, comea com a pessoa: Alex was given a present. (No A present was given TO Alex.) We were shown the samples. (No The samples were shown to us.) Need + verbo com ing = passiva The house needs cleaning. = needs to be cleaned. The report needed checking = needed to be checked. Have something done = servios que precisam ser feitos por outra(s) pessoa(s) Ive just had my hair cut. Shes having her flat repainted. Get something done = conseguir fazer com dificuldade Did you get all your work finished? We got the whole house cleaned. It is said/believed/thought/expected/claimed etc. (that ) It is believed that Vikings sailed to America. Bush is thought to have avoided military service. Let no usado na voz passiva (precisa usar allowed to) They were allowed to use dictionaries. Make/help/see/hear inclui to na voz passiva: The students was made to do the homework again. He was heard to say that he was guilty. CAUSATIVE FORM (HAVE SOMETHING DONE) formado por have + objeto + particpio passado do verbo principal, tem sentido passivo e expressa uma ao que algum faz a nosso pedido ou em nosso favor, ou seja, no somos ns que realizamos a ao. Ex.: The roof of Lisas house was damaged in a storn. (O telhado da casa de Lisa foi estragado em uma tempestade.) Yesterday a workman came and repaired it. (Ontem um trabalhador veio e consertou isto.) Lisa had the rood repaired yesterday. (Lisa teve o telhado consertado ontem.) This means: Lisa arranged for somebody else to repair the roof. She didnt repair it herself. (Isto significa: Lisa chamou outro algum para consertar o telhado. Ela no consertou isto.) Veja outros exemplos: She doesnt have her nails done every week. (Ela no faz as unhas toda semana.)

14

Do you have your house cleaned every week? (Voc limpa sua casa toda semana?)

7. DISCURSO DIRETO e INDIRETO


DISCURSO DIRETO usado para reproduzir as palavras exatas do interlocutor, em citaes, conversaes nos livros, peas de teatro, relatrios, depoimentos de testemunhas, etc. Ex.: See you tomorrow, Mary said. ( -At amanh disse Maria.) OBS: Em ingls, as palavras do interlocutor so colocadas sempre entre aspas. No discurso direto, na maioria dos casos, existe mais de um verbo e eles podem ser empregues no incio, no meio ou no final das frases. Ex.: John said, I think she likes you. (Joo disse: Acho que ela gosta de ti.) I think, said John, she likes you. (Acho disse Joo que ela gosta de ti.) I think she likes you, John said. (Acho que ela gosta de ti disse Joo.) OBS: Exceto no incio de um enunciado, existem trs formas de marcar a presena de um verbo: John said. I think she likes you, he said. Said John. (said he.) essa forma j no mais utilizada. Regras de pontuao no discurso direto: - Tudo que citado colocado entre aspas (simples ou duplas), incluindo vrgulas, ponto finais, pontos de interrogao e exclamao. - Quando se marca a presena de um verbo no incio de um discurso direto, esse sempre seguido de uma vrgula e o que vem a seguir comea com letra maiscula. - Quando essa marcao no fim de um discurso direto, a vrgula vem antes da segunda aspa. - Quando a marcao aparece no meio do discurso direto, a segunda metade da frase comea com letra minscula. - Quando a frase citada termina com ponto de interrogao ou exclamao, no se utiliza vrgula. - Quando cita-se duas ou mais frases como um todo, o verbo includo no discurso direto seguido de ponto final e a segunda parte comea-se com letra maiscula. - H ainda o caso de uma citao dentro de outra. Nesse caso so empregadas as aspas duplas e as aspas simples. Ex.: When she saw me she said, I need you, John told me. ou When she saw me she said, I need you, John told me. - Quando ela me viu disse <Preciso-te> - disse-me Joo. OBS: Quando cita-se nomes de filmes, livros, jornais, etc. esses so empregues em itlico em vez de aspas. Ex.: Have you ever read The Times? he asked. (Have you ever read The Times? he asked.) ou Have you ever read The Times? he asked. (Have you ever read The Times? he asked.) - J alguma vez leste <The Times>? perguntou ele. DISCURSO INDIRETO usado para relatar algo dito por outra pessoa. O verbo que introduz o discurso indireto pode estar no presente ou no passado, mas usado com mais freqncia no passado. Na passagem do discurso direto para o indireto, os pronomes normalmente sofrem alterao. Ex.: John said, Ill lend you my book, Mary. (discurso direto) John told Mary (that) he would lend her his book. (discurso indireto relatado por outra pessoa) John said (or told me) (that) he would lend me his book. (discurso indireto relatado por Mary) I told Mary (that) I would lend her my book. (discurso indireto relatado por John) Quando o verbo que introduz o discurso indireto est no passado, existe dois mtodos diferentes para escolher o tempo verbal no enunciado relatado: mtodo gramatical e mtodo contextual. - Mtodo gramatical Nesse mtodo, o tempo verbal empregue no discurso direto recua para o passado Ex.: I need a holiday. (discurso direto) He said (that) he needed a holiday. I saw her. (discurso direto)

15

He said (that) he had seen her. - Mtodo contextual Nesse mtodo, o tempo verbal no enunciado depende do ponto de vista do relator, que se baseia nos fatos da situao como ele os v na altura em que os relata. Imagine que encontrou o Joo s 11 horas da manh e ele contou como seria o seu dia: I arrived at work at eight. (- Cheguei ao trabalho s oito.) I will have lunch at one. (- Vou almoar uma.) I will leave work at five. (- Vou sair do trabalho s cinco.) Agora voc encontra com uma amiga as 3 horas da tarde e ela pergunta-lhe de Joo. Poder dizer o seguinte: He said (that) he had arrived at work at eight. (Ele disse que tinha chegado ao trabalho s oito.) He said (that) he would have lunch at one. (Ele disse que ia almoar uma.) He said (that) he will leave work at five. (Ele disse que vai sair do trabalho s cinco.) Quando o verbo est no presente, o tempo verbal usado no enunciado o mesmo que se utiliza no discurso direto. Ex.: I can swim. (discurso direto) He says (that) he can swim (discurso indireto) No, I didnt. (discurso direto) He says (that) he didnt. (discurso indireto) REPORTED SPEECH usado para relatar o que aconteceu em um dilogo, sem o uso das aspas ou sem fazer citaes, assim o texto fica mais bonito, mais organizado e fcil de ler. Ex.: He said: I dont want to go to school. (discurso direto) He said he didnt want to go to school. (reported speech) Para relatar uma frase que foi dita por algum no passado, usamos um verbo introdutrio, como say, tell, explain, em sua forma no passado (said, told, explained), e depois a frase dita, com as devidas alteraes de acordo com a tabela abaixo: Discurso Direto Reported Speech Example Simple Presente Simple Past He said: "I want some oranges." He said he wanted some oranges. Presente Continuous Past Continuous They said: "We are studying hard." They said they were studying hard. Simple Past Past Perfect She said: "I needed you, but uou weren't here." She said she had needed him, but he hadn't been there. Past Continuous Past Perfect Continuous Tom said: "I was talking to Mary." Tom said he had been talking to Mary. Present Perfect Past Perfect They said: "We've worked together." They said they had worked together. Going to Future was/were going to I said: "I'm going to visit Jim"! I said I was going to visit Jim" Must Had to She told me: "I must hurry up." She told me she had to hurry up.

8. PHRASAL VERBS
So verbos de duas ou mais palavras, geralmente empregues em linguagem informal. Ex.: blow up (explode/explodir), put out (extinguish/apagar). Formam-se da seguinte maneira: VERB + PREPOSITION = get over (an illness) restabelecer-se de (doena) VERB + ADVERB = set off partir (em viagem) VERB + ADVERB + PREPOSITION = put up with (something) suportar, tolerar (algo) Deve-se considerar cada uma das combinaes dos phrasal verbs como um todo. No necessrio distingui-las para os aplicar, mas sim saber se ele um verbo transitivo ou intransitivo. Pode-se ainda combinar a parte que corresponde ao verbo com preposies ou advrbios diferentes para formar novos phrasal verbs. Ex.: put about (espalhar rumores), put aside (pr de lado, pr de parte), put back (repor, atrasar)

16

Cada um deles pode ter mltiplos significados, como put out apagar (fogo), estender (a mo), anunciar notcias), espalhar (rumores), incomodar (algum), deslocar (parte do corpo), fazer-se (ao mar), etc. Os transitivos dividem-se em separveis e inseparveis: * Separveis (2 palavras/2 formas): expressam-se de duas formas. Quando o complemento direto no for um pronome, poder vir antes ou depois da preposio/advrbio, embora essa regra no seja geral. Ex.: Take your coat off (Tire o casaco!), Take off your coat! (Tire o casaco!) Se o complemento direto for um pronome, este vem sempre antes da preposio/advrbio. Ex.: Take it off! (Tire-o!), I told him off.(Eu ralhei com ele.) Separaveis (2 palavras/1 forma): o complemento direto (incluindo os pronomes) vem sempre antes da preposio/advrbio. Ex.: She let them down. (Ela desiludiu-os.), They didnt let them enter. (Eles no os deixaram entrar.) * Inseparveis (2 ou mais palavras/1 forma): o complemento direto vem sempre depois da preposio/advrbio. Ex.: Look after my baby, please. (Cuido do meu beb, por favor.), The thief go away with the money (O ladro conseguiu fugir com o dinheiro.) Phrasal Verb BE be about be after be down with be in on

estar por lugares estar atrs de (algum) estar de cama com (doena) estar envolvido em (algo)

intransitivo transitivo transitivo transitivo

be off be on to be over be up for be up to be up to2

ir-se embora suspeitar, desconfiar (de algum) acabar, terminar estar disponvel para (venda) estar a querer, estar a tramar caber a algum decider (algo)

intransitivo transitivo intransitivo transitivo intransitivo transitivo

Is John about? O Joo est por a? The police are after him. A polcia est atrs dele. He is down with flu. Ele est de cama com gripe. I think John should be in on this conversation. Penso que Joo devia participar nesta conversa. Im off. See you tomorrow. Vou-me embora. At amanh. The police are on to him. A polcia suspeita dele. The party is over. A festa acabou. The house is up for sale. A casa est venda. What is he up to? O que ele est querendo / a tramando? It is up to you to decide whether to pass or fail the student. Cabe a si decidir de passar ou reprovar o aluno.

Phrasal Verb BREAK The thief tried to break away from the guard as he was being taken to anoter jail. O ladro tentou escapar-se do guarda quando ele estava sendo transferido para outra priso. The fireman broke down the door and managed to get in to save the babys life. O bombeiro arrombou a porta abaixo e conseguiu entrar para salvar a vida do beb. Someone broke in through the window and stole the paiting. Algum entrou pela janela e roubou o quadro. Theyve just broken of their engagement. Eles acabaram de romper o noivado. He broke out in a rash. Apareceu-lhe uma irritao na pele.

break away (from)

libertar-se, escaper-se

intransitivo

break down

deitar abaixo, arrombar

transitivo

break in break off break out

forar uma entrada terminar, acabar, romper (noivado, acordo, etc.) aparecer coberto de (manchas, etc.)

intransitivo intransitivo/ transitivo intransitivo

17

break through break up

transport, atravessar, penetrar separar-se (amigos, namorados, sociedade)

intransitivo/ transitivo intransitivo

The demonstrators broke through the police cordon. Os manifestantes furaram o cordo policial. John broke up with his girlfriend. O Joo separou-se da sua namorada.

Phrasal Verb COME come about acontecer, suceder come across encontrar por acaso

intransitivo transitivo

come after

perseguir

transitivo

come apart

desfazer-se

intransitivo

come back

responder, retorquir ralhar repreender entrar, chegar herdar ter lugar, ocorrer, realizar-se aparecer coberto de (manchas, etc.) fazer uma visita up defrontar-se (dificuldades) com com,

intransitivo

come down come in come into come off

transitivo intransitivo transitivo intransitivo

come out in come over come against

transitivo intransitivo transitivo

How did the accident come about? Como que aconteceu o acidente? I came across some old letters yesterday. Encontrei umas cartas velhas ontem. As I was walking in the park, a dog came after me for no apparent reason. Enquanto eu estava caminhando no parque, um co comeou a perseguir-me sem nenhuma razo aparente. The table hadnt been assembled properly. Thats why it came apart. A mesa no tinha sido montada corretamente. por isso que ela se desprendeu. She came back at me ith an impolite question. Ela respondeu-me com uma pergunta grosseira. The whole thing came down to nothing. What waste of time! Tudo resumiu-se a nada. Que perda de tempo! He has just come in. Ele acabou de entrar/chegar. She cam into a fortue hen her aunt died. Ela herdou uma fortuna quando a tia dela morreu. The meeting will come off a week today. A reunio se realizar daqui a uma semana. He came out in a ras after drinking some milk. Apareceu-lhe uma irritao na pele depois de beber leite. John came over the other day to see me. O Joo veio no outro dia para me ver. She came up against a number of problems. Ela defrontou-se com uma srie de problemas.

Phrasal Verb GET get about mexer-se, pr-se a fazer algo arranjar-se, governorget along se, desenrascar-se ir-se embora, sair get away get back at vingar-se (algum) regressar a casa acostumar-se a, adquirir o hbito de ter relaes ntimas com (algum) continuar com algo (aps interrupo) de

intransitivo

intranstivo

intransitivo transitvo

get in

intransitivo

get into get off with get on with

transitivo transitivo transitivo

Despite her age, she still gt about quite a lot. Apesar de sua idde, ela ainda se mexe bastante. Dont worry. I can get along fine without his help. No te preocupes. Consigo desenrascar-me sem a ajuda dele. Sorry, I couldnt get way earlier. I had a lot of work in the office. Desculpe, eu no pude sair mais cedo. Tive muito trabalho no escritrio. One of thse days Im going to get back at him. Um dia destes vou vingar-me dele. My children were already in bed whe I got in. Os meus filhos j estavam dormindo quando cheguei em casa. Dont get into the habit of smoking. Its ba for your health. No adquiras o hbito de fumar. Faz mal sade. Mary got off with John at the party. A Maria teve relaes ntimas com o Joo na festa. Come on! Get on with your work! Vamos! Continua com o teu trabalho!

18

get over

restabelecer-se, recuperar-se doena, etc.)

(de

transitivo

get round get to get up

persuadir, convencer irritar levantar(-se)

transitivo transitivo intransitivo

Dont worry about her. Shell get over the shock of losing her husband. No se preocupes com ela. Ela vai superar o choque em relao perda do seu marido. I finaly manged to get round her to lend me her car. Consegui finalmente convenc-la a emprestar-me o seu carro. Dont let her get to you. No deixe ela irritar voc. What time do you usually get up? Que horas voc costuma levantas?

Phrasal Verb GIVE give away trair, denunciar transitivo Dont give me away. I dont want anybody to know about this. No me denuncie. No quero que ningum saiba disso. Give your exam papers in before you leave. Entreguem suas provas antes de sarem. The engine of the plane gave out. O motor do avio parou de trabalhar. You should give up smoking. Voc deveria deixar de fumar.

give in give out give up

entregar deixar de trabalhar, parar (motor) desistir de, deixar de

transitivo intransitivo transitivo

Phrasal Verb GO espalhar-se doena) (rumor, intranstivo The rumour is going abouth that our teacher is retiring next mont. Est espalhando-se o boato de que o nosso professor vai se aposentar no prximo ms. I dont like doing this that go against my principles. Eu no gosto de fazer as coisas que vo contra os meus princpios. Suddenly the animal went at him. Subitamente o animal atacou-lhe. His speech has gone down well with the crowd. O seu discurso foi bem recebido pelo pblico. Ill o for the roast beef. Eu escolho o rosbife. The lights went off. Apagaram-se as luzes. Please go on. Continue por favor. John is not in. He has gone out. O Joo no est em casa. Ele saiu. The road was blocked. We had to go round. A estrada estava bloqueada. Tivemos que fazer um desvio. In my opinion, this tie and the shirt dont go together. Na minha opinio, esta gravata e a camisa no combinam. I had to go without breakfast because I was late for work. Eu tive que sair sem o caf da manh porque estava atrasado para o trabalho.

go about

go against go at go down go for go off go on go out

ir contra, ser desfavorvel a, ser contrrio a atacar se acolhido, recebido escolher apagar-se continuar sair ser

transitivo transitvo intransitivo transitivo intransitivo intransitivo intransitivo

go round

fazer um desvio

intransitivo

Go together

combinar

intransitivo

Go without

passa sem

transitivo

Phrasal Verb HOLD deixar que algo hold against negative influencie a opinio de hold back hesitar conservar a distancia, impedir o

transitivo intransitivo

They may hold your criminal record against you. Eles podem influenciar seu registro criminal contra voc. The man held back, not knowing what to do. O homem hesitou, sem saber o que fazer. We managed to hold off the enemy.

hold off

transitivo

19

hold on hold out

avano de aguentar, manter-se firme, persistir oferecer (esperana, possibilidade to fazer com que (algum) cumpra (algo) aprovar, concordar com

intransitivo transitivo

hold (sth)

(sb)

transitivo transitivo

hold with

Conseguimos impedir o avano do inimigo. Try to hold on for a while. Im going to get some help. Tente aguentar por algum tempo. Eu vou buscar ajuda. The job holds out prospects of promotion. O emprego oferece possibilidade de promoo. Her father promised to buy her a doll. Now she wants to hold him to it. O pai dela prometeu-lhe comprar uma boneca. Agora ela quer fazer com que ele cumpra a sua promessa. I dont hold with letting children see violent films. No concordo em deixar as crianas verem filmes violentos.

9. SUBJECT-QUESTIONS
So frases interrogativas que perguntam pela identidade do sujeito. No existe inverso de verbo e a ordem das palavras igual a de uma frase declarativa. Ex.: Someone stole my bicycle. (frase declarativa) Who stole my bicycle? (subject-question) Consideremos a seguinte frase na interrogativa direta do tipo subject: He asked, Who broke the vase? Para convert-la em interrogativa indireta: - comece por: He asked - introduza o pronome me (opcional): He asked (me) - adicione o advrbio who da interrogativa indireta: He asked (me) who - recue o tempo verbal para o passado: He asked (me) who had broken -adicione o resto da frase da interrogativa indireta: He asked (me) who had broken the vase - termine a frase com ponto final: He asked (me) who had broken the vase. OBS: Na interrogativa indireta no so usados recursos de pontuao (vrgula, aspas e interrogao). Mais exemplos: He asked, Whose car was stolen? (He asked whose car had been stolen.) He asked, How many of you can lift this box? (He asked how many of us coult lift that box.)

10. ADVRBIOS
So empregues para modificar ou dar mais informao sobre verbos, adjetivos, outros advrbios e/ou frases. (verbo) + ADVERB He ate quickly. (Ele comeu depressa.) She drank slowly. (Ela bebeu devagar.) ADVERB + (adjetivo) It is fairly hot today. (Est bastante quente hoje.) The book is reasonably cheap. (O livro razoavelmente barato.) ADVERB + (outro advrbio) He ate fairly quickly. (Ele comeu bastante depressa.) She did the test fairly well. (Ela fez o teste bastante bem.) ADVERB + (frase) Frankly, I dont trust him. (Sinceramente, no confio nele.) Politically, he is finished. (Politicamente, ele est acabado.) Os advrbios so divididos em vrias categorias: intensidade, freqncia, modo, lugar e tempo. * Advrbios de intensidade servem para modificar ou dar mais informao sobre: adjetivos, outros advrbios e verbos. So eles:

20

almost (quase), barely (mal, apenas), fairly (razoavelmente, bastante), hardly (mal, apenas), nearly (quase), quite (bastante), rather (bastante, um tanto), scarcely (mal, quase no), very (muito), absolutely (absolutamente), really (realmente) etc. Ex.: The room is nearly empty. (A sala est quase vazia.) She did the exercises fairly quickly. (Ela fez o exerccio bastante depressa.) He hardly understans me. (Ele mal me conhece.) * Advrbios de freqncia (definida e indefinida). Definida: once (a day) uma vez (por dia), twice (a week) duas vezes (por semana), three times (a month) trs vezes (por ms), several times (a year) vrias vezes (por ano), hourly (de hora em hora), every hour, every day de hora em hora, todos os dias, on Saturdays, on Sundays nos sbados, nos domingos, on weekdays nos dias da semana, at weekends nos fins de semana. Esses advrbios posicionam-se, normalmente, no fim das frases. Ex.: John plays tennis every day. (O Joo joga tnis todos os dias.) I visit my dentist twice a year. (Vou ao dentista duas vezes por ano.) Indefinida: always (sempre), nearly always, almost always (quase sempre), usually, normally (normalmente), sometimes, occasionally (s vezes, ocasionalmente), never (nunca), again and again (repetidas vezes), now and again, now and then (de quando em quando), at times (por vezes). Esses advrbios posicionam-se de vrias formas: - No incio ou fim da frase para nfase Ex: Occasionally, you hear noises in the basement. (Ocasionalmente, ouve-se barulho na cave.) Do you see her often? (Costumas v-la muitas vezes?) - No incio da frase para nfase (advrbio + imperativo) Ex.: Always brush your teeth. (Escove sempre os dentes.) Never forget what I said. (Nunca esqueas o que eu te disse.) - No meio da frase antes do verbo se este tiver a forma de uma nica palavra. Ex.: I always get up early. (Eu sempre levanto-me cedo.) I never said that. (Eu nunca disse isso.) - No meio da frase a seguir ao verbo BE Ex.: He is always late. (Ele chega sempre tarde.) The weather is sometimes unpredictable. (O tempo s vezes imprevisvel.) - No meio da frase a seguir ao primeiro verbo auxiliar (exceto os verbos auxiliares used to, have to e ought to, que vm somente depois do advrbio) Ex.: He is always complainig. (Ele est sempre a queixar-se.) Your request will never be granted. (O seu pedido nunca ser aceito.) - No meio da frase a seguir ao sujeito nas interrogativas Ex.: Do you always come here? (Vens sempre aqui?) Does she usually study? (Ela estuda normalmente?) - Antes do verbo auxiliar nas respostas curtas Ex. She always works hard. (Ela sempre trabalha muito.) But I often do! (Mas eu vou muitas vezes.) * Advrbios de modo formam-se juntando um sufixo a um adjetivo, substantivo, ou advrbio de direo. - adjetivo + LY: beautifully (lindamente, maravilhosamente), carefully (cuidadosamente), easily (facilmente), quickly (depressa, rapidamente), slowly (lentamente, vagarosamente). - substantivo + WISE: crosswise (transversalmente), lengthwise (longitudinalmente), moneywise (em relao a dinheiro), timewise (em relao a tempo), clockwise (no sentido dos ponteiros de um relgio) - substantivo/advrbio de direo + WARD(S): upward(s) (para cima), downward(s) (para baixo), northward(s) (em relao ao norte), eastward(s) (em relao a leste). Outros advrbios de modo formam-se combinando adjetivos terminados em ly com way, manner ou fashion: - adjetivo terminado em ly + WAY/MANNER/FASHION: She spoke in a motherly way. (Ela falou de um modo maternal.) They walked in an ordely fashion. (Eles caminharam de uma forma ordeira.) Existem advrbios de modo com significado e forma iguais aos seus adjetivos correspondentes: Adjetivo = advrbio He works fast. (Ele trabalha depressa.) He came last. (Ele chegou em ltimo lugar.)

21

Existem advrbios de modo em forma de sintagmas preposicionais: He went home by train. (Ele foi para casa de comboio.) We met by chance. (Encontramo-nos por acaso.) Os advrbios de modo, podem aparecer depois do verbo, depois do verbo + complemento, ou ainda entre o sujeito e o verbo. Ex.: She drives carefully (Ela guia cuidadosamente.) He rad the letter quickly. (Ela leu apressadamente a carta.) I furiously slammed the door. (Fechei furiosamente a porta.) Quando no incio da frase, caracteriza dramatizao: Silently, the soldier crept behind the enemy lines. (Silenciosamente, o soldado rastejou atrs das linhas inimigas.) * Advrbios de lugar podem ser: - Palavras tais como: here (aqui, para c), there (a, ali, l, acol), upstairs (em cima, no andar superior, para o andar superior), somewhere (algures, em algum lugar, para algum lugar), anywhere (em qualquer parte, em parte nenhuma), everywhere (em toda a parte, toda a parte). - Palavras que podem igualmente ser preposies: in (dentro, para dentro), up (em cima, para cima), out (fora, l fora, fora de casa), over (por cima), round (em volta, em crculo, em roda, em torno), behind (atrs, para trs) - Sintagmas preposicionas: at school (na escola), to school ( escola), in hospital (no hospital), on the right ( direita), at my aunts (em casa da minha tia), from London (de Londres) Esses advrbios nunca so colocados entre o sujeito e o verbo. Quando houver mais de um advrbio de lugar numa mesma frase, comea-se pelo menor seguido do prximo advrbio, o maior (John lives in a smal flat in London. / O Joo mora num pequeno apartamento em Londres.) Quando for empregue com outros advrbios (modo, tempo, etc.), deve vir depois do advrbio de modo e antes do advrbio de tempo. (The man waited silently in the room all afternoon. / O homem esperou na sala em silncio toda a tarde.) Quando for empregue com verbos de movimento (go, drive, fly, walk), vir depois do verbo e antes dos outros advrbios. (She flew to London by British Airways last summer. / Ela foi de avio para Londres atravs da British Airways no vero passado.) Quando no incio de uma frase, o verbo dever ser seguido pelo sujeito, desde que este no seja um pronome. (Down fell our books. / L caram os nossos livros.) Ainda no incio da frase, sendo o sujeito um pronome, o verbo vem depois do sujeito. (Down they fell. / L caram eles.) * Advrbios de tempo (definido, indefinido e de durao) Definido: tomorrow (amanh), next month (no prximo ms), at seven oclock (s sete horas), in December (em Dezembro), at Christmas (no Natal), on Christmas Day (no dia de Natal). No incio da frase: At te oclock Mr. Brown will be makig an announcement. (s dez horas o Sr. Brown ir fazer uma comunicao.) No fim da frase (mais comum): See you at Christmas. (At Natal.) Indefinido: then (ento, depois, em seguida), soon (brevemente, (dentro) em breve), yet (ainda, at agora), lately (ultimamente), eventually (finalmente, por fim), recently (recentemente, ultimamente), suddenly (de repente, inesperadamente). No incio da frase: Suddenly he appeared. (De repente ele apareceu.) No meio da frase: He has already left. (Ele j saiu.) No fim da frase: I havent finshed it yet (Ainda no acabei.) Durao (em forma de sintagmas) preposicionais: since yesterday (desde ontem), from Monday to Friday (de segunda a sexta-feira), until Wednesday (at quarta-feira), by seven oclock (antes das sete horas). No fim da frase: I have known her since 1990. (Conheo-a desde 1990.) I work from Monday to Friday. (Trabalho de segunda a sexta-feira.) Os advrbios podem apresentar os graus comparativos e superlativos. Advrbios de uma nica slaba: Advrbio Comparativo Superlativo Exemplo fast faster fastest She walks faster than me. (Ela anda mais depressa que eu.)

22

high slow early

higher slower earlier

highest slowest earliest

He climbed higher this time. (Ele trepou mais alto dessa vez.) I drive slower than you. (Eu guio mais devagar que tu.) She left earlier this morning. (Ela saiu mais cedo esta manh.)

Advrbio de duas ou mais slabas: Advrbio Comparativo Superlativo fluently more fluently most fluently quickly slowly more quickly more slowly most quickly most slowly

Exemplo She speaks Englih more fluently these days. (Ela fala mais fluentemente ingls hoje em dia.) She works far more quickly than me. (Ela trabalha muito mais depressa que eu.) Could you drive more slowly? (Podias conduzir mais devagar?)

Advrbios com formas de comparativo e superlativo irregulares: Advrbio Comparativo Superlativo Exemplo badly worse worst He sings a lot worse these days. (Ele canta muito pior hoje em dia.) much more most I drive more than I used to. (Eu conduzo mais do que dantes.) far farther/further farthest/furthest I cant walk any farther. (No consigo andar mais longe.) Outras formas de comparativo dos advrbios: THE + (comparativo) + THE + (comparativo) The more clearly you speak, the more I will understand you. (Quanto mais clara voc for, mais irei te entender.) AS + (advrbio) + AS He speaks English as fluently as his brother. (Ele fala to fluentemente ingls como o seu irmo.) NOT ... AS / SO + (advrbio) + AS He doesnt cook as well as his mother. (Ele no cozinha to bem como a sua me.) (comparativo) + AND + (comparativo) He is studying less and less these days. (Ele estuda cada vez menos hoje em dia.)

11. NUMERAIS
* Cardinais indicam uma quantidade:
0 nought, zero 1 one 2 two 3 three 4 four 5 five 6 six 7 seven 8 eight 9 nine 10 - ten 11 eleven 12 twelve 13 thirteen 14 - fourteen 15 - fifteen 16 sixteen 17 seventeen 18 eighteen 19 nineteen 20 twenty 21 twenty-one 22 twenty-two 23 twenty-three 24 twenty-four 25 twenty-five 26 twenty-six 27 twenty-seven 28 twenty-eight 29 twenty-nine 30 thirty 31 thirty-one 32 thirty-two 33 thirty-three ..... 40 forty 50 fifty 60 sixty 70 sevnty 80 eighty 90 ninety ..... 100 one hundred 101 one hundred and one 102 one hundred and two ..... 1,000 one thousand
1,001 one thousand and one 1,002 one thousand and two

...... 2,000 two thousand 3,000 three thousand ..... 1,000,000 one milion 2,000,000 two milion

OBS: Quando se sabe o nmero exato, diz-se os seguintes no singular: hundred, thousand, million, billion, trillion, etc. Quando no se sabe o nmero exato, diz-se os seguintes no plural: hundreds, thousands, millions, billions, trillions, etc. Ex.: hundreds (of people) centenas (de pessoas) * Decimais esquerda do ponto, l-se o nmero como inteiro e direita do ponto, lem-se os algarismos individualmente. 0.125 zero point one two five 4.051 four point zero five one * Ordinais so adjetivos que indicam ordem:
1st first 2nd second 3rd third 4th fourth 12th twelfth 13th thirteenth 14th fourteenth 15th fifteenth 23rd twenty-third 24th twenty-fourth 25th twenty-fifth 26th twenty-sixth ..... 40th fortieth 50th fiftieth 60th sixtieth ..... 1,000th one thousandth 1,001st one thousand and first 1,002nd one thousand

23

5th ffth 6th sixth 7th seventh 8th eighth 9th ninth 10 tenth 11 - eleventh

16 sixteenth 17 seventeenth 18 eighteenth 19 nineteenth 20 twentieth 21st twenty-first 22nd twenty-second

27th twenty-seventh 28th twenty-eighth 29th twenty-ninth 30th thirtieth 31st thirty-first 32nd thisty-second 33rd thirty-third

70th seventieth 80th eightieth 90th ninetieth ..... 100th one hundredth 101st one hundred and fist 102nd one hundred and second

and second ..... 2,000th two thousandth 3,000th three thousandth ..... 1,000,000th one millionth 2,000th two millionth 3,000th tree millionth

Esses numerais, colocam-se sempre antes dos substantivos: Ex.: the First Wordl War (a Primeira Guerra Mundial) the third chapter (o terceiro captulo) Os nomes de soberanos escrevem-se em numerao romana, mas verbalmente so empregues com numerais ordinais. Ex.: George IV George the Fourth (Jorge Quarto) Louis XIV Louis the Fourteenth (Luis Quatorze) * Fraes - Simples - a half, one half 1/3 a third, one third -a quarter, one quarter, a fourth, one fourth 1/8 one eighth 1/12 one twelfth 2/3 two thirds 7/10 seven tenths - Complexas 12/35 twelve over thirty-five 32/115 thirty-two over one hundred and fifteen 43/235 forty-three over two hundred and thirty-five - Nmeros inteiros e fraes 3 - three and a half 7 - seven and a quarter 12 - twelve and three quarters * Telephone Numbers lem-se os algarismos individualmente com pausas aps grupos de trs ou quatro nmeros. 309 704 635 three zero nine, seven zero four, six three five 369 4022 three six nine, four zero two two * Time Uma das formas mais comuns de dizer as horas : 6.00 six oclock 6.30 six thirty 6.05 six oh five 6.35 six thirty-five 6.10 six tem 6.40 six forty 6.15 six fifteen 6.45 six forty-five 6.20 six twenty 6.50 six fifty 6.25 six twenty-five 6.55 six fifty-five Outra forma, com o uso de after e to: 6.05 five after six 6.10 tem after six 6.15 a quarter after six 6.20 twenty after six 6.25 twenty-five after six 6.35 twenty-five of seven 6.40 twenty of seven 6.45 a quarter of seven 6.50 ten of seven 6.55 five of seven

* Datas O ms vem sempre antes do dia, tanto na forma abreviada como por extenso e sempre escrito com letra mauscula e a virgula empregue depois do dia, que lido como nmero ordinal. Ex.: 6.15.00 ou 6-15-00 ou 6/15/00 (June 15,2000)

24

* Idades - Existem vrias formas de dizer a idade e uma pessoa - BE + (NUMBER) I am twety-five. (Tenho vinte e cinco anos.) - BE + (NUMBER) + YEARS OLD I am twenty-five years old. (Tenho vinte e cinco anos.) - BE + (NUMBER) + OF AGE He is twenty of age. (Ele tem vinte anos de idade.) (NUMBER) YEAR-OLD a ten-yar-old boy. ((um rapaz de dez anos.) - AT/BY/BEFORE/UNDER THE AGE OF (NUMBER) He emigrated to Australia at the age of fifteen. Ele migrou para Austrlia aos quinze anos.) - AGED + (NUMBER) (adjtivo) Hes ot two children, aged five and seven. (Ele tem duas crianas, uma de cinco anos e outra de sete.) - IN HIS/HER (EARLY/MID/LATE) IES (13-19) in his teens (entre 13 e 19 anos) - (NUMBER) + YEARS OF AGE Applicants under twenty-five years of age are not eligible for thepost. (No se aceitam candidatos com menos de vinte e cinco anos de idade para cargo.) - BE + (NUMBER) + MONTHS OLD My son is three months old. (O meu filho tem trs meses de idade.) - (NUMBER) + ISH She looks twentyish. (Ela parece ter cerca de vinte anos.) Quando a idade se refere a coisas, YEARS OLD sempre empregue. * YEARS Os anos (civis) lem-se da seguinte forma: AA00 AA0A 1500 fifteen hundred 1506 fifteen oh six 1600 sixteen hundred 1601 sixtee oh one 1700 seventeen hundred 1703 seventeen oh three 1800 eighteen hundred 1807 eighteen oh seven 1900 nineteen hundred 1909 nineteen oh nine A000 2000 (the year) two thousand 3000 (the year) three thousand A00A
2001 two thousand and one 2002 two thousand and two 2005 two thousand and five
2007 two thousand and seven

AAAA 1526 fiteen twenty-six 1631 sixteen thirty-one 1753 seventeen fifty-three 1897 eighteen ninety-seven 1999 nineteen ninety-nine AAAA 2011 twenty eleven 2015 twenty fifteen
2025 twenty twenty-five

AA00
2100 twenty-one hundred 2200 twenty-two hundred 2300 twenty-three hundred

2150 twenty-one fifty

12. PREPOSIES divididas em 4 categorias


Movimento TO: de um lugar para outro, deslocamento: go to, come to, bring to, take to, drive to, walk to, get into, send to e muitos outros. Existem duas excees principais: I Movimento para lugares gerais: go home, come here, drive there, run up/downstairs take in/outside, go downtown, go away II Verbos formados com Go + -ing: go shopping, go sightseeing, go swimming, go surfing, go skiing, go skating, go jogging, go riding, go camping Lugar AT: Prdios pblicos, instituies at school/university/college, at work/at the office at the supermarket/shopping centre, at the cinema/theatre, at the bank/restaurant/post office etc. Excees: be in hospital/prison (mais permanncia). Eventos pblicos, profissionais ou sociais at a meeting, at party, at a show, at a barbecue, at a conference, at a trade far, at an event etc. Pontos especficos: at the top/bottom (of the hill), at the side (of the road), at the front/back (of the class), at the edge (of the fields). IN: Dentro de um espao maior in (side) a box, in the cupboard, in the kitchen in Rua Tiradentes, in So Paulo, in Parana, in Brazil, in South America, in the world s vezes, possvel usar incom prdios ou eventos pblicos, quando quer enfatizar algo dentro do espao fsico, como: the desk in my office, smoke in the school, hot in the bank etc. ON: Cobrir uma area ou superfcie on the table (mas: sit at the table), on a chair, on page 30, on TV/video/the screen/the internet/a computer, on the corner (of the street), on the beach (na areia), on the side (of the truck), on the bottom (of the sea), on the back (of a t-shirt), on top (of the wardrobe). Outras: on the radio, on 5th Avenue, on a farm Tempo

25

AT: Horas e momentos at 9.30, at midnight, at the moment, at the same time, at the beginning, at the end Noite/fim de semana/celebraes at night, at the weekend, at Christmas/Easter/New Year ON: Dias e datas on Monday, on 12th June, on my birthday, on New Years Eve IN: Perodos maiores que um dia in April, in the winter, in 2002, in the 1960s, in the 12th century Partes do dia in the morning/afternoon/evening Daqui a - in 5 minutes, in 2 years (time/from now) FOR/SINCE: Perodo de tempo for 25 minutes, for 3 months, for a long time Quando comeou since 8.00, since 1989, since the beginning UNTIL/BY: At + tempo Were staying until Thursday At + prazo You have to finish by Tuesday Combinaes de preposies com outras palavras Preposio + substantivo on holiday/vacation, on a trip/tour/cruise, on business, on the phone, on foot, on the way, on a diet, on fire, on time, on behalf of, on sale, in the world, in cash, in bed, in favour of, in particular, in town, at home, at last, at present, by cheque/credit card, by mistake, by + transporte, by chance, by mail, out of date, out of work, out of breath, go for a walk/drink/swim/drive, under control etc. Adjetivo + preposio interested in, afraid/frightened/terrified of, worried about, good/bad at, similar to, different from/to, married to, tired of, fed up with, covered in etc. Verbo + preposio think of doing (pensar em fazer), think about (pensar em algo), ask for, ait for, depend on, happen to, dream about, hear of (de algo/algum), hear about (de um acontecimento), apologise for, insist on, accuse someone of, explain/complain to sb about st, spend money on, remind sb of etc. Outra combinao de verbo e preposio acontece com os chamados phrasal verbs, verbos cujo sentido idiomtico (diferente do sentido literal), por exemplo: bring up (crier), look for (procurar), put out (apagar), speak up (falar mais alto). As principais diferenas entre FOR e TO FOR Fazer algo para outra pessoa, para o benefcio de ou para ajudar algum: Ill open the door for you. He bought a dog for his son. Please fill in the form for me. Opinio pessoal e emoes, como elas nos afetam: For me, Pele is the best player of all time. Money is so important for him.

TO Quando faz parte do verbo, expressando: a) Movimento: go, come, take etc. b) Transferncia: give, pay, lend, send, offer, show. He gave the dog to his son. I sent a postcard to my mum. O objetivo ou motivo para fazer algo: I write books to make money. To leave a message, press 3.

13. AFIXOS: PREFIXOS e SUFIXOS


So elementos lingusticos acrescentados a uma palavra para produzir uma forma flexionada ou derivada. Podem vir no incio (prefixo) ou no final (sufixo) da palavra. Veja abaixo alguns PREFIXOS: ante-: in front of or before in space / prior to or before in time antediluvial, antediluvian, antepenultimate bi: two times biannual, bifacial ex-: former ex-mayor, ex-boyfriend Il-: contrary illegible, illegitimate ir-: without irrelevant, irresistible non-: absence of nonliteral, nonliving over-: excessive overcrowded, overload post-: coming after - postwar sub-: inferior subclass, subartic under-: inferior underline, undersea ante-: em frente a ou antes no espao / mais cedo ou antes no tempo antediluvial, antediluviano, antepenltimo(a) bi: duas vezes bianual, bifacial ex-: anterior ex-prefeito, ex-namorado il-: contrrio ilegvel, ilegtimo ir-: sem irrelevante, irresistvel no: ausncia de no-literal, no-vivo super/supra/sobre: excessivo superlotado, sobrecarga ps-: que vem depois ps-guerra sub-: inferior subclasse, sub-rtico sub-: inferior sublinhar, sub-marinho

26

Veja agora alguns SUFIXOS: -al: similar/relative to formal, partial -en: made of wooden, golden -ic: relativo to economic, atomic -less: without useless, topless -ory: thing or place for - laboratory -y: full of sunny, sandy, oily -ent: forms nouns and adjs.: precedent / evident / subsequent -ness: forms abstract nouns: happiness -tion: forms nouns from verbs: devotion -en: sweeten / darken / widen -ize: legalize / initialize / finalize -al: semelhante/relativo a formal, parcial -ado: feito de de madeira, dourado -ico: relativo a econmico, atmico menos: sem - intil sem a parte de cima -rio: coisa ou lugar para laboratrio -ado/ido/io/oso/etc.: cheio de ensolarado, arenoso, oleoso -ente: forma nomes e adjs: precedente / evidente / subsequente forma subst. abstratos: felicidade -o: forma subst. de verbos: devoo -ar/ecer: adoar / escurecer / alargar -izar: legalizar / inicializar / finalizar

14. ARTIGOS
- Definidos Emprega-se o artigo definido the. usado em diversas situaes, tais como: * com substantivos contveis e no contavis (the book / o livro) * para se referir a algo j mencinado na frase anterior (I saw a man and a woman yesterday. The man was short and the woman was tall. / Eu vi um homem e uma mulher ontem. O homem era baixo e a mulher era alta.) * quando se espera que a pessoa com quem falamos saiba a que estamos nos referindo. (Its hot in here. Coul you open the window, please? / Est calor aqui. Podias abrir a janela, por favor?) * com expresses que tem a ver com instrumetos musicais no singular (She plays the piano. / Ela toca piano.) * nos superlativos. (the best book. / o melhor livro * com numerais ordinais. (the first exercise / o primeiro exerccio) * com perodos do dia e da noite. (in the morning/ de manh) * para se referir a algo que nico. (the moon / a lua) * com nomes de lugares. (the Azores / os Aores, the United States / os Estados Unidos) * com adjetivos. (the old / os idosos) * com apelidos plurais. (the Smiths / os Smiths/a famlia Smith) - Indefinidos No h nenhuma diferena entre a e an. So empregados somente com substantivos contveis singulares e servem para identificar, classificar, definir, quantificar, se referir a algo, falar sobre algo, com certos nmeros e em exclamaes. Ex.: Its a book. ( um livro.) A dog is a domestic animal. (O co um animal domstico.) I have a book. (Tenho um livro.) A Mr. Brown would like to see you. (Um tal Sr. Brown deseja v-lo.) He is a doctor. (Ele um mdico.) Id like a lemon, please. (Quero um limo, por favor.) I saw a car crash into a tree. (Eu vi um carro bater numa rvore.) three times a day (trs vezes por dia) Ive got a cold (Estou constipado.) What a day (Que dia!) a hundred (cem) OBS: Utilize a quando o substantivo iniciar com som de uma consoante e an quando o substantivo iniciar com som de uma vogal.

15. CONJUNES
Servem para juntar duas palavras, dois sintagmas ou duas oraes e apresentam-se em trs formas bsicas: simples (uma palavra), composta (duas ou mais palavras) e correlativa (palavras aos pares). Podem ser de dois tipos (coordenativas ou subordinativas). - Coordenativas: juntam duas palavras, dois sintagmas ou duas oraes independentes de idntica funo:

27

He is hungry and thirsty (adjetivo + and + adjetivo) The man in black and the woman in red are suspects. (sintagma + and + sintagma) I like cheese but I dont like garlic. (orao independente + but + orao independente) Simples - unem palavras, sintagmas ou oraes de mesma funo: and (e), but (mas), for (porque, visto que), nor (nem), or (ou), so (portanto, por conseguinte), yet (contudo, todavia, embora, porm, no entanto). Correlativas - juntam palavras, sintagmas ou oraes de idntica funo: both and (tanto como), either or (ou ou, quer quer), neither nor (nem nem), not only but also (no s mas tambm). - Subordinativas: juntam uma orao subordinada a uma orao principal. No juntam palavras ou sintagmas individuais. Although he studied, he didnt pass the exam. (although + orao subordinada + orao principal) I dont like him because he is a rude person (orao principal + because + orao subordinada) Simples - juntam uma orao subordinada a uma orao principal. No juntam palavras ou sintagmas individuais: after (depois que), as (como, quando, enquanto, porque, visto que), however (de qualquer maneira, seja como for), since (visto que, uma vez que, porque, desde que), until (at que), where (onde, aonde), whilst (enquanto, ao passo que) Composta terminadas com as, juntam uma orao subordinada a uma orao principal. No juntam palavras ou sintagmas individuais: according as (conforme, proporo que), as long as (desde que, contanto que, enquanto), in so far as (visto que, porquanto, atendendo a), so far as (tanto quanto, at). terminadas com than: rather than (em vez de), sooner than (em vez de) terminadas com that: buth that (a no ser que, seno que), given that (dado que, levando em conta o fato de que), in that (porque, visto que, devido a), so that (para que, a fim de que, de modo que). outras terminaes: as if (como se), as though (como se), in case (no caso de) Correlativas: juntam uma orao subordinada a uma orao principal. No juntam palavras ou sintagmas individuais: as as (to como), as so (do mesmo modo que, medida que), no sooner than (no mesmo instante em que, mal), such as (to como), whether or (quer quer).

16. QUESTION TAGS


As question tags so perguntas do tipo yes/no, empregues normalmente no fim de uma frase, na maioria das vezes utilizadas oralmente ou na escrita informal e servem para pedir confirmao, tendo a forma de verbo auxiliar + pronome pessoal. Ex.: Its fun, insnt it? ( divertido, no ?) You can teach me, cant you? (Voc pode me ensinar, no pode?) It isnt hot, is it? (No est calor, est?) John dint do it, did he? (O Joo no fez isso, ele fez?) comum o interlocutor empregar o verbo na afirmativa na question tag quando o verbo da orao principal estiver na afirmativa. Ele reage ao que ouve, mostrando seu interesse, surpresa, decepo, desaprovao ou uma outra reao. Entoao ascendente (interesse, surpresa): So youre going to the cinema, are you? (Ento voc vai ao cinema!, no ?) Entoao descendente (decepo, desaprovao) So youre getting married, are you? (Ento vais casar, no ?) Afirmativo/Negativo Quando o verbo na orao principal estiver na afirmativa, o verbo da question tag normalmente estar na negativa. Ex.: Its easy, isnt it? Quando o verbo na orao principal for be, emprega-se o mesmo na question tag. Ex.: Im late, arent you? (Estou atrasado, no estou?) Quando o verbo na orao principal for there+be, emprega-se o mesmo na question tag. Ex.: Theres a lot of hunger in the world, isnt there? (Ha muita fome no mundo, no h?) Quando o verbo na orao principal for precedido por um verbo auxiliar, emprega-se este ltimo na question tag. Ex.: He has finished, hasnt he? (Ele acabou, no acabou?) Quando o verbo na orao principal for precedido por mais que um verbo auxiliar, emprega-se sempre o primeiro dos auxiliares na question tag.

28

Ex.: Youve been running, havent you? (Voc tem corrido, no tem?) Quando o verbo na orao principal no precedido por um verbo auxiliar, emprega-se do, does ou did na question tag. Ex.: She likes it, doesnt she? (Ela gosta, no gosta?) Negativo/Afirmativo Quando o verbo na orao principal estiver na negativa, o verbo da question normalmente empregue na afirmativa. Ex.: It isnt difficult, is it? As regras para utilizao dos verbos so as mesmas usadas na forma afirmativa/negativa (acima). Negativo/Negativo Verbos na negativa tanto na orao principal quanto na question tag muito raro e s empregue para exprimir agressividade. Ex.: So he wont give me my money back, wont he? (Ento ele no vai me devolver o dinheiro, no ?) Imperativo Quando o verbo na orao principal estiver no imperativo, emprega-se o verbo auxiliar will na question tag. Esta forma usada para pedir a algum para fazer algo. Ex.: Turn on the light, will you? (Podes acender a luz?) Existem outras formas possveis para o imperativo, alm de will, so elas: wont, would, can, cant, shall. Ex.: Close the window, would you?(Podes fechar a janela?) Lets go, shall we? (Vamos?) Shut up, cant you? (Voc no pode se calar?) It e they na question tag It empregado quando o sujeito do verbo na orao principal for nothing ou everything. Ex.: Nothing is the same, is it? (Nada igual, no ?) They empregado quando o sujeito do verbo na orao principal for somebody, anybody, no one, nobody, etc. Ex.: Somebody broke the vase, didnt they? (Algum quebrou o vaso, no ?)

17. CONECTIVOS
Words of connection so conjunes, advrbios, preposies, locues, etc., que servem para estabelecer uma relao lgica entre frases e idias. O uso correto dessas palavras confere solidez ao argumento e consequentemente elegncia ao texto. Veja alguns exemplos: - First of all, / In the first place,/ To begin with, (Em primeiro lugar / Para comear) - Especially / Mainly (Principalmente ) - for some time / for a while / For the time begin (Por enquanto (no futuro) / At que mude de idia) - As a rule (Via de regra) - As time goes by ( medida em que o tempo passa) - By the way / Speaking of that (A propsito / Por falar nisso) - From the standpoint of (Do ponto de vista de) - On the one hand (Por um lado) - Likewise (Da mesma forma / Por seu turno) - However (Entretanto / No entanto) - In spite of / Despite (Apesar de) - At least (Pelo menos) - Unlike (Ao contrrio de) - Unless (A no ser que) - Since (Uma vez que / J que) - For this reason / With this in mind (Por esta razo / Neste sentido / Desta forma / Diante do exposto ) - Therefore / So (Portanto) - All things considered / Finally / In summary (Levando tudo isso em considerao / Em resumo)

18. VOCABULRIO
Falsos Cognatos

29

Actually (na verdade) nowadays (atualmente), eventually (afinal), apparently (ficar sabendo), ultimately (no final das contas), combine (combinar duas coisas) arrange (combinar com outra pessoa), compromise (meio termo) arrangement (compromisso), deception (enganao) disappointment (decepo), educated (boa formao) [im]polite ([mal-]educado), indicate (mostrar) recommend (indicar), nervous (nervosa, ansioso), annoyed/irritated (nervosa, chateado), particular (especfico) private (particular), prejudice (preconceito) loss (prejuzo), pretend (fingir) intend (pretender) think of doing (pensar em fazer), propaganda (informaes enganosas) advertising/advertisement (propaganda/uma propaganda), resume (recomear) summarize (resumir), reunion (encontro de ex-colegas, famlia etc.) meeting (reunio), sensible (sensato) sensitive (sensvel), a shopping centre (um shopping), go shopping (fazer compras), support (apoiar) cant bear/stand (no suportar), sympathetic (solidrio) nice/friendly (simptico) tax (imposto) fee/charge (taxa), use (usar coisas) wear (usar roupas, jias, maquiagem etc.) Palavras que causam erros com freqncia Erevy day (todo dia) all day (o dia todo), another 3 weeks (mais trs semanas), go/come back (voltar), cook (cozinheiro) cooker (fogo), hes dead (Ele est morto.) he died in 1990 (Ele morreu em 1990.), he works hard (Ele trabalha duro/muito.) hardly ever (quase nunca), a story (uma histria) history (histria), listen to (escutar, voluntariamente) hear (ouvir), I left the house (sa de casa), I left it a home (esqueci em casa), lend (emprestar) borrow (pegar emprestado), steal some money (roubar dinheiro) rob a house/bank etc. (roubar casa/banco etc.), lose your wallet (perder a sua carteira) miss the planet/lesson (perder o/a avio/aula), do a test (fazer uma prova) I met him in 1998 (Conheci ele em 1998.), I know his brother (Conheo o irmo dele.), I remember him (Eu me lembro dele.), you remind me of him (Voc me lembra dele.), they raise the price (Eles aumentam o preo.) the price rises (o preo subiu), he said (Ele falou.) he told me (Ele me falou.), he got/be came annoyed (Ele ficou nervoso.), there is/are (tem, existe), shes away/out of town (Ela est viajando.), a small town (uma cidadezinha) a big city (uma cidade grande), I usually work (Eu costumo trabalhar.) I used to work (Eu trabalhava.) Im used to working (Estou acostumado a trabalhar.), shes so clever (Ela to esperta.) shes such a clever girl (Ela uma menina to esperta.), shes 10 years old (Ela tem 10 anos.) a 10 year old gr (uma menina de 10 anos), beside (do lado) besides (alm disso), bored (chateado/entediado), annoyed/irritated (chateado/nervoso), impressive (impressionante), stressful (estressante).

EXERCCIOS
Read the extract and answer questions 01 and 02. Britain has one of the longest coastlines in Europe: 12,500 kilometres of varied and spectacular shoreline that has shaped the character of this island nation. Over the years, many people have landed at and embarked from British beaches; invaders and explorers, sailors and fishermen, merchants and missionaries. The most numerous visitors, however, have _______ holidaymakers. GLOSSARY shoreline = costa, litoral holidaymakers = turistas 01) Choose the best alternative to have the text completed. a) been b) gone c) visited d) arrived

02) Choose the alternative that presents an irregular plural form of the noun. a) beaches b) fishermen c) kilometres d) missionaries Read the joke and answer the question. Doctor, doctor, I keep thinking Im invisible. Who ________ tha? 03) Choose the best alternative to complete the blank in the dialog. a) said b) did say c) do dyou say d) did you say Read the joke and answer the question. Teacher: Johnny, how can you prove the world is round? Johnny: I never said it was, miss. 04) The modal verb, underlined in the dialog, expresses: a) ability b) advice c) possibility

d) permission

30

Read the extract and answer questions 09 and 10. Jade Barbosa lost her mother when she _____ just nine years old. At thirteen she had to leave her father and brother in Rio to train at the Curitiba training center. At just sixteen she is the new star of Brazilian gymnastics. In the Pan American Games she won one gold medal, one silver and one bronze.
(Taken from Maganews October 2007)

05) Complete the tex with the missing verb. a) was b) got c) had 06) According to the text, it is not true to say that: a) Jades parents died. c) she moved to Curitiba.

d) made b) she isnt an only child. d) she won three medals as gymnast.

Read the text and answer questions 07, 08, 09, 10 and 11. Who Sleeps? Reptiles, birds and mammals all sleep. Some fish and amphibians reduce their awareness but do not ever become uncoscious like the higher vertebrates do. Insects do not appear to sleep, although they may 5 become inactive in dayligth or darkness. By studying brainwaves, it is known that reptiles do not dream. Birds dream a little. Mammals all dream during sleep. Whales and dolphins are conscious breather and because they nedd to keep conscious 10 while they sleep in order to breathe, only one half of Their brain sleeps at a time. 1 GLOSSARY: awareness = conscincia 07) According to the text, a) all animals fall asleep. c) reptiles neither sleep nor dream. b) frogs are never asleep. d) insects are very active to become uncounscious.

08) In although they may become inactive , (line 4), the underlined word implies an idea of a) addition b) purpose c) contrast d) comparison 09) only one half ot their brain sleeps, (lines 10 and11), means that a) they keep conscious half a day. b) the largest part of their brain sleeps. c) just fifty percent of their brain in asleep. d) dolphins and whales dream half and hour. 10) Birds dream a little, (line 7), means that a) they dream a bit. c) only some birds can dream b) they dont dream at all. d) just a few birds dream while sleeping.

11) In like the higher vertebrates do, (lines 3 and 4), the underlined word was used a) as an adverb b) for emphasis c) as a main verb d) as an auxiliary verb. 12) Choose the best alternative to fill in the blanks. A: Do you have ________ book on Biology? B: No, I have ___________. But I know there are _________ at the library. a) some/any/some b) any/none/some c) any/some/none Read the text and answer questions 13, 14 and 15. These are some of the questions that parents ask themselves as their children grow up and move on: Will they sleep through the nigth? Will they learn to read? Will they get good grades? Will they avoid drugs?

d) some/none/any

31

Will they be responsible about sex? Will the get into a decent scholl? Will they go to college? 13) According to the text, a) children dont want to grow up and move on. b) children ask all these questions to their parents. c) parents are worried about their childrens future. d) parents dont want to know anything about their childrens life. 14) The sentences with will, in the text, were used to a) change habits b) ask for advice 15) move on, (line 2), is closets in meaning to a) marry b) go away c) give permission c) have fun d) question about future d) give a party

16) According to the use of the definite article, choose the best alternative. a) He is learning guitar and piano. b) The France is famous for its wine. c) The Biology is an important science. d) The Queen of England lives in London. Read the text and answer questions 17, 18 and 19. Im Brenda. Im a housewife, age 36. I can organise my week as I want. So long as there are clean clothes to wear and meals to eat, nobody really minds how or when I do the housework. The bad thing is that housework is so repetitive and unrewarding. Nobody notices if you do clean the bathroom. Its only if you dont clean it that they will say anything. GLOSSARY: Unrewarding = sem compensao 17) According to the text, Brenda a) decides how and when to do her housework. b) doesnt mind if the housework is repetitive. c) should clean the bathromm first. d) is very efficient. 18) We can infer from the text that people only care about the housework when they realize a) the bathromm is really clean. b) how repetitive the housework is. c) they cant find clean clothes to wear. d) how difficult is to organize the house. 19) Brenda is a housewife means that she works a) for another family b) as a housekeeper c) in a hotel d) at home

Read the text and answer the question. Linda Bates is a teacher at Allentown Adult School. She teaches English as a second language. Students say, Ms Bates is a very good teacher. She works very hard. Students like her classes. They say, Her classes are interesting. We learn a lot from her. 20) According to the text, we can conclude that Ms Bates teaches a) well b) children c) hard things d) two languages

Read the extract and choose the best alternative to fill in the blank. When you read you sometimes want to find specific information, such as a price, a phone number, or an address. You dont read every word. You only look for the information you want. This skill is called scanning. 21) scanning, underlined in the text, is _________ for reading. a) a rule b) an advice c) a technique d) some information 22) In A microscope has many uses. Medical scientists use microscopes to see tiny organisms., the underlined word means a) harmful b) invisible c) biological d) extremely small

32

As questes de 23 a 29 referem-se ao seguinte texto:

Smarter Clothes. Europe wants to own the market for fabrics that can monitor you and your
environment SALLY MCGRANE/PAVIA AT THE EUCENTRE, A RESEARCH SITE cofounded by the Italian Civil Protection Departament in Pavia, Italy, a young engineer dons a firefighters uniform that has been in testing for six months. The first prototype of the Proetex project, the ordinary looking navy blue jacket and pants contain high-tech fabrics that can keep track of a firefighters vital signs, warn him if the fire is too hot up ahead, provide GPS readings of his position and alert the command center if he has passed out. () Though the technology was pioneered in the U. S., the Europeans have taken the reins in a bid to revitalize their traditional-textile industry, which has been hammered by Asian competition. We want to develop state-of-the-art know-how that cant be found in Asia, says Andreas Lymberis, a scientific officer with the European Commission who has championed smart textiles. Our purpose is to create a new markeet. Bringing industry partners like Phillips and traditional clothing and textile companies together with university researchers from across the E.U. and Switzerland, Comission-funded teams have already produced prototypes with limited commercial availability, such as a tank top that wirelessly monitors cardiac patients and sports clothes that keep track of breathing. Other projects include fabrics that look and feel normal but are embedded with microcomputers, solar panels and energy-harvesting systems, as well as fabrics that measure blood oxygen levels and track biochemicals in sweat and bedsheets that monitor depression. The world market for smart textiles is still small about R 440 million in revenue in 2008 but that could double by 2010, according to Massachusetts-based venture Development Corp. The challenge is to fit applications to the market, says Lutz Walter, R&D manager at Euratex, a group representing the $ 326 billion European clothing-and-textile industry. In the medical field, theres high value added. But to be approved as devices takes 10 years, says Walter. In other areas, its price: How much are consumers going to be willing to pay for a smart jogging shirt or for a baby suit that detects sudden death syndrome? () The development of these technologies is currently taking place largely in the biomedical and safety fields, but Annalisa Bonfiglio, a professor of electrical and electronic engineering at the University of Cagliari who coordinates the Proetex projetc, thinks sports could be the sector where the most potential lies. Sportswear is an extremely powerful means for promoting the acceptance of these new technologies by common people, says Bonfiglio, noting that the technology Proetex develops for rescue workers could easily be used later for sports applications. At the Spaulding Rehabilitation Hospital in Boston, researchers are testing a glove made by Smartex, an Italian smart-materials company, that tracks motos functions in poststroke patients. Smartex founder and University of Pisa biomedical-engineering professor Danilo de Rossi says there is no way of knowing if Europe will maintain its edge. Right now we are leading in this field, he says, since Europe tends to be concerned with medicine, social welfare and the elderly, whereas the U.S. tends to focus on military technology. Thag could change. But in a business driven by technology rather than price, the Europeans woulds still have a fighting chance. 23) Assinale a opo que melhor indica o tema central do texto. a) Levantamento de necessidades do mercado mundial para o desenvolvimento de tecidos inteligentes. b) Descrio de peas de vesturio desenvolvidas por engenheiros europeus e americanos. c) Disputa do mercado mundial para deteno da tecnologia para desenvolvimento e produo de tecidos inteligentes. d) Concorrncia entre diversas indstria do setor txtil. e) Disputa entre universidades e indstrias europias para o desenvolvimento de pesquisa tencolgica na rea txtil. 24) De acordo com o texto, a indumentria desenvolvida no Projeto Proetex permite, dentre outras funes, que: I. os sinais vitais e a localizao do usurio sejam monitorados. II. o usurio seja alertado sobre aumento da temperatura externa. III. um possvel desmaio do usurio seja evitado. Est(o) correta(s):

33

a) apenas a I

b) apenas a II

c) apenas a III

d) apenas I e II

e) apenas II e III

25) De acordo com o texto: I. a tecnologia hoje utilizada para o desenvolvimento de tecidos inteligentes para uniformes de bombeiros poder ser facilmente adaptada para roupas de esportistas. II. h consumidores dispostos a pagar qualquer preo por uma pea de roupa infantil que sinalize a doena morte-sbita. III. em breve, os asiticos passaro a dominar o mercado de tecidos inteligentes, hoje nas mos dos europes. Est(o) correta(s): a) apenas a I b) apenas a II c) apenas a III d) apenas I e II e) todas 26) Assinale a opo em que o termo da coluna II NO pode substituir o termo da coluna I no texto. COLUNA I COLUNA II a) dons (pargrafo 1) wears b) the reins (pargrafo 2) control c) a bid (pargrafo 2) an attempt d) hammered (pargrafo 2) stopped e) championed (pargrafo 2) supported 27) Assinale a opo que indica o projeto, ou prottipo, de uso de tecido inteligente que NO mencionado no texto. a) Roupa de cama capaz de monitorar depresso. b) Coletes sem fio para monitorar pacientes cardacos. c) Roupas esportivas para monitorar respirao. d) Tecidos com painel solar embutido. e) Meias para monitorar movimentos de pacientes ps-derrame. 28) De acordo com o texto: I. estima-se que a renda do mercado mundial de tecidos inteligentes poder atingir 1.1 bilho de dlares em aproximadamente dois anos. II. Smartex uma empresa italiana que foi fundada por um professor universitrio. III. a Comisso Europia subsidiou uma equipe composta pela Philips, por empresas tradicionais das reas txtil e de vesturio e por pesquisadores universitrios americanos e suos. Est(o) correta(s): a) apenas a I b) apenas a II c) apenas a III d) apenas I e II e) apenas II e III

29) Considere as seguintes frases extradas do texto e as respectivas reescritas. I. a young engineer dons a firefighters uniform that has been in testing for xis months. (pargrafo 1) a firefighters uniform that has been in testing for xis months is donned by a young engineer. II. Commission-funded teams have already produced prototypes with limited commercial availability (pargrafo 3) prototypes with limited commercial availability have already produced by Commission-fuded teams. III. researchers are testing a glove made by Smartex, an Italian smart-materials company, (paragrafo 6) a glove made by Smartex, an Italian smart-materials company, has been tested by researchers. Est(o) correta(s): a) apenas a I b) apenas a II c) apenas a III d) apenas I e II e) apenas II e III

As questes de 30 a 31 referem-se ao texto abaixo: Persuading Leonardo Although both Ben Shneidermans Leonards Laptop: Human Needs and the New Computing Technologies and B. J. Foggs Persuasive Technology: Using Computers to Change What We Think and Do are written by academics, the books transcend academia to provide a different view of the Internts potential. Shneiderman prepares the groundwork for what he calls the new computing while Fogg describes how to make that computing persuasive.

34

The idea behind Leonardos Laptop is a consideration of what Leonardo da Vinci world demand from a laptop computer and what he would do with it. To Shneiderman, who is founding director of the Human-Computer Interaction Lab at the University of Maryland, the new computing puts users first. Shneiderman begins with a brief history of computing and computer applications, declaring that, These founders of the old computing overcame technological limitations to build impressive projects and then turned to producing tools for themselves, giving little thought to the needs of other users. Although not a founder, I admit to being of the old computing generation. I programmed in dead languages such as IBMs 1401 Autocoder and 360 Assembler before progressing to Cobol and RPG. I have now learned Visual Basic and C++, and I can report that there is nothing intrinsic to any of these languages that center a programmers focus on those who use their applications. The new computing is not about languages but, as Shneiderman suggests, about understanding human activities and human relationships. With Leonardo as both creator and user, his laptop will enable greater creativity and grander goals. This book goads you with ideas for applications in e-learning, e-business, e-healthcare, and egovernment. Each area is built around a framework for technology innovation that Shneiderman calls the four circles of relationships and the four stages of activites. () Althought the mental picture of Leonardo with a notebook computer excites the imagination, as a literary device, it does not wear well as the book progresses. Nonetheless, Shneiderman achieves the objective os Leonardos Laptop creating a foundation for the new computing. With a new computing application in hand, B. J. Foggs Persuasive Technology: Using Computers to Change What We Think and Do gives you advice on its implementation. To Fogg, who lauchend Stanfords Persuasive Technology Lab and who holds seven patents in the area of UI* design, a web site must first be credible to be persuasive. Fogg has coined the term captology to describe this branch of the study of computers. From the books Introduction. Captology focuses on the design, research, and analysis of interactive computing products created for the purpose of changing peoples attitudes or behaviors. It is the computers ability to provide interactivity that gives its applications an advantage over other forms of media. Persuave Technology describes three basic roles that computers play: the computer as a tool, as media, and as a social actor. Further, there are seven types of persuasive tools describes by Fogg. Such tools persuade by simplifying, tunneling (guiding), customizing, being there at the right time, removing tedium, rewarding after observation, and reinforcing proper behavior. As media, computers can modify behavior by simulating new endeavors. As a social actor, computers persuade through praise. Howeveer, no matter the role, to persuade, the application must be credible. Perhaps the most interesting parts of Foggs book are the two chapters that discuss the ways in which computer applications destroy their own credibility and what an application or web site must do to be considered, by its users, trustworthy. According to Fogg, a computing device or application is perceived to be credible only if it is first perceived as believable-trustworthiness based on expertise. In brief, an application is trustworthy if it is thought to be fair and unbiased. It is trustworthy if its author or origin is thought to be skilled and knowledgeable. The crux of the issue is that credibillity matters. Both books are thoroughly documented and both are excellent points of departure for a more detailed inquiry into the available material. If both books are taken to heart, using computers and their applications will become enjoyable and satisfying.
U.I. - *User Inteface

30) Indique o gnero, em ingls, ao qual o texto acima pertence. a) summary b) review c) essay d) abstract

e) report

31) Considere as seguintes afirmaes. I. As duas obras discutidas no texto tm como assunto principal o uso do computador e suas aplicaes atuais e potenciais. II. Shneiderman e Fogg, autores do texto, mostram a potencial aplicao da internet nos dias atuais. III. De acordo com Shneiderman, o computador eficaz deve ser, concomitantemente, uma ferramenta capaz de persuadir e um agente interativo. Est(o) correta(s): a) apenas a I b) apenas a II c) apenas a III d) apenas I e II e) apenas I e III

32) Com relao a Leonardos Laptop: Human Needs and the New Computing TechnologiesI, NO se pode dizer que a obra: a) tem como foco o usurio de computadores, seja ele um iniciante ou especialista no assunto.

35

b) destaca a importncia de programas como Autocoder e Assembler, assim como COBOL, RPG, Visual Basic e C++. c) discute o tipo de uso que Leonardo da Vinci faria, caso tivesse um computador porttil. d) mostra a importncia das relaes humanas no uso do computador. e) apresenta ao usurio possibilidades de diferentes usos do computador, dentre eles, para negcios eletrnicos. 33) Com relao a Persuasive Technology: Using Computer to Chang What We Think and Do, analise as afirmaes a seguir: I. O trabalho foi idealizado no Laboratrio de Tecnologia Persuasiva da Universidade de Stanford e consiste na stima criao intelectual do ator. II. Ao propor um novo conceito na rea computacional, o autor destaca mudanas de atitude ou de comportamento dos usurios. III. A obra argumenta que uma pgina da web deve ser confivel para seduzir o usurio. Est(o) correta(s): a) apenas a I b) apenas a II c) apenas a III d) apenas I e II e) apenas II e III

As questes de 34 a 37 referem-se entrevista abaixo: Ten Questions Over a Cell Phone Milton Hatoum is the award winning author of Dois Irmos (Two Brothers) and Cinzas do Norte (Ashes from the North). His new novel, rfos do Eldorado (Eldorado Orphans), will be released next April. Which was your best trip ever? The trip I took with my father to Lebanon, in July 1992. He had not seen his Lebanese family for over 30 years. Visiting Lebanon and meeting dozens of relatives was a very emotional experience.

1 2

What is your dream trip? To go to Kashmir and some parts of India. I also would like to visit several African countries. In what other country would you like to live? Well, I have already lived in three countries and eight different cities. I now just want to stay around here. But when I think of Provence or Tuscany, I feel like spending some time in France and Italy. What do you admire most about a person? His or her character. What elevates or demeans a human being is not religion, gender, color, ethnicity none of that. Its the caracter. What do you hate most in a person? I think an arrogant person looks ridiculous. I hate meanness, deceit, dishonest people. Would you be happy without friends? I would be unhappier without them. What animal would you like to be? The very same one I was destined to be. Our fate is to be human. What do you do when you have nothing to do? I get bored when I dont do anything. Right now, after finishing a novel, I feel a bit like Im hanging in mid-air, aimless. But theres always a book to read or re-read. Who is your favorite film director? I love Rossellini, Visconti and the directors of Italian neo-realism. What caracter would you like to be? Its hard to say I would be a terrible actor. But all the characters in my novels have a bit of me in them.

3 4 5 6 7 8 9 10

34) Leia as informaes abaixo sobre Milton Hatoum: I. Viveu em diferentes pases e cidades e pretende conhecer lugares na ndia e na frica. II. Destaca a viagem ao Lbano, com seu pai, h 30 anos, como uma das mais marcantes de sua vida. III. Elegeu Provena ou Toscana para fixar residncia. Est(o) correta(s): a) apenas a I b) apenas a II c) apenas a III d) apenas I e II e) nenhum

36

35) De acordo com a entrevista, Milton Hatoum I. um renomado escritor, que recebeu premiao recentemente pela novela Dois Irmos. II. costuma reler suas obras quando est com tempo livre. III. reconhece caractersticas pessoais nos personagens que cria. Est(o) correta(s): a) apenas a I b) apenas a II c) apenas a III d) apenas I e II e) todas

36) Considere as seguintes tradues das respostas de Milton Hatoum: I. What elevates or demeans a human being is not religion, gender, color, ethnicity none of that. O que enaltece ou descaracteriza um ser humano no a religio, o sexo, a cor, a tica nada disso. II. I hate meanness, deceit, dishonest people. Eu odeio mesquinharia, falsidade, pessoas desonestas. III. Right now, after finishing a nvel, I feel a bit like Im hanging in mid-air, aimless. Agora, depois de terminar uma novela, eu me sinto totamente no ar, sem rumo. Est(o) correta(s): a) apenas a I b) apenas a II c) apenas a III d) apenas I e II e) apenas II e III

37) Marque a opo incorreta: a) Can you describe the trip you took in 1992? pode substituir a pergunta n 1, sem comprometer a resposta de Milton Hatoum. b) What is the main aspect that attracts your attention in a person? pode substituir a pergunta n 4, sem comprometer a resposta de Milton Hatoum. c) What is an arrogant person like? pode substituir a pergunta n 5, sem comprometer a resposta de Milton Hatoum. d) None except a human being pode ser outra resposta de Milton Hatoum pergunta n 7. e) How do you feel when you have nothing to do? pode substituir a pergunta n 8, sem comprometer a resposta de Milton Hatoum. Texto The Reluctant Learner My friend Tom is one of those six-to-midnight, enthusiastic, determined, and well-mentioned studiers. At six oclock he approaches his desk, and carefully organizes everything in preparation for the study period to follow. Having everything in place, he next carefully adjusts each item again, giving him time to think up the fist excuse; he recalls that in the morning he did not have quite enough time to read all the items of interest in the newspaper. He also realizes distractions completely out of the way before setting down to the task at hand. 38) The sentence, he recalls that in the morning he did not have quite enough time to read all the items of interest in the newspaper a) returns home in order to read the newspaper later b) complains about his needs of having more time to read c) assumes how organized he is every time he reads the news d) remembers his lack of time in doing things 39) He also realizes that if he is going to study it is better to have such distractions completely out of the way before setting down to the task at hand. The underlined word can be replaced by a) learns b) understands c) hopes thinks Read the following opinions from two children, Chelsea and Eryn (both aged 8) and then answer questions 40 related to them. Is the future for us? Chelsea: The biggest problem with the environment is the ozone layer theres a hole, and its getting bigger. Its made by cars and airplanes things which give off fumes. Eryn: The ozone layers like a piece of paper covering a rock. Its supposed to pretect us. Im scared the hole will get bigger and move around the world and people will get cancer.

37

Chelsea: We could get tandems, and longer bikes, so children could ride on the back. Cars should be very, very expensive. Eryn: You also get bad pollution from burning down the rainforest. We should give money to poor people in Africa and places. Chelsea: We should spread out the people evenly. We could say, Put your hands up all those who want to live in Africa. And then we could spread out the food. Theres enough to go around. Eryn: We use up far more of the earth than people in Africa so its a good idea for the whole world to discuss environment.
(Move up Heinemann)

40) Both Chelsea and Eryn think that we should a) let poor people spread out the food c) burn down rain forest Read the text bellow and answer questions 41 to 43. Biotechnology

b) not to burn a lot of fuel d) clean rivers and farmlands

Biotechnology is one of the new professional courses of studies offered by universisites. Biotechnology uses chemical and biological knowledge and the knowledge of new technologies in the areas of healthcare, food, chemistry, and the environment. Biotechnology graduates are multidisciplinary professionals. They study biology, chemistry, physics, statistics, and information technology. In the area of microbiology, these professionals study fungi, bacteria, viruses, and protozoa and the diseases that they cause in plants, animals, and human beings. They research the methods to use such microorganisms in the production of foods and beverages, such as dairy products, beear, and wine. The biotechnologist specializing in immunology uses the microorganisms in the production of vaccines and kits for diagnosis. In the food and pharmaceutical industries, they control microbial growth, safety, and hygiene at the workplace. They work in research for the development of new pharmaceutical drugs. They also work in the environmental area, to evaluate and prevent water and soil contamination.
(Challenge Richmond)

41) The one who graduates in biotechnology a) will be able to prescribe and its quite likely that he/she can produce forms of organic stuff. b) shall develop the capacity of producing microorganisms. c) is able to diagnose, predict and sabotage diseases. d) can acquire the knowledge to several professions and also carry out researches. 42) The text shows that I. the one who studies Biotechnology is able to recognize different studies of health, cooking and chemistry among others. II. thestudy of this subject is a new area the universisites are offering. III. tecnology involves multiple studies in the biotechnology area. IV. the environment is actually a new version of the biotechnology study. a) I, II and III b) III and IV c) I and II d) I, II and IV 43) The only matter that is NOT mentioned in the text is a) different professions b) some subjects Read the text bellow to answer questions 44 and 45. The Car Washer Who Became An Executive Robert L. Johnson is the CEO (Chief Executive Officer) of BET. Black Entertainment Television, a cable TV channel in the United States, BET specializes in producing programs for the African-American community. Read what this successful executive says about his first job. I __________ in Freeport, Illinois. My first job was at the local carwash. I _____________ sixteen years old. I _____________ every day, all summer, for a dollar an hour. I worked with ten other guys. All of us c) drinks d) illnesess

38

_________ from different racial, religious and economic backgrounds. We had to clean cars in teams, and we quickly learned to work together. I learned the better way to become indispensable: you know how to do all aspects of your job. At the carwash, all all aspects included vacuuming the interior, scrubbing whitewalls and polishing chromes until it shined. Sixteen years later, when I started my own business, I again had to know how to do every job in the company advertising, marketing, producing and negotiation contracts. Working at the carwash taught me that there is a direct connection between working and a feeling of self-steem. Young people who refuses jobs that they consider inferior or low-paying are only hurting themselves. As long as you do your best, every job is a learning experience and a step to a better job.
(Adapted from Readers Digest, January, 1999)

44) Mark the option which ISNT applied correctly in the text. a) themselves b) of your c) the better

d) a better

45) Mark the option that completes the gaps respectively. a) woke up / were / came back / came b) grew up / was / worked / were c) was born / had/ traveled / got d) lived / got / tried / lived Answer the questions 46 and 47 after reading the paragraph bellow. Americans are well-known for being friendly. If were taking a walk in the park and we pass someone, we usually say a few words to people in stores, bars and banks. But remember: friendliness is not friendship; its politeness. In the United States, its just as hard to make real friends as it is anywhere else.
(Move up Heinemann)

46) In the statement, we usually say a few words to people in stores the underlined words may be understood as a) some words can be said by us b) many words could be spoken by people in stores c) lots of words are used to say people about us d) a small number of words are said by people in stores 47) The sentence, In the United States its just as hard to make real friends as it is anywhere else, means a) making friends for them is something they arent able to do. b) Americans show the rest of the world how easy relationship is in their country. c) only in the United States people cant find easily real friends. d) although difficult, real friends is something we have to fight for finding in America. Read the following paragraph and then answer questions 48 according to it. The Kremlin hoping a young strong man can preserve its brutal victory in Chechnya. 48) The underlined verb is a (an). a) regular one and means permission c) irregular form followed by an infinitive b) modal giving an idea of ability d) defective verb which expresses possibility

Read the following paragraph and then answer questions 49 to 52 according to it. Dealing with Sensitive Materials on the Internet With the emergency of user-friendly online systems, the Word Wide Web and its introduction into the classroom, more and more children are taking advantage of the power of the internet. However, it remains largely and adult forum, and so it carries with it adults subject matter. Does it raise the question of what ________ when adult topics and a childs nave explorations meet? The debate has raised not only questions of obscenity, harassment, free speech, and censorship, but also of government control of the Internet, and its very nature as a communications resource. Whatever the outcome of this war is, it will set a precedent for how society and government deal with the exchange of information in the future. Is the Internet a free forum for discussion or is it a broadcasting service and therefore subject to the same restrictions as television, print, or radio? Are to internet communications on the right privacy covered by the, or can e-mail messages be legally observe ? Are web pages free speech or are some subjects taboo on the internet because a

39

child may stumble upon them? And who is responsible for internet content in a communications medium where traditional publishing scenarios no longer apply and content can be posted anonymously? For people who wish to control or limit the use of the internet, the issue of children and pornography has been a valuable tool for gaining public support. The key is to find a solution to protect our children while at the same time, avoid setting up a climate of control that will limit our rights as adults.
(Adapted from Franework Level 3 Richmond)

49) The underlined question has been mixed up. Put the word in the right order according to the context. a) Are communications on the Internet covered by the right to privacy? b) Are the internet covered on privacy by the right to communications? c) Are the internet to communications covered on by the rigth privacy? d) Are the right by privacy to communications on the internet covered? 50) Comparing the pieces of content the internet provides, it may be sai that. a) there are more childish articles than grown up ones b) children have to surf the internet less than adult c) the adults content provides much more useful information d) there isnt any solution to control our kids in the internet without limiting ourselves. 51) Complete the gap with the rigth verbal tense. a) does happen b) happen c) do happen 52) Mark the correct question to the answer below extracted from the text. The debate a) Which subject has explored children? b) What has raised questions of obscenity, harassment, etc? c) Which topic protected children from the argument of the text? d) What matter is trying to control the use of internet by children? Read the following paragraph and then answer questions 53 and 54 according to it. Upside dow Whos to say Whats impossible Well they forgot This world keeps spinning And with each new day I can feel a change in everything And as the surface breaks reflections fade But in some ways they remain the same And as my mind begins to spread its wings Theres no stopping curiosity
(Jack Johnson)

d) happens

53) In the lyrics the autor affirms that a) people have forgotten their past b) everything is moving and hes not interested in knowing it c) although this world spins things do not change at all d) the surface of the earth reflects some lofty ideals 54) Mark the right definitions for the title of the lyrics above a) To cause something to change completely and in a bad way b) To be friendly with someone, especially because they can help you c) Having the part which is usually at the top turned to be at the bottom d) When you cause something to move in a circle round a fixed point. Leia a passagem a seguir e resolva s questes de 55 a 59. If you happened to (55) dow Londons Regent Street this (56) Christmas, you may have noticed, just above the festooned storefronts and package-laden shoppers, a series of clusters of glowing translucent globes. If youd taken a (57) look, you would have realized that the globes were pulsating with color, the light emitting diodes (LEDs) within varying their hue and intensity according

40

to the number of (58) , the wind speed, and the amount of sunlight. And if youd looked really close, you would have discovered the quad-core Xeon computers running customized software that took inputs from people-monitoring video cameras and environmental sensors to precisely (59) the display. Escolha, em cada questo, a alternativa que completa corretamente a lacuna correspondente, tornando o texto coeso e coerente. 55) a) stroll 56) a) next 57) a) larger 58) a) passersby 59) a) choreograph b) tiptoe b) past b) opener b) passes b) listen c) crawl c) future c) closer c) passengers c) dance d) trek d) following d) locally d) passings d) rehearse e) paddle e) ago e) nearby e) passwords e) sing

60) Martin Hellman, professor emeritus at Stanford, used engineering risk analysis methods to determine the failure rate for the United States nuclear deterrence strategy and came up with a shocking 1 percent chance per year that a nuclear war will break out. What can be understood about this passage? a) According to the United States, a nuclear war is not likely to erupt. b) Engineering risk analysis methods are inefficient because they hardly determine failure rates. c) The USA develops strategies to avoid nuclear issues. d) There have been found failures concerning the risk analysis methods used by Professor Martin Hellman. e) According to the USAs nuclear deterrence strategy, 1 percent of the USA population is afraid of a nuclear war eruption. 61) Chip makers replaced aluminum interconnects with better conducting copper ones about seven years ago, but now coppers days are numbered too. What is the present condition of copper interconnects? a) They were substituted by aluminum ones. b) They will soon be replaced. c) They are numbered according to their conductivity. d) Their conductivity is as powerful as aluminum ones. e) They receive serial numbers. 62) Don McMillan likes to say that the only time people laugh at engineers is when they mess up at work. But hes the exception. Hes a trained electrical engineer, and people laugh at him everyday unless he messes up. Thats because he tells jokes for a living. What do we know about Don McMillan? a) Hes a comedian. b) He messes up at work. c) People laugh at him because he messes up. d) He pretends he is a trained electrical engineer. e) He makes money as an electrical engineer. 63) Designers use database of North American and European body measurements to create their products but feel a lack of data to adapt the designs to Ansians different body measurements. That will soon change? a) Databases of Norty American and European body measurements do well to all cultures. b) Designers will soon chance North American and European body measurements databases. c) Asians have the same body measurements as North American and European. d) The body measurements databases used by designers are not perfect but please all their costumers. e) One size never fits all. 64) Remember when Barbie whined that math is hard. Maybe you got annoyed at hearing a popular female doll say that to little girls. Or maybe you aiso had a nagging suspicion thati, in fact, boys are better at math. Well, the latest research is in, and the answer is a resounding no: an analysis of performance on math tests finds that girls match boys. The findind appears in the Jully 25 issue of the journal Science. Which of the new information bellow could be coherently added to the passage? a) And no gender difference can be found among top performers either. b) So, once more its proven that male chromosomes are more efficient. c) Therefore, the gender struggle is over: women have shown better performances that men. d) But playing Barbie is a way of learning Math.

41

e) Playing with the doll is a good stimulus to female X chromosome. 65) A legion of 38 solar-battered cars gathers in State Square in Darwin, on the northern coast of Autralia. The tlat, rectangular bodies hug the ground like three-wheeled UFOs, their etherealness accentuated by their motos eerie, barely perceptible hum. Three adjectives that can be used to describe the cars mentioned in this paragraph are a) green, not curving, spherical b) UFO-like, secret, noisy c) delicate, mysterious, silent c) heavy, unusual, humming e) stable, unique, polluting 66) The midday sun had chased the last of the mornings chill from the air when David Downey turned into the Garmin International parking lot, in Olathe Kan., winding up a 20-kilometer run. Hed been out on the road for nearly 2 hours, a little longer than usual, but he wanted to enjoy the perfect fall weather while he could. But the description of the situation presented in this paragraph, what do you know about the weather? a) It was quite hot at 12 oclock. b) It was windy because it was autumn. c) The wind was blowing at 20 kilometers per hour. d) The bad weather had been delaying peoples activities. e) The morning had been cold. 67) As one of us the heaviest one approached the fist major hill on a test ride of Brammo Motorsports brand new Enertia electric motorcycle, we were doubtful that this light, elegantly designed bike could haul a 109-kologram (240-pound) rider up the incline. We shouldnt have worried: it effortlessly propelled him to the top of Portland, Ores West Hills. What is NOT true abouth the product mentioned in this paragraph? a) Very heavy people can ride and trust it. b) Brammo Motorsports latest model is just out. c) It seems fragile. d) IT weights 109 kolograms. e) It succeeds riding heavy people up hill. 68) Improving the diversity of biological habitats and ecosystems is a vital goal in itself, yet policies to encourage biodiversity, like most legislation, will have both supporters and naysayers. By the information in this sentence, what do we know about biodiversity? a) Everybody agrees with it. b) Life depends on its objectives. c) The police will assure habitats and ecosystems goals. d) Most of the policemen encourage this legislation. e) Some people take a negative view of it. 69) Fisheries may be an ancient economic activity, but nowadays they are at the forefront of globalization. For instance, when it comes to the trade itself: a blue hake caught off the coast of New Zealand by a Japanese vessel may be processed in China before being flown to a market in London or Paris. Saying thatfisheries are at the forefront of globalization means that a) Fisheries are an old and traditional economic activity. b) The world has become globalized due to fisheries. c) Blue hake fishing is an activity that unites New Zealand, Japan, China, England and France. d) Fisheries have enabled diverse economies to engage round a common activity. e) The demanding markets of London and Paris have made countries such as New Zealand, China and Japan unite the vessel manufacturing activity. Leia o texto e resolve as questes de 70 a 74. E-NOSES
Adapted from IEEE Spectrum, 03.08

Several hundred years ago, village doctors in rural China diagnosed diabetes by the characteristically sweet semll of a patients breath. Today hospitals use a battery of blood tests and laboratory analyses to make that same diagnosis, but doctors may soon be sniffing their patients breath again. This time the doctors will have electronic noses small and cheap enough to carry in their pockets.

42

This e-nose will be the culmination of decades of work at counties laboratories, where researchers have sought to create a tiny, cheap, automatic sniffer that would let wine bottles monitor the aging of their, contents, allow meat packages to flag spoilage, and enable mailboxes to check for bombs. Imagine barroom coasters that double as Breathalyzers, bumper stickers that monitor car emissions. Until now, its been just so much sci-fi. E-nose technology has quietly advanced during the past two decades. Commercial models equipped with sensor arrays came to market in the mid-1990s, and today theyre used to distinguish wines, analyze food flavors, and sort lumber. Benchtope systems are also used in the pharmaceutical, food, cosmetics, and packaging industries, while smaller, portable units are used to monitor air quality. But these noses cost in the range of US $5000 to $100,000. A coming convergence between enose technology and advances in printed electronics will finally bring the price down way down. Within a decade well see e-noses that cost tens of dollars and appear in smart packaging for high-end items like pharmaceuticals or as part of intelligent or interactive appliances picture a refrigerator that nows when milk has gone bad. Prices could easily drop to under a dollar by 2020. The secret? Conducting polymers. Developers of both electronic noses and printed electronics are exploiting these materials, which can be sensitive to the chemicals that make up odors and are also capable of producing electrical signals. E-nose developers are concentrating on honing the sensing properties of conducting polymers, while the printed-electronics people are investigating ways of using these materials to fabricate ultralow-cost electronics. Combining the fruits of these two separate efforts will finally bring e-noses into our supermarkets, homes, and daily life. O quadro abaixo apresenta um ttulo apropriado para cada pargrafo do texto, conforme seu contedo. Observe o quadro e responda s questes de 75 a 79. a) The innovative material b) Personified inanimate objects c) Past inspired sci fi d) Already in the market e) Allying technologies promise the product accessible in ten years Atribua o ttulo apropriado a cada pargrafo, de acordo com o quadro acima. 70) Ttulo para o primeiro pargrafo a) b) c) d) e) 71) Ttulo para o segundo pargrafo a) b) c) d) e) 72) Ttulo para o terceiro pargrafo a) b) c) d) e) 73) Ttulo para o quarto pargrafo a) b) c) d) e) 74) Ttulo para o quinto pargrafo a) b) c) d) e) 75) In which of the following you are likely to find the text E-NOSE? a) a tourist brochure b) a safety leaflet d) a comic book e) a billboard c) a scientific journal

76) Escolha a palavra ou expresso que apresenta um significado DIFERENTE do significado da palavra sublinhada em: This e-nose will be the culmination of decades of work at countless laboratories a) highest point b) end c) apogee d) climax e) result 77) De acordo com o contedo do texto E-nose, qual das seguintes frases provvel ser encontrada no texto? a) E-noses are a nonsense and deserve no more research. b) E-noses will hardly be more than fantasy in peoples mind. c) E-noses will soon be omnipresent in this centurys societies. d) E-noses will remain unaffordable despite serious research is developed. e) E-noses are mere toys in the hands of imaginative scientists. 78) Qual o significado do seguinte fragmento retirado do texto E-nose: Imagine barroom coasters that double as Breathalyzers? a) There will be twice as much barroom coasters as Breathalyzers. b) Two different people will use the same barroom coaster. c) Breathalyzers will be replaced by barroom coasters. d) Every drink ordered will allow the costumer the right to use a Breathalyzer. e) Barroom coasters will have one more function. 79) Qual dos ttulos seguintes pode ser usado apropriadamente para substituir o ttulo do texto E-nose? a) ELECTRONIC NOSES SNIFF SUCCESS b) SNEEZING NOSES

43

c) INVENTED DIAGNOSIS e) EXPENSIVE ODORS TURNED CHEAP

d) HUMAN NOSES

Based on the text below, answer questions 80, 81 and 82. CLIMATE CHANGE WILL DESTROY US Climate change over the next 20 years could result in a global catastrophe costing millions of lives in wars and natural disasters. A secret report, suppressed by US defense chiefs and obtained by The Observer, warns that major European cities will be sunk beneath rising seas as Britain is plunged into a Siberian climate by 2020. Nuclear conflict, mega-droughts, famine and widespread rioting will erupt across the world. The document predicts thag abrupt climate change could bring the planet to the edge of anarchy as countries develop a nuclear threat to defend and securedwindling food, water and energy supplies. The threat to global stability vastly eclipses that of terrorism, say the few experts privy to its contents. Disruption and conflict will be endemic features of life, concludes the Pentagon analysis. Once again, warfare would define human life.
(Adapted from http://www.guardian.co.uk/climatechange/styry/0,12374,1153530,00.html.)

80) According to the text above, what will be the result of climate change? a) Hunger, war, lack of rain, public disturbance. b) War, eclipses, cold weather, lack of rain. c) Hunger, cold weather, conflict, eclipses. d) Terrorism, nuclear, threat, eclipses, rising seas. e) Public disturbance, rising seas, global stability, nuclear threat. 81) What is NOT true according to the text? a) Humans will have violent times. b) England will be colder than it is today. c) Climate changes will happen unexpectedly sudden. d) Climate change will be more serious than terrorism. e) Europe will be under the sea. 82) Considering the text, what does the word dwindling mean in this extract? () countries develop a nuclear threat to defend and secure dwindling food, water and energy supplies. a) widerspreading b) decreasing c) stable d) existing e) proper 83) Read the text below. Why was this Army soldier in eastern Afghanistan praised by the Defense Secretary? GATES HAILS SOLDIER SNAPPED IN PINK BOXER SHORTS Defense Secretary Robert Gates on Thursday praised on Army soldier in eastern Afghanistan who drew media attention this month after rushing to defend his post from attack while wearing pink boxer shorts and flip-flops, Reuters reported. Gates said in prepared remarks that he wants to meet the soldier and shake his hand the next time he visits Afghanistan. Any soldier who goes into battle against the Taliban in pink boxers and flip-flops has a special kind of courage, Gates said in a speech to be delivered in New York. I can only wonder about the impact on the Taliban. Just imagine seeing that: a guy in pink boxers and flip-flops has you in his crosshairs. What an incredible innovation in psychological warfare, he said. Army Specialist Zachary Boyd, 19, of Fort Worth, Texas, rushed from his sleeping quarters on May 11 to join fellow platoon members at a base in Afghanistans Kunar Province after the unit came under fire from Taliban positions. A news photographer was on hand to record the image of Boyd standing at a makeshift rampart in helmet, body armor, red T-shirt and boxers emblazoned with the message: I love NY. When the image wound up on the front page of the New York Times, Boyd told his parents he might lose his job if President Obama saw him out of uniform.
(Adapted from http://www.foxnews.com/story/0,2933,521138,00.html)

This Army soldier in eastern Afghanistan was praised by the Defense Secretary a) because even geing tired he kept his position. b) because he helped his feloows invade Afghanistan. c) because he presented an incredible innovation in psychological warfare.

44

d) because President Obama saw him oit of uniform. e) because he went into battle in his underwear. Based on the text below, answer questions 84 and 85. CULLING PIGS IN FLU FIGHT, EGYPT ANGERS HERDERS AND DISMAYS U.N. Cairo Egypt has begun forcibly slaughtering the countrys pig herds as a precaution against swine flu, a move that the United Nations described as a real mistake and one that is prompting anger among the countrys pig farmers. The decision, announced Wednesday, is already adding new strains to the tense relations between Egypts marjority Muslims and its Coptic Christians. Most of Egypts pig farmers are Christians, and some accuse the government of using swine flu fears to punish them economically.
(Adapted from http://www.nytimes.com/2009/05/01/health/01egypt.html)

84) Which is the best alternative considering some of the statements are true (T) and other are falses (F)? I The action Egypt has taken against the swine flu increased the conflict between Muslims and Christians. II The action Egypt has taken against the swine flu caused anger among pig farmers. III The U.N. considered the swine flu a real mistake. IV The U.N. supported the decision taken by Egypt. V The population of Egypt is mostly Muslims. VI The government wants to punish the Edyptians. The best alternative is: a) I (T), II (T), III (F), IV (F), V (T), VI (F) b) I (T), II (F), III (T), IV (T), V (F), VI (F) c) I (F), II (F), III (T), IV (F), V (F), VI (T) d) I (T), II (T), III (T), IV (F), V (T), VI (F) e) I (F), II (T), III (F), IV (F), V (F), VI (T) 85) The word prompting is this extract from the first paragraph () and one that is prompting anger among the countrys pig farmers. Has the same meaning as: a) realizing b) finishing with c) encouraging d) responding to e) preparing for 86) Which alternative below is NOT CORRECT, based on this OBAMA SIGNALS MORE ACTIVE RESPONSE TO PIRACY The rescue of Captain Phillips drew widespread praise for the Navy and Mr. Obama, but some experts warned that it could escalate the campaign by Somali pirates, who have vowed to take revenge on Americans and are holding more than 200 hostages from other countries. Mr. Obama praised Captain Phillips for his courage and leadership and selfless concern for his crew, and he said he was very proud of the Navy and other American agencies involed in the operation.
(Adapted from http://www.nytimes.com/2009/04/14/world/africa/14pirates.html?_r=1)

a)The word it refers to the rescue of Captain Phillips. c) The word his refers to Captain Phillips. e) The word he refers to Captain Phillips.

b) The word who refers to the Somali pirates. d) The word his refers to Captain Phillips.

87) Read the text below. Why has the US government changed credit card regulations? NEW CREDIT CARD LIMITATIONS IN THE US Credit card companies in the US will soon be bound by new restrictions on their ability to charge fees, or raise interest rates on existing borrowings. The bill is designed to protect credit card users from unexpected fees or increases to their interest rates. The US government has been concerned to tighten its regulations of the banking system in the light of the credit crunch and banking crisis. This cements a victory for every American consumer who has ever suffered at the hands of the credit card industry, said Senator Christopher Dodd, chairman of the Senate banking comminttee.

45

Americans currently owe nearly $1 trillion on their credit cards. The US government has been concerned to tighten its regulation of the banking system in the light of the credit crunch and banking crisis.
(Adapted from http://news.bbc.co.uk/2/hi/business/8063108/stm)

The US government has changed credit card regulations a) to keep their ability to charge fees and raise interest rates. c) to raise the credit limitations. e) to avoid a credit crunch and banking crisis. 88) Read the text and check the statements below. REAL BLUE SKY RESEARCH

b) to help financial companies. d) to raise consumer protection

Just where in the world is the bluest sky? Expedia wanted to know for its Blue Sky Explorer project. They asked NPL to develop a blue sky standard and some cheap equipment to measure it. Their solution was to use cheap light-emitting diodes as the light standard,rather than the typical expensive noble gas lamps, and the belt-and-braces device was calibrated against an international colourimetry standard. The result? Rio de Janeiro came out on top, followed by the Bay of Islands in New Zealand and Uluru in Australia. Unsurprisingly the UK did not figure in strongly in the top 10, but Castell Dinas Bran in Wales came in at number nine.
(Adapted from http://www.bbc.co.uk/2/hi/science/nature/8059502.stm)

Which is the best alternative considering some of the statements are true (T) and others are false (F)? I The research is about the colour of the sky in different parts of the planet. II Expedia used a previous blue sky standard in the research. III The lamps that are currently used in the measurement were replaced by cheap ones. IV NPL adjusted the device to use an international colourimentry standard. V Rio de Janeiro and New Zealand got first rank in the Research while Australia came in second. VI It was a surprise that the UK did not figure in strongly in the top 10. The best alternative is: a) I (T), II (F), III (F), IV (F), V (T), VI (F) b) I (F), II (F), III (T), IV (T), V (F), VI (F) c) I (F), II (T), III (T), IV (F), V (F), VI (T) d) I (T), II (F), III (T), IV (T), V (F), VI (F) e) I (T), II (T), III (F), IV (F), V (F), VI (T) 89) Which is the correct option to complete the text below? ________(1) last month I spent four days in _______(2) Angra with _____(3) cousin from ______(4) Paran. Her father is ______(5) uncle of mine who moved to _____(6) south 2 year ago. a) , the, a, the, the, the b) The, , a, , an, the c) , , a, , an, the d) The, the, a, the, the, the e) , , the, , an, 90) Which is the correct option to complete the sentences below? The furniture for our living room _______________________ devilvered. a) has not been b) have not been d) does not have e) has not

c) were not

91) Analyse the sentences below. Which TWO sentences express the same idea? (1) Those poor children must have new shoes. (2) Those poor children have had new shoes. (3) New shoes must be provided for those poor children. (4) New shoes must have been given for those poor children. The correct answer is: a) 1 and 2 b) 1 and 4 c) 2 and 3 d) 2 and 4 e) 1 and 3

92)Mary saw her boyfriend with another girl and they had a quarrel. She shouthed: Dont come here anymore! Which alternative best reports what she said? a) She said her boyfriend do not come here anymore. b) She told to her boyfriend not to come here anymore.

46

c) She said to her boyfriend to not to go there anymore. d) She told her boyfriend not to come there anymore. e) She told her boyfriend not to go there anymore. 93) Which sequence best complete the text below? That we are ____(1) the midst ____(2) crisis is now well understood. Our nation is ____(3) war against a far reaching network ____(4) violence and hatred. Our economy is badly weakened, a consequence of great and irresponsibility ___(5) the part os some, but also our collective failure ____(6) make choices and prepare the nation ____(7) a new age.
(Adapted from Barack Obamas speech/January, 2009)

a) at, to, in, of, at, in, to d) at, of, in, for, at, to, for

b) in, to, on, with, at, in, to e) in, of, at, with, in, to, to

c) in, of, at, of, on, to, for

94) Which sequence best completes the text below? When we go out on weekends, I dont mind ______(1). I enjoy ________(2) by car because it is more comfortable and I cant resist __________(3) to take photos. However, I avoid ________(4) at night as I prefer not _________(5) the risk of _________(6) on the wheel. a) to drive, travel, to stop, driving, run, sleep b) driving, to travel, to stop, driving, run, sleep c) drive, traveling, stopping, to drive, to run, sleep d) driving, traveling, stopping, driving, to run, sleeping e) to drive, traveling, stopping, driving, to run, sleeping 95) Which alternative is grammatically correct? a) He would like that I could come. c) He would like that I came. e) He would like me come. b) He would like me to come. d) He would like me coming.

96) Choose the best reply to this startement. Mary: I have all the books the teacher told us about last class. You: __________________________ a) Neither have I b) Nor have I d) So do I e) So did I 97) Which sequence best completes the text below? WHY SO MANY BOOKS?

c) So had I

Conventional wisdom among college and university students (and many of their parents) in early 2007 is that everything needed for research is available free on the Web. Therefore, academic libraries are often viewed as costly dinosaurs unnecessary expenses in todays environment. This idea is uninformed at best and foolish at worst. If college and university libraries and librarians (1)__________, we (2) ________ to invent, better yet, re-invent them.
(Adapted from http://www.educause.edu)

a) dont exist, will have d) exist, wont have

b) existed, would have e) didnt exist, wouldnt have

c) didnt exist, would have

98) Choose the best sequence to complete the blanks. GENUINE BUSINESS LESSONS FROM DONALD TRUMP Trump has been so _________(1) in large part because he has managed to build a lifestyle brand around the ___________(2) life that he lives and most people aspire to. He works ___________(3) to cultivate the image, and he uses it to expand into new business lines and find new areas for profit.
(Adapted from http://www.forbes.com)

a) success, luxurious, hard c) successfully, luxuriously, hardly e) successful, luxurious, hard

b) successful, luxury, hardly d) successfully, luxurious, hard

99) Read the extract below and choose the only sequence that can complete it. CHILDREN

47

The other day I was chatting to a friend about children. We were discussing the fact that we had both been __________(1) by rather strict parents, and had both decided to try to be more tolerant when we _____________(2) and had children of our own. The problem that my friend had found with this strategy was in finding a way to control his kids now that they are teenagers. He said the fact that he had __________(3) them for more than 13 years, providing all the love and care they needed, is not enough to persuade them to respect his decisions. a) brought up, grew up, looked for b) grown up, brought up, kooked for c) brought down, had grown up, looked after d) brought up, grew up, looked after e) grown up, had grown up, looked like NATO, ships, helicopters hunt down 7 pirates NAIROBI, Kenya NATO warships and helicopters pursued Somali pirates for seven hours after they attacked a Norwegian tanker, NATO spokesmen said Sunday, and the high-speed chase only ended when warning shots were fired at the pirates skiff. Seven pirates attempted to attack the Norwegianflagged MV Front Ardenne late Saturday but fled after crew took evasive maneuvers and alerted warships in the area, said Portuguese Lt. Cmdr. Alexandre Santos Fernandes, aboard a warship in the Gulf of Aden, and Cmdr. Chris Davies, of NATOs maritime headquarters in England. How the attack was thwarted is unclear, it appears to have been the actions of the tanker, Davies said. Fernandes said no shots were fired at the tanker. Davies said the pirates sailed into the path of the Canadian warship Winnipeg, which was escorting a World Food Program delivery ship through the Gulf of Aden. The American ship USS Halyburton was also in the area and joined the chase. There was a lengthy pursuit, over seven hours, Davies said. The pirates hurled weapons into the dark seas as the Canadian and U.S. warships closed in. The ships are part of NATOs anti-piracy mission. The skiff abandoned the scene and tried to escape to Somali territory, Fernandes said. It was heading toward Bossaso but we managed to track them. Warning shots have been made after several attempts to stop the vessel. Both ships deployed helicopters, and naval officers hailed the pirates over loudspeakers and finally fired warning shots to stop them, Fernandes said, but not before the pirates had dumped most of their weapons overboard. NATO forces boarded the skiff, where they found a rocket-propelled grenade, and interrogated, disarmed and released the pirates. The pirates cannot be prosecuted under Canadian law because they did not attack Canadian citizens or interests and the crime was not committed on Canadian territory. When a ship is part of NATO, the detention of a person is a matter for the national authorities, Fernandes said. It stops being a NATO issue and starts being a national issue. The pirates release underscores the difficulties navies have in fighting rampant piracy off the coast of lawless Somalia. Most of the time, foreign navies simply disarm and release the pirates they catch due to legal complications and logistical difficulties in transporting pirates and witnesses to court. Pirates have attacked more than 80 boats this year alone, four times the number assaulted in 2003, according to the Kuala Lumpur-based International Maritime Bureau. They now hold at least 18 ships including a Belgian tanker seized Saturday with 10 crew aboard and over 310 crew hostage, according to an Associated Press count.
(Adapted from: www.ap.org, 04/19/09)

100) The Canadian Navy is involved in the incident described in the text because: a) The pirates accidentally crossed in front of the Canadian warship Winnipeg which was on its way through the Gulf of Aden. b) The pirates tried to attack an escorted ship filled with valuable supplies for the World Food Program. c) The Norwegian tanker chased the pirates until the Canadian and the North American warships arrived. d) NATO ships were already in the area looking for the pirates skiff. e) The USS Halyburton asked their Canadian counterparts for assistance after locating the skiff. 101) In the sentence There was a lengthy pursuit, over seven hours, there is a word formed by the suffix y. In which option below the word is formed by the same suffix? a) lately b) mostly c) fury d) ally e) healthy 102) According to the text, the crew members of the Somali shops seized CANNOT be held in custody because: a) Their guns are hurled into the seas and no proof of their status as pirates can be found on board of their ships.

48

b) International Law forbids NATO members to arrest crewmembers of other nationalities. c) Somalia is a member of the North Atlantic Treaty Organization and therefore its ships are only subjected to that countrys national legal persecution. d) NATO members cannot make arrests abroad under a number of circumstances, such as when their nationals are not directly affected. e) Piracy is not a crime NATO countries are allowed to fight or prevent in the national waters of non-NATO countries. 103) Which option contains words that replace with accuracy the following words that are underlined in the text respectively: SKIFF EVASIVE THWARTED- RAMPANT: a) vessel, elusive, avoided, increasing b) small boat, dangerous, facilitated, violent c) yacht, ambiguous, planned, violent d) small boat, elusive, prevented, uncontrolled e) vessel, dangerous, spoilt, increasing 104) In the fragment naval officers hailed the pirates over loudspeakers, the word highlighted could be accurately replaced with: a) alerted b) threatened c) called d) informed e) advised 105)According to the text, the inability to seize pirates and take them to court ends up: a) Encouraging piracy in international waters because Somali authorities are too slow to introduce measures to protect ships in its own territory. b) Leading to difficult and lengthy court cases where witnesses and pirates have to be transported to different locations. c) Being the result of a number of legal and technical difficulties in Somalia to carry out proper legal action against pirates due to particular international organizations such as NATO. d) Underscoring the efforts of foreign navies to disarm and release pirates. e) Increasing the attacks of pirates who benefit from the fact that legal persecution will not be sought due to its inherent difficulties. 106) The economic downturn has affected many households in the United States. U.S. homeowners have continued investing their money in the stock market though. The underlined connective expresses the idea of: a) conclusion b) time c) emphasis d) contrast e) addition 107) Choose the option in which the prepositions complete the verb phrases with accuracy, respectively: I The ship is bound _______ Africa. II We set sail ______ a tour of the Caribbean. III The captain was responsible _______ the incident. IV Attention has to be given ______ the weather conditions. V Crew members are expected to comply ______ safety regulations. a) for, to, for, for, with b) in, for, for, to, to c) to, for, for, to, with d) to, for, to, to, to e) for, to, to, for, with 108) Complete the sentences with the correct verb tenses: I Little __________________ what problems he creates for others. II No sooner ____________________ the receiver with a great sigh of relief, help arrived. III Not for one moment __________________________ his integrity. IV Under no circumstances _________________________ class. a) Does he realize / he had put down / would I doubt / should you miss b) Does he realize / had he put down / would I doubt / should you miss c) He realizes / he had put down / will I doubt / you should miss d) He will realize / he puts down / will I doubt / should you miss e) He realized / he would put down / would I dout / you should miss 109) People must be aware of the consequences of their actions. One can do whatever he pleases as long as he doesnt do harm to others. This may not be followed by many people, but it certainly should. If a person has many friends, he must know this already. The underlined modal verbs express: a) advice, permission, permission, advice, obligation b) adivce, ability, permission, ability, obligation c) obligation, permission, possibility, advice, deduction

49

d) obligation, ability, possibility, obligation, deduction e) deduction, ability, permission, obligation, obligation 110) The suffix ly forms adverbs of manner from adjectives as, for example, in: calm (adjective) calmly (adverb of manner). Which of the adjectives below DOES NOT follow this rule? a) quick b) fast c) fine d) quiet e) exquisite 111) She has tried to reach them four times on the phone without success. Hence she needs to write them as her last option. The underlined connective expresses the idea of: a) addition b) conclusion c) enumeration d) contrast e) concession 112) Choose the correct option to complete the sentences: I A stranger came into the hall _____________ he opened the front door. II - __________________ you begin to look at the problem there is almost nothing you can do about it. III - __________________ extensive inquiries were made at the time, no trace was found of any relative. IV You cannot be put on probation _______________ you are guilty. a) Like, although, despite, as soon as b) Since, although, despite, as soon as c) Since, even though, although, unless d) As, once, although, unless e) As, once, in spite of, even if 113) The conditions expressed in the sentences IF they had known the outcomes, they would have taken different measures and you should know people better, if you really want to make more friends are, respectively: a) unlikely, unlikely b) impossible, unlikely c) likely, unlikely d) impossible, likely e) likely, impossible 114) People believe that saving money is the key to happiness. Nevertheless, enjoying life also involves spending money on things that make you feel happy and accomplished. The underlined connective could be replaced with: a) Thus b) Moreover c) Still d) Therefore e) Furthermore 115) Mark the alternative in which DO and MAKE are all used correctly: a) Do a favor / make repairs / make amends / make a bargain b) Make a bid / make a demand / do ends meet / make an entry c) Make an excuse / make a face / make as much as you can / make business d) Do a proposal / make a scene / do silly things / make a review e) Do an errand / make fun of / make known / do friends As questes de 116 a 118 referem-se ao seguinte texto: Botelho a worthy Award Winner Thys year, Brazil and the world have been celebrating the 100 th anniversary of the first fight at a public event by Brazilian Alberto Santos Dumont. With his imminent retirement next April as president and chief executive officer of Embraer, Maurcio Botelho must be seen as another Brazilian aviation hero, one who turned a small money-losing company into a vibrant word-class aircraft manufacturer. Botelho spent the first 15 years of his working career at Embraer before leaving to pursue other opportunities. When he returned in 1995, Embraer had been privatized and was embarking on production and delivery of its first regional jets, several years behind competitor Bombardier. The Embraer ERJ-145 had its maiden fight just weeks before Botelho came on board. First delivery was in late 1996, and the company has never looked back. Nearly 1,000 aircraft based on the ERJ-145 platform had been deliverd to 87 operators around the world. A decate on, the EJR-145 program has reached the 10 million fight-hour milestone. Under Botelhos leadship, Embraer has had a penchant for seizing market opportunitites. It saw an unfilled market niche for aircraft seating 70-110, and its four-model 170/190 program has won 731 orders to date from airlines around the world. Embraer also used the ERJ platform to develop and Intelligence, Surveillance and Reconnaissance aircraft, and to produce the Legacy, its first foray into business jets. It is now raking in orders for three other business aircraft in development.

50

Botelhos legacy is a dynamic company, one of Brazils leading exporters, investing in its employees and technology to produce top-notch aircraft.
(Airline Business Daily @ALTA, 2 December, 2006 p.6)

116) De acordo com o texto, pode-se inferior que Maurcio Botelho: a) trabalha na EMBRAER h quinze anos e considerado um heri da aviao brasileira. b) deixou as atividades na EMBRAER em 1995, poca em que o modelo ERJ-145 foi projetado. c) voltou a trabalhar na EMBRAER em 1995, antes da privatizao da empresa. d) iniciou sua vida professional na EMBRAER. e) deixou a presidncia da EMBRAER em abril de 2.006 117) Considere as seguintes informaes: I O modelo ERJ-145, fabricado pela EMBRAER, teve Maurcio Botelho como um dos idealizadores. II A empresa Bombardier passou a produzir jatos regionais para competir com os jatos do mesmo padro, fabricados pela EMBRAER. III A gesto de Maurcio Botelho na EMPRAER foi bem-sucedida devido ao investimento em tecnologia e em recursos humanos. Est(o) correta(s): a) apenas a I b) apenas a II c) apenas a III d) apenas I e II e) apenas I e III 118) Considere as seguintes afirmaes: I maiden em The Embraer ERJ-145 had its maiden flight just weeks before (4 pargrafo) pode ser substitudo por single. I penchant em Embraer has had a penchant for seizing market opportunities. (5 pargrafo) pode ser substitudo por tendency. III foray em its first foray into business jets. (6 pargrafo) pode ser substitudo por attempt. Est(o) correta(s): a) apenas a I b) apenas a II c) apenas a III d) apenas II e III e) todas As queses de 119 a 122 referem-se ao texto a seguir: Taking Measure With Hardware and Software The researchers who founded National Instruments Corp switched from building their equipment to beefing up other peoples. You could start a company. That offhand comment by Jim Truchard got Jeff Kodosky and Bill Nowlin thinking. Within days, Truchard and his two employees at the Applied Research Laboratories (ARL) at the University of Texas as Austin (UT) decided to give it a go. That was in February 1976. By May, the trio had incorporated. Today, National Instruments Corp has annual Sales topping $425 million, employs more than 3100 people, sells some 1500 hardware and software products, and, for five years running, has been rated by Fortune magazine as one of the 100 best companies to work for. At ARL, Truchard headed an underwater acoustic measurements lab. I had about two dozen different projects, all the way from basic acoustics to pragmatic testing of military sonar beam formers, he says. Truchard went into science because of Sputinik. I was right on the cusp of that movement. We were all taking Russian and physics, he says. He earnd bachelors and masters degrees in physics and did his PhD on a nonlinear parametric acoust receiving array in electrical engineering, all at UT. Kodosky and Nowlin both worked part-time for Truchard while enrolled at U.T. Nowlin earned a masters in electrical engineering and Kodosky, who has a bachelors in physics from Rensselaer Polytechnic Institute, worked toward a PhD: he drifted between theroretical high-energy physics and computer science but did not complete the degree. () Physics intuition Both Kodosky and Truchard point to their physics training as playing a role in their success with National Instruments. Says Truchard, Acoustical measurements are fairly tricky, and it happens to be a smaller are where you didnt have off-the-shelf equipment. You had to build equipment. That background, and the measurements themselves, created a basis. Whats more, he adds, the physics background helps create good intuition. I think having solved differential equations and learned about gradients, you know how things are going to work out. Ive always felt it helped me develop intuition about business.

51

In leaving academic research, says Kodosky, we took a giant step back from what we were workin on. ARL was cutting edge. Now it was customers who were doing the interesting experiments. Still, he and Truchard say they have more impact on science by supplying tools than they would have had as researchers. We can have a nonlinear effect on the productivity of the science and engineering community. There is plenty of anectodal evidence that our virtual instrumentation can make people 5 to 10 times as productivity. Kodosky says. I personally would find it frustrating [to do research] because its slow, but dropping by a customers site every couple of years is fun. We live vicariously through them.
(Tony Feder)

119) Considere as seguintes afirmaes: I Truchard, Nowlin e Kodosky fundaram a empresa National Instruments Corp, em 1976. II A National Instruments Corp atua na area de informtica e, de acordo com a Revista Fortune, est entre as 100 melhores empresas mais produtivas nesta area. III Nowlin e Kodosky trabalhavam para Truchard, em tempo parcial, enquanto estudavam na Universidade do Texas. Est(o) correta(s): a) apenas a I b) apenas a II c) apenas a III d) apenas I e III e) apenas II e III 120) Considere as seguintes afirmaes: I Truchard estudou Fsica na Rssia e tem doutorado em Engenharia Eltrica. II Nowlin mestre em Engenharia Eltrica. III Kodosky no concluiu o doutorado. Est(o) correta(s): a) apenas a II b) apenas I e II c) apenas I e III d) apenas II e III

e) todas

121) Considere as seguintes afirmaes: I A formao acadmica dos proprietrios da National Instruments Corp foi importante para o sucesso da empresa. II Para Truchard, Nowlin e Kodosky, o ingresso no mundo corporativo representou um retrocesso em termos de pesquisa acadmica. III A graduao de Kodosky e Truchard em Fsica foi um fator relevante para o desenvolvimento da intuio para os negcios. Est(o) correta(s): a) apenas a I b) apenas I e II c) apenas I e III d) apenas II e III e) todas 122) A expresso Whats more (2 coluna, 1 pargrafo) NO pode ser substitudo por: a) Nevertheless b) Besides c) Also d) Moreover e) Furthermore Leia o texto a seguir e responda as questoes 123, 124 e 125. Babies can spot languages on facial clues alone
(Adapted from NewScientist.com, May 2007)

(1) Young babies can discriminate between different languages just by looking at an adults face, even if they do not hear a single spoken word. And babies who grow up bilingual can do this for longer than monolingual infants. The work suggest that visual information help to tell languages apart. (2) This supports the idea that infants come prepared to learn multiple languages and to discriminate them both auditorily and visually, says Whitney Weikum from the University of British Columbia in Vancouver, Canada, who discovered babies keen eye for speech. Looking at a face may help identify speakers of your native language. (3) Weikum and her colleagues, showed babies videos of adults speaking various sentences, but with the sound turned off. The infants soon got bored, but as soon as speakers switched from Engelish to French, they noticed the change and watched with renewed interest. (4) Laura-Ann Petitto, who researches language and child development at Dartmouth College in Hanover, US, previously studied visual language perception in deaf babies who were learning sign language. She is excited by Weikums results: Never did we dream that young hearing babies also use visual cues in this stunning way. Bilingual asset (5) A good eye for different languages appears to be especially important if you need to tell them apart regularly. At eight months old, bilingual babies could still see the switch happen, but their monolingual peers stopped noticing it after the age of six months.

52

(6) This shows us how a babys language development is closely related to their learning environment, says Weikum. Only if they are explosed to more than one language, do they remain able to discriminate the languages visually. (7) However, Weikum does not think that parents who are keen to help their babies learn to speak need to introduce a second language before the visual discrimination ability disappears, of start using visually exaggerated speech. Our study does not show visual speech cues help infants learn languages, only to tell them apart. Parents should just continue talking to their babies in fun, engaging conversations. (8) The researchers now want to discover more about how bilingual babies maintain and take advantaged of visual discrimination, and find out what the precise visual cues are in a speakers face that help a baby to identify different languages. 123) Uma das frases a seguir expressa a idia Principal (P) do texto, uma outra expressa uma idia muito Ampla (A) para ser considerada a idia principal e uma terceira expressa uma idia muito Restrita (R) para ser a idia principal do texto. Identifique-as: a) ( ) Visual language perception is the focus of different researchers. b) ( ) Young babies have a keen eye for identifying a second language switch. c) ( ) Parents should not worry about introducing a second language to their kids before the visual discrimination ability disappears. 124) Numere as frases abaixo de 1 a 8, conforme o nmero do pargrafo indicado no texto que expressa a idia de cada sentena. a) ( ) Babies in whose environment only one language is spoken can hold the special ability for a shorter priod of time. b) ( ) Exposing a baby to visual linguistic stimuli wont necessarily make them better language learners. c) ( ) Language visual information is used either by babies who can hear as by babies who cannot. d) ( ) Scientists still dont have the answers for a series of questions. e) ( ) Summary of the text. f) ( ) The conclusion of the research agrees with the idea that humans have an innate biological apparatus for learning different languages. g) ( ) The context where a child is raised has to do with the development of their linguistic abilities. h) ( ) The experiment procedure. 125) Complete as frases a seguir com and, because, but, or e so para que elas faam sentido de acordo com o texto Babies can spot languages on facial clues alone. Os conectivos no devem ser repetidos. a) Differently from monolingual babies, bilingual ones can discriminate between different languages after six months old __________ that doesnt mean this ability them better languages learners. b) Whitney Weikum discovered babies keen eye for speech __________ Laura-Ann Pelitto, who also studies visual language perception in babies, got very excited with the results. c) A good eye for different languages appears especially important to bilingual babies _________ they need to tell them apart regularly. d) The sound of the videos shown to the babies had to be turned off ________ the results couldn have concluded that visual information helps to tell languages apart. e) The researches want to discover more about how bilingual babies maintain and take advantage of visual discrimination ___________ the studies should be carried on. Leia o texto a seguir e responda as questes 126, 127, 128 e 129. Wi-F? Why Worry? (Adapted from BBC, April 2007) Scare stories about the dangers of wireless networks lack credibility, argues Bill Thompson (1) Students at Canadas Lakehead University have to be careful how they connect to the internet because Wi-Fi is banned on large parts of the campus. (2) University president Fred Gilbert, whose academic interests include wildlife management, environmental studies and natural resources science, is worried about the health impact of the 2.4 GHz radio waves used by wireless networks. (3) Last year decided to adopt the precautionary principle and refused to allow Wi-Fi in those areas that have what he calls hard wire connectivity until it is proved to be safe. (4) Mr. Gilbert believes that microwave radiation in the frequency range of Wi-Fi has been shown to increase permeability of the blood-brain barrier, cause behavioural changes, alter cognitive functions,

53

activate a stress response, interfere iwht brain waves, cell grouth, cell communication calcium ion balance, etc., and cause single and double strand DNA breaks. (5) Unfortunately the science says he is wrong, and his students are suffering as a resuft. Smog talk (6) While the heating effects of high exposures to electromagnetic radiation can be damaging, the power levels of wireless connections are much lower than the microwave ovens and mobile phones which share the frequency range, and treating them in the same way is the worst sort of scaremongering. (7) Yet Mr. Gilbert is not alone. (8) In 2003 parents sued a primary school in Chicago because it had dared to provide children with easy access to computing resources over a wireless network. (9) And there are a number of pressure groups, campaigning organizations and ill-informed individuals who believe that wireless networks pose a threat to health and want to see them closed down. (10) Now it seems they have been joined by the editor of the UK newspaper the Independent on Sunday, which this weekend filled its front page with a call for research into the electronic smog that is permeating the nations schools and damaging growing childrens brains. (11) An accompanying editorial with the even-handed headline high-tech horrors called for an official inquiry, while the article outlining the perceived dangers asked Is the Wi-Fi revolution a health time bomb? (12) The answer, of course, is no. (13) That will not stop the newspaper stoking up a wave of opposition to one of the most liberating technologies to have come out of the hi-tech revolution, limiting childrens access to networked computers at schools and even blocking plans to develop municipal wireless networks in our towns and cities. (14) If the journalists were really concerned about the dangers of radio frequency electromagnetic radiation on the sensitive brains of the young, they should be calling for the closure of TV and radio transmission towers rather than asking us to turn off our Wi-Fi laptops. (15) The modulated frequencies that carry Radio 4 and ITV into our homes are just as powerful as the wireless networks, and a lot more pervasive. (16) And my wireless network is only carrying data when Im online, while Radio 3 burbles all day long, possibly exciting electrons in my brain and causing headaches. (17) Then there is the danger from photons of visible light streaming down onto us as we work, since these carry more energy than microwaves and could surely do more damage. (18) Perhaps we should demand that our children work in the dark. 126) Retiro do texto Wi-Fi? Why Worry? as informaes pedidas nos itens seguintes. The frequency of wireless network radio waves. ________________________a______________________. Three health damages network radio frequency are supposed to cause: ____________b____________, ______________________c______________________ and ___________________d______________________. An example of a technological device whose power levels are higher and can be more dangerous to health than that of Wi-Fi: ______________e_________________. 127) Retire do pargrafo indicado uma palavra ou expresso que seja o sinnimo da expresso dada em cada item. a) pargrafo 6 to have something in commom: b) pargrafo 9 are risky: c) pargrafo 10 request: d) pargrafo 14 insteat of: e) pargrafo 17 harm: 128) Complete as frases a seguir usando as palavras e expresses do quadro conforme o contedo do texto Wi-Fi? Why Worry? Bill Thompson Fred Gilbert Students at Canadas Lakehead University Ill-informed individuals The editor of The Independent Journalists

54

a) _____________________ thinks it is a mistake to forbid people to Wi-Fi connect to the internet under the argument of health impact of the 2.4 GHz radio waves used by this kind of technology. b) _____________________ devotes attention to diverse issues concerning how to live in nowadays world without the pervasive effects of technology. c) ______________________ are not allowed to use Wi-Fi resources in areas where the wire connectivity is available. d) ____________________, e)_____________________, f) ____________________ and g)__________________ agree when it comes to believing that wireless network technology threatens peoples healthy. h) _____________________ were imposed a precautionary principle by i) ___________________, and are experiencing and unpleasant situation. j) ______________________ are not really worried about the effects Wi-Fi technology can cause in humans bodies, if so they would also claim agains radio frequencies and electromagnetic radiation. GABARITO 01 a, 02 b, 03 a, 04 a, 05 a, 06 a, 07 b, 08 c, 09 c, 10 a, 11 d, 12 b, 13 c, 14 d, 15 b, 16 d, 17 a, 18 c, 19 d, 20 a, 21 c, 22 d, 23 c, 24 d, 25 a, 26 d, 27 e, 28 d, 29 a, 30 b, 31 a, 32 b, 33 e, 34 a, 35 c, 36 b, 37 a, 38 d, 39 b, 40 b, 41 d, 42 c, 43 a, 44 c, 45 b, 46 d, 47 d, 48 b, 49 a, 50 d, 51 d, 52 b, 53 c, 54 c, 55 a, 56 b, 57 c, 58 a, 59 a, 60 c, 61 b, 62 a, 63 e, 64 a, 65 c, 66 e, 67 d, 68 e, 69 d, 70 c, 71 b, 72 d, 73 e, 74 a, 75 c, 76 b, 77 c, 78 e, 79 a, 80 a, 81 e, 82 b, 83 e, 84 a, 85 c, 86 e, 87 d, 88 d, 89 c, 90 a, 91 e, 92 e, 93 c, 94 d, 95 b, 96 d, 97 c, 98 e, 99 d, 100 a, 101 e, 102 d, 103 d, 104 c, 105 e, 106 d, 107 c, 108 b, 109 c, 110 b, 111 b, 112 b, 112 d, 113 d, 114 c, 115 a, 116 d, 117 c, 118 d, 119 d, 120 d, 121 e, 122 a, 123 A, P, R, 124 5, 7, 4, 8, 1, 2, 6, 3, 125 a) but, b) and, c) because, d) or, e) so 126 - a) 2.4 GHz, b) aumento da permeabilidade da barreira hematoenceflica, c) mudanas comportamentais, d) alteraees nas funes cognitivas, e) microondas ou celulares, 127 a) share, b) pose a threat, c) request, d) rather than, e) damage, 128) a) Bill Thompson, b) Mr. Gilbert, c) Students at Canadas Lakehead University, d)Mr. Gilbert, e) Ill informed individuals, f) The editor of the Independent, g) Journalists, h) Students at Canadas Lakehad University, i) Mr. Gilbert, j) Journalists VEJA ABAIXO ALGUMAS PROVAS DE CONCURSOS ANTERIORES, GABARITADAS E COM RESOLUO.

01. AFA ACADEMIA DA FORA AREA


TEXTO QUESTES 1 e 2 Lets spend the night together He used to love her, but its all over now. A couple for more than two decades, Mick Jagger, and Jerry Hall, said they would have their marriage annulled. Hall filed for divorce after a Brazilian model claimed Jagger had made her pregnant. But Jagger challenged the action, saying his marriage to Hall in Bali was nhot legally valid. Undoubtedly, the hangup was money. London papers claimed that the Rolling Stones lead singer will pay $11 million to $15 million. That shouldnt be a problem. The Stones, even after rolling for a third of a century, are still the worlds top musical attraction, taking in nearly $88 million so far from this years touring.
(Adapted from Time, July, 1999)

01) The Statement from the text, But Jagger challenged the action saying his marriage to Hall in Bali was not legally valid means that Jagger a) displayed how his determination in relation to his marriage in Bali was not valid and legal in count. b) expressed his disagreement about the authorities rules, affirming his speech was true and theme were some mistakes with the laws. c) wanted to challenge Hall by saying their marriege had happened as a result of her pregnancy. d) divorced his wife after confirming her judgment accepting the consideration of a court of law shes brought an action against him. RESOLUO: ALTERNATIVA B Jagger casou-se em Bali, mas afirma agora que o casamento no tem validade jurdica. 02) The world hangup in the text means. a) something you wait for a short time and solve tightly. b) a particular place where people get money after requiring it in court. c) an informal way to keep something updated instead of asking for money judicially. d) a subject you have to solve.

55

RESOLUO: ALTERNATIVA B O dicionrio refere-se a hang-up como um sentimento permanente e irracional de ansiedade com relao a uma caracterstica pessoal. Portanto, pode-se entender, no contexto da passagem acima, que a palavra expressa um assunto a ser resolvido (uma pendncia). TEXTO QUESTES 3 a 5 Where Astronauts Are Gods In a country which learned not to believe in God, it reigns the belief in science. Like saints, the cosmonauts (the way Russians call astronauts) occupy a place of eminence in the pantheon of the national Russian heroes. They have multiple talents, being the greatest one, the capacity of going to space. Streets, avenues and schools are named after astronauts. There are a lot of statues and museums all over the country. Theres also a date to celebrate them, April, 12. Up to now, ninety nine Russians have already gone into space since 1961, when Yuri Gagarin became the first man to go into orbit. Not by chance, hes adored among the conquerors of Cosmo. His premature death when he was 34 years old (seven years after his first and unique space flight in a tragic plane accident whose causes are still mysterious) contributed to create the myth.
(Adapted from O Globo, April, 02-2006.)

03) According to the text, its understood that a) Russians are people whose religion isnt so strong as it is in the rest of the world. b) Yuri Gagarin went into orbit and since then science is a great subject to be discussed. c) Russia believes that God may be more important than its capacity to send cosmonauts to space. d) Nowadays astronauts occupy a high position in the news. RESOLUO: ALTERNATIVA A O texto fala sobre o sucesso dos expoentes cientficos russos, e, devido falta de religio, estes expoentes acabam se tornando heris nacionais, corroborando para que a alternativa seja a letra A. ressaltada a importncia dos cosmonautas (astronautas) na cultura e na mente do povo russo com exemplos como data comemorativa, esttuas, nome de ruas entre outros. 04) Another way to say the statement, Streets, avenues and schools are named after astronauts, would be: a) Astronauts gave their names to streets, avenues and schools. b) The names of astronauts give them a place of heroes on streets, avenues and schools. c) Streets, avenues and schools are given to astronauts by the meaning of their names. d) Russians honor their astronauts by giving their names to streets, avenues and schools. RESOLUO: ALTERNATIVA D A traduo do trecho Streets, avenues and schools are named after astronauts Ruas, avenidas e escolas recebem os nomes dos astronautas. a) FALSA, pois no so os astronautas que do os nomes, mas sim as autoridades, em homenagem aos astronautas. b) FALSA, pois esta afirmativa significa Os nomes dos astronautas lhes do um lugar de heris nas ruas, avenidas e escolas, sendo esta frase incoerente com o trecho de referncia da questo. c) FALSA, pois esta afirmativa significa Ruas, avenidas e escolas so dadas a astronautas por meio de seus nomes, sendo esta frase incoerente com o trecho de referncia da questo. d) VERDADEIRA: a traduo desta alternativa Os russos homenageiam seus astronautas dando seus nomes a ruas, avenidas e escolas, frase esta que nada mais do que uma parfrase do trecho original. 05) Taking the sentence, There are a lot of statues and museums all over the country. Theres also a date to celebrate them., the word in boldface refers to a) the statues and museums in Russia b) streets, avenues and schools in Russia c) the astronauts from Russia d) scientists from Russia RESOLUO: ALTERNATIVA C O dia 12 de abril uma data comemorativa para celebrar os astronautas na Rssia, alm disso, naquele pas h esttuas e museus para homenage-los. TEXTO QUESTES 6 e 7 June 12th, 1997. Diana, Princess of Wales, addresses a seminar on landmines. Here are some excerpts of her speech: I. I welcome this conference on landimines () because the world is too little aware of the waste of life, limb and land which antipersonnel landmines are causing among some of the poorest people on earth. Indeed, until my journed to Angola early this year, () I was largely unaware of it too. () 06) According to the except above the undefined statement means that

56

a) many people not even know or care about limbs and lands that are being lost due to landmines among the poorest people. b) anti-personnel landmines are causing awareness among not only the poorest people but the landmines, too. c) the poorest people on earth are waiting their lives in landmines thats why waste their healthy, too. d) limbs and lands that are among the poorest population are not aware of the world. RESOLUO: ALTERNATIVA A A princesa Diana discursa de modo claro falando que o mundo tem pouca conscincia ( to little aware ) sobre as perdas causadas por minas terrestres. 07) According to Dianas speech we way observe that her first concem is related to the a) ones who are not killed outrigth b) terrible injuries peopole from Angola suffer. c) little information the world has about the deaths caused by landmines. d) world that remains largely aware of the jandicapped ones. RESOLUO: ALTERNATIVA C No comeo do texto lemos The world, with its many other preocupations, remains largely unmoved by a death roll of some 800 people every month (), cuja traduo : O mundo, com suas muitas outras preocupaes, permanece imvel diante de um corredor da morte de 800 pessoas todos os meses. Ao fim do texto lemos que esse death roll (corredor da morte) causado pelas minas terrestres, o que nos leva alternativa C. 08) Read the paragraph below to answer question 8. Os brasileiros so os grandes beneficiados com a auto-suficincia na produo nacional de petrleo e com os investimentos que esto sendo feitos pela Petrobrs.
(Revista PETROBRS, Abril de 2006.)

According to the text. a) our fuel is being produced in our country and we need to import it. b) a lot of investment should be done in order in put Brazil in a high position. c) Brazilians now have their own production of fuel thats why we may be proud of Petrobras. d) Petrobras has been the first company to produce fuel in great quantity. RESOLUO: ALTERNATIVA C A traduo da alternativa C Os brasileiros agora possuem sua prpria produo de petrleo e por isso podemos nos orgulhar da Petrobrs. As demais alternativas no expressam corretamente o texto porque: a) afirma que necessitamos importar petrleo (o que no dito no texto, pelo contrrio, o texto subentende auto suficincia do Brasil em petrleo); b) afirma que, para colocar o Brasil em uma posio elevada so necessrios elevados investimentos (o texto nada afirma sobre posio elevada, mas sim sobre a auto suficincia e investimentos que j esto sendo feitos e no investimentos futuros que seriam necessrios); d) diz que a Petrobrs a primeira a produzir petrleo em grande quantidade (alm de no ser verdade, no dito no texto). TEXTO QUESTES 9 a 11 The following extract is from a childrens scholl science book. Read it and answer questions 9 to 11. When houses are double glazed, only a relatively small amount of heat is lost through the windows. Double glazed windows have two panes of glass and are strapped between them, preventing escaping of the heat. Double glazing also ensures that condensation is reduced and noise is decreased. Heating bills can be reduced when double glazing is installed. People living near busy roads or airports also find that double glazing has to be fitted.
(Taken from Exploring Grammar in Context Cambridge University Press)

09) only a relatively small amount of heat is los through the windows. The boldface expression in the sentence can only be substituted for a) a great deal of b) plenty of c) a small number of d) a little RESOLUO: ALTERNATIVA D A small amount of usado com expresses incontveis, e portanto s pode ser substituda por a little. 10) Mark the option that means Heating bills can be reduced when double glazing is installed. a) When double glazing is installed heating bills can reduced it. b) Double glazing can reduce heating bills when it is installed. c) Double glazing is installed to reduce heating bills. d) When you install double glazing you reduce heating bills.

57

RESOLUO: ALTERNATIVA B A questo trata da passagem de voz passiva para ativa; por isso, para a passiva, temos: custos de calefao podem ser reduzidos com vitrificao dupla, que, na voz ativa, fica a vitrificao dupla pode reduzir o custo de calefao. 11) Mark the item that contains the right information about the text. a) When double glazed is installed in a house it gets warmer and with less noise. b) Houses become more secure if the windows are glazed because of the condensation. c) A safe house is recognized when we can check that its glazed indeed and not so noisy. d) Double glazed houses are less noisy and prevent us from the high temperature. RESOLUO: ALTERNATIVA A No texto lemos sobre as vantagens de colocar camadas duplas de vidro nas janelas de uma casa: only a relatively small amount of heat is lost through the windows. Double glazed windosw preventing escaping of the heat. () also ensures that condensation is reduced and noise is decreased. Em portugus: somente uma relativamente pequena quantidade de calor perdida atravs das janelas. Janelas com camada dupla de vidro, eivta-se a perda de calor e tambm reduz a condensao e o barulho. Em nenhum momento o fator segurana levado em considerao, eliminando as alternativas B e C. J a alternativa D pode ser eliminada uma vez que, segundo a mesma, a dupla camada de vidro nos protegeria das altas temperaturas e esta afirmao no sustentada pelo texto. Alm disso, o contexto de uma regio fria, sendo citado no texto Heating bills can be reduced , isto , despesas com aquecimento podem ser reduzidas, assim, a alternativa correta a A. TEXTO QUESTES 12 a 14 Read the article below carefully and answer questions 12 to 14. Fireworks Fireworks constitute a variety of devices used for religious or entertainment purposes. They contain incendiary compunds that produce sound, color, smoke, or movement, or combination of these. The ingredients consist of fuels, oxidizers, and modifying agents. The fuels used are charcoal, sulfur, antimony sulfide, and powdered metals. Chlorates and nitrates usually provide oxygen for the reaction, since fireworks do not use atmospheric oxygen. Various colors are obtained from metals salts. Sodium produces yellow; copper, blue green; calcium, red; strontium, scarlet; and barium, green. Picric acid or sulfur tends to intensify the colors, and ammonium salts increase the shades obtainable. Addition of iron and aluminum powder provides sparks and fiery displays. The craft of blending mixtures packaging them is an ancient one in China, where fireworks are used for religious purposes. In Europe fireworks for entertainment have been manufactured since the 13th century. In many places throughout the United States and Canada the sale of fireworks is restricted by law because of the danger of injury.
(The International Encyclopedia)

12) Que alternativa est correta em relao ao texto? a) O oxignio atmosfrico, na verdade, um dos ingredientes dos fogos de artifcio. b) Fogos de artifcio vm sendo produzidos na Europa h oito sculos. c) Religiosos usavam fogos de artifcio somente para diverses. d) Nos Estados Unidos e Canad as pessoas no podem comprar fogos de artifcio. RESOLUO: ALTERNATIVA B a) FALSA: no texto encontramos fireworks do not use atmospheric oxygen, isto , os fogos no utilizam oxignio atmosfrico. b) VERDADEIRA: trecho transcrito do texto: In Europe fireworks () have been manufactured since the 13th century, ou seja, fogos tm sido fabricados na Europa desde o sculo 13 (isto , h 8 sculos). c) FALSA: o que o texto diz que os fogos so utilizados para fins religiosos e para de diverso e no que os religiosos o utilizam para diverso. d) FALSA: no texto encontramos In many places throughout the United Stares and Canada, isto , em vrios lugares dos EUA e Canad e no em toda a extenso destes pases como afirma a alternativa. 13) The craft of blending mixtures packaging them is an ancient one in China, where fireworks are used for religious purposes. The word one in this sentece a) refers to the world craft. b) denotes the quantity of fireworks used in China. c) is used to refer to a particular thing within a group. d) is used to emphasize a situation.

58

RESOLUO: ALTERNATIVA A The craft of blending mixtures packaging them is an ancient one, ou seja, an ancient craft. A palavra one usada para que a palavra craft no tenha que ser repetida. 14) Change the sentence extracted from the text into passive voice. Chlorates and nitrates usually provide oxygen for the reaction. a) Oxygen is provided for the reaction usually by chlorates and nitrates. b) The ones that usually provide oxygen for the reaction are chlorates and nitrates. c) Oxygen for the reaction is usually provided by chlorates and nitrates. d) Chlorates and nitrates are usually provided by oxygen for the reaction. RESOLUO: ALTERNATIVA C Nesta questo temos a mudana do emprego de voz ativa para voz passiva. Assim, se dissermos que os cloratos e nitratos normalmente fornecem o oxignio para a reao sua passiva ser o oxignio para a reao ser normalmente fornecido pelos cloratos e nitratos. TEXTO QUESTES 15 a 17 Read the passage below and answer questions 15 to 17 Who Moved My Cheese? Who Moved My Cheese? Its a parable that reveals deep truths about changes. Two little mice and two little men live in a labyrinth searching for some cheese a metaphor used to express what we wish to have in life, from a good job to spiritual peace. One of them is successful and writes what he learnt from his experiences on the walls of the labyrinths. The scribbled words on the walls teach us how to deal with the changes to live with less stress and achieve much more success at work and personal life. 15) Its clear that Who moved my chesse is a) a self-confidence book that tries to make us smarter to modify plentiful situations in our lives. b) a piece of advice for the ones who cant solve their problems at all, and still insist on being stuck. c) a self-improvement reading that shows how we should deal with ourselves and make life better. d) a special speech for the ones that are involved with the great changes that occur when they get amazed. RESOLUO: ALTERNATIVA C O texto refere-se a um famoso livro da categoria Auto ajuda, que poderia ter uma traduo livre para o ingls como self-development. O ltimo pargrafo nos leva diretamente alternativa correta, quando diz teach us how to deal with the changes to live with less stress and achieve much more success at work and personal life, que em portugus significa nos ensina como lidar com as mudanas da vida com menos estresse e alcanar muito mais sucesso no trabalho e na vida pessoal. 16) Two little mice and two little men live in a labyrinth searching for some cheese. The world little can only be substituted for ___________ and refers to the ___________. a) small / emphatic opinion given by the author to the characters. b) few / size of the men and mice. c) brief / height of them. d) short / the lack of importance showed by the author. RESOLUO: ALTERNATIVA A A palavra little no trecho significa pequenos. b) FALSA: few melhor traduzido por poucos c) FALSA: bried significa breve d) FALSA: short significa curto 17) Who moved my cheese? Whatx expected from the one who reads it? a) Knowing life and the labyrinths to succeed more then usual. b) Searching spiritual peace and living as the characters, always complaining abouth the career and life and general. c) Scribbling new experiences and admitting the one success to use them deeply. d) Dealing with changes, living better and achieving success. RESOLUO: ALTERNATIVA D O texto afirma que a parbola pode nos ensinar a lidar com mudanas, viver com menos stress e ter mais sucesso: teach us how to deal with the changes to live with less stress and achieve much more success at work and personal life. TEXTO QUESTES 18 a 20 Read a letter and answer questions 18 to 20.

59

As a survivor of the Holocaust, I lost the I led more than 60 years, when the world didn t give a damn on, like today, acknowledgde that a storm of destruction was imminent (The lost City, Sept, 12). Sitting in my comfortable, dry home watching the horror of New Orleans made me cry the tears I didnt have when I was a child losing everything, feeling with just the clothing I wore that day. More than 60 years ago, I was on a different continent. The disaster that unfolded in front of my eyes today took place in my adopted country, among my adopted people. We didnt know is an unpardonable excuse. We didnt care is more like it. Without hesitation, America spends billions and wastes human life in a country that is not interested in democracy. Yet we quibble about the cost of Katrina, a cost that will affect everyone in our backyard for years to come.
(Lucie L. Liebman Staten Island, New York)

18) The letter was written by someone who a) subsisted a terrible hurricane even in her own country. b) resisted the Holocaust though she had been in an unfamiliar country. c) continued living in New Orleans otherwise she had passed by away. d) didnt die despite the tragedy. RESOLUO: ALTERNATIVA D No incio da carta, sua autora cita que foi uma sobrevivente do Holocausto, o qual teria sido uma tragdia que se concretizara porque o mundo no teria dado ateno devida ao risco daquele desastre. A alternativa B pode ter confundido muitos candidatos, porm, est incorreta porque afirma que a autora teria sobrevivido ao Holocausto embora tivesse em um pas desconhecido, sendo que o trecho em negrito no se confirma no texto. 19) The Present Tense of the underlined verb in the first line is a) led b) lead c) lid d) leaden RESOLUO: ALTERNATIVA B O verbo em questo tem as seguintes formas: Infinitive To lead Simple past Led Past participle Led E quer dizer levar, passar (a vida) 20) We realize that Lucie feels extremely sorry about ___________ because ____________. a) the whirlpool / it destroyed part of her life. b) the hurricane / she lost almost everything. c) America / it isnt democratic. d) Katrina / it destroyed her country. RESOLUO: ALTERNATIVA C O sentimento de tristeza de Lucie pelo fato de a Amrica ter passado pelos desastres que poderiam ter sido evitados. A melhor resposta a alternativa C, contudo necessrio ressaltar o fato de que a segunda coluna no adequadamente preenchida por nenhuma das alternativas. TEXTO QUESTES 21 a 23 Read the text and answer questions 21 to 23 according to it. Fakes a world of copycats Every year, criminals make millions of dollars selling fake perfumes, clothes, medicines and computer software. Counterfeit goods account for about 7% of total trade across the globe. And the criminals gains are other peoples losses. Take, for example, governments which are unable to collect revenue from indirect taxes and customs duty on legitimate sales. Over 30% of sales in mainland China are estimated to be counterfeit. In India, falke products account for 10% of the revenue for the entire health sector. Five out of six Yamaha bikes sold worldwide are not the real thing. Nike, the brand which tells you to just do it, loses $70 million annually to the menace of fake brands and pirate products. Identical fakes cost Gillette $20 million a year, and Proctor and Gamble loses a staggering $150 million on a twelve-month average in China alone. Sometimes consumes prefer to buy an illegal copy of a video, CD-ROM, cassette or software package because it costs less. In Russia, for example, copies of Microsofts Office 2000 program sell for just 1% of the list price. Frequently, however, buyers dont realize they are buying a fake instead of the genuine article.
(Taken from Skyline 5 Macmillan)

21) The sentence the criminals gains are other peoples losses means a) criminals gain more than everyone. b) people lose more than expected by the criminals c) criminals gain, whereas others lose. d) people expect to lose less than the criminals gain.

60

RESOLUO: ALTERNATIVA C Whereas d a idia de contraste, dando a idia de que enquanto uma pessoa ganha, a outra perde. 22) Read the statements below and classify them as (T) true or (F) false, respectively ( ) More than ninety percent of the worldwide trade is made of fake products. ( ) Two famous brands in China are responsible for a loss of more than a hundred million dollars per year. ( ) More than two thirds of Yamahas bikes sold across the globe are fake products. ( ) The brand Nike has a great profit caused by counterfeit goods. Mark the correct alternative. a) F/F/T/F/ b) T/F/T/T c) T/T/F/F d) F/T/T/F RESOLUO: ALTERNATIVA A (F) Apenas 7% do comrcio mundial de produtos piratas/ (F) O texto afirma apenas que a empresa Procter and Galbme tem prejuzos com a pirataria (superiores a 100 milhes de dlares por ano), portanto, embora certamente haja outras empresas com prejuzos na China devido pirataria, somente uma citada; (T) Cinco em cada seis motos Yamaha vendidas so ilegais; (F) O texto informa que a Nike tem prejuzo de 70 milhes de dlares devido ameaa de imitaes e produtos piratas. 23) According to the last paragraph, consumers I have always intended to byu counterfeit goods. II are sometimes deceived by the fakes. III occasionally prefer pirate products because they are cheaper. IV see the illegal copies as a solution to the growing economical problems. The only correct sentences are a) I and III b) III and IV c) II and III d) II and IV RESOLUO: ALTERNATIVA C - As proposies corretas so somente as nmero II e III e os trechos se encontram a seguir: Para a proposio II temos as duas ltimas linhas do texto: Frequently, however, buyers dont realize they are buying a fake instead of the genuine article. (Frequentemente, no entanto, os compradores no notam que esto comprando um artigo falsificado ao invs de genuno). Para a proposio III temos as duas primeiras linhas do ltimo pargrafo: Sometimes consumers profer to buy an illegal copy () because it costs less. (s vezes os consumidores preferem comprar uma cpia ilegal porque ela custa menos). A afirmativa Iest incorreta devido ao termo sempre que no condiz com o texto, segundo o qual s vezes os consumidores preferem comprar produtos no originais. J a afirmativa IV est incorreta porque nada citado no ltimo pargrafo sobre a crena dos consumidores a respeito de solues para problemas de crescimento econmico. TEXTO QUESTES 24 e 25 Read the fragment and answer questions 24 and 25. Image, image, image Heres the coolest wildest hippest funkiest object! Image-conscious person cannot do without it! This style is ideal for fashion and consumer objects (cell phones, backpacks, watches, and so on). Its the most common form of advertising for many products. But dont forget all those other types of message. Be smart: dont let the advertisers fool you!
(Taken from Consumers Portfolio)

24) Theres one option which DOESNT match the context, mark it. a) According to ads, the image-conscious person doesnt have to live without their products. b) You may be persuaded by the advertisers. c) You need to be perceptive to avoid being influenced by the ads. d) You should pay attention to the message behind the advertisements. RESOLUO: ALTERNATIVA A O modal usado no texto para exemplificar as propagandas o modal can (na forma cannot). No possvel substitu-lo, no contexto, por have to, que expressa obrigao em vez de possibilidade, capacidade. O uso de can no contexto sugere muito mais a idia que o autor do texto deseja passar sobre as propagandas do que o modal have to. Alm disso, o texto enfatiza os riscos de ns, consumidores, no sermos cautelosos em relao s propagandas, o que adequadamente expresso nas alternativas, exceto a A.

61

25) According to image, image, image to sell fashion and consumer objects people announce them as the most Mark the option DOESNT suit the text. a) stylish b) different c) natural d) modern RESOLUO: ALTERNATIVA C O fragmento fala de como a publicidade procura demonstrar que os seus objetos so diferenciados, modernos, etc, e no naturais (comuns). TEXTO QUESTES 26 a 28 Read the paragraph and answer questions 26 to 28 according to it. The concept of generation gap is widely accepted in Canada and the United States. It was a concept that grew in prominence in the 1960s and 1970s, when a common belief among young people was Never trust anyone over thirty and many older people feit they could no longer understand young people. These days, manu people think that the generation gap is lessening because the baby-boom generation (those born between 1946 and 1961) has moved past the age of fhirty.
(Taken from Passages 1 TM, CUP)

26) The sentence in italicx, when a common thirty has the function of: a) establishing contrast b) emphasizing a concept c) showing a condition d) explaning something RESOLUO: ALTERNATIVA D A palavra when (quando), neste contexto, serve para colocar um fato em considerao, no sentido de explic-lo; no caso do texto dado, a inteno explicar o que significa o generation gap, conflito de geraes que baseava-se na frase Nunca confie em algum com mais de trinta anos. 27) Whats the main idea of the text? a) Never believe older people. b) The excessive amount of births after was caused the generation gap. c) The United Sttes and Canada have a lot of problems concerning people coming from different generations. d) What is known as generation gap was spread by the young people in the sixties and seventies. RESOLUO: ALTERNATIVA D O texto refere-se a um conflito de geraes ocorridos nos anos 60 e 70, o qual est perdendo fora nos dias atuais, entre outros fatores, pelo envelhecimento (e, portanto, mudana de lado no conflito) dos jovens daquele perodo. Notadamente, a segunda linha do texto diz it (generation gap) was a concept that grew in prominence in the 60s and 70s. Alm disso, nas 3 ltimas linhas temos These days, () past the age of thirty. 28) The text affims that the generation gap is lessening. It means it has a) increased b) stabilized c) decreased d) not changed RESOLUO: ALTERNATIVA C Da leitura do texto, infere-se que the generation gap is lessening, o que signifca dizer que o conflito em questo est diminuindo. (Lessen sinmino de decrease). 29) Read the paragraph of an article from Newsweek, Nov/14, 2005, whose title is Rethinking Arafat and answer question 29 according to it. It might also take years for Palestinians to assess Arafats legacy. Given the lingering suspicion that he was poisoned (the cause of death is listed as undetermined), many Palestinians arent in the mood to start sorting through his record. As long as this matter is not resolved, you wont hear people questioning his leadership, says Hafes Barghouti, editor of the Palestinian daily Al-Jadidah, just bemoaning the consequences. According to the writer, a) Arafats legacy will certainly take time to be assessed. b) Arafat must have been poisoned. c) many Palestinians want to clarify the suspicion concerning Arafats death as soon as possible. d) Arafats leadership wont be questioned, unless the matter of his death is resolved. RESOLUO: ALTERNATIVA D O texto coloca que, enquanto o assunto no for resolvido, ningum ouvir as pessoas questionando a liderana de Arafat: As long as this matter is not resolved, you wont hear people questioning his leadership. 30) Read the headline of the article referred on the previous question and answer question 30.

62

If the Palestinians leader was the real problem, why havent things improved in the year since his death? The author, a) assures things will be better after the leaders death. b) wonders if Arafat was really a problem to his country. c) thinks improvements need time. d) says things have changed after the leader died. RESOLUO: ALTERNATIVA B O autor indaga se Arafat era realmente o problema da Palestina, j que, de acordo com ele, as coisas no teriam mudado depois da morte da lder palestino.

02. ITA INSTITUTO TECNOLGICO DA AERONUTICA


As questes 1 a 7 referem-se aos seguintes textos: Texto I A text familiar to many, George Orwells classic satire has cropped up on school reading lists eversince the year of its creation. Few readers can fail to be touched by the tragedy within, by its wonderful synthesis of unthreatening symbolism and incisive criticism. This familiarity is conveniente since, as an adaptation, Animal Farm spends too little time on the details of time and place. Instead directors Joy Batchelor and John Halas thrust us directly into the depression that in Manor Farm, briefly explaining the situation with pictures and narration by Gordon Heath. () Sadly, Batchelor and Halas make it vital to have read Orwells biting satire on Soviet history before viewing Animal Farm, just to know whats been left out. As it is, the firm grasps the supercial aspects of Orwells allegoric fable without his deeper message. In missing so badly, were left within an impressive attempt that never matches up to its birthright. Texto II Power, corrupts, but absolute power corrupts abolutely and this is vividly and eloquently proved in Orwells short novel. Animal Farm is a simple fable of great symbolic value, and as Orwell himself explained: It is the history of a revolution that went wrong. The novel can be seen as the historical analysis of the causes of the failure of communism, or as a mere fairy-tale; in any case it tells a good story that aims to prove that human nature and diversity prevent people from being equal and happy, or at least equally happy. () In Animal Farm, Orwell describes how power turned the pigs from simple comrades to ruthless dictators who managed to walk on two legs, and carry whips. The history may be seen as an analysis of the Soviet regime, or as a warning against political pwer games of an absolute nature and totalitarianism in general. For this reason, the history ends with a hair-raising warning to all humankind: The creatures outsides looked from pig to man, and from man to pig and from pig to man again: but already it was impossible to say which was which. Texto III When the farms prize winning pig, Old Major, calls a meeting of all the animals of Manor Farm, he tells them that be has had a dream un which mankind is gone, and animals are free to live in peace and harmony. () When Old Major dies, () Snowball and Napoleon assume command, and turn his dream into a fullgledged philosophy. One nigth, the starved animals suddenly revolt and drive the farmer Mr. Jones, his wife, and his pet raven off the farm and take control. The farm is renamed Animal Farm as the animals works towards a future read, the seventh and most important of which is that all animals are equal (). Many year pass, and the pigs learn to walk upright, carry whips, and wear clothes. The Seven Commandments are reduced to a single phrase.: All animals are equal. But some animals are more equal then others. Napoleon holds a dinner party for the pigs, and the humans of the area (in the adjacent Foxwood Farm run by Mr. Pilkington), who congratulate Napoleon on having the most hard working animals in the country on the least feed. Napoleon announces his alliance with the humans against the labouring classes of both worlds. The animals discover this when they overhear Napoleons conversations and finally realize that a change has come over the ruling pigs. During a poker match, an argument breaks out between Napoleon and Mr. Pilkington when they both play an Ace of Spades, and the animals realize how they cannot tell me difference between the pigs and the humans. 01) Assinale a opo que indica, respectivamente, o gneros dos Textos I, II e III I II III a) Sinopse de livro/filme crtica literria crtica cinematogrfica. b) Crtica literria sinopse de livro/filme crtica cinematogrfica.

63

c) Crtica literria crtica cinematogrfica sinopse de livro/filme. d) Crtica cinematogrfica crtica literria sinopse de livro/filme. e) Crtica cinematogrfica sinopse de livro/filme crtica literria. RESOLUO: ALTERNATIVA D O primeiro texto fala sobre um filme que adaptao de um livro. as an adaptation, Animal farm spends too much time on details of time and place. Instead, directors () O Segundo uma crtica literria. The novel can be seen as the historical analysis of the causes of the failure of communism, or as a mere fairy-tale. O terceiro texto conta a histria do livro e, consequentemente, tambm do filme (que uma adaptao do livro), portanto uma sinopse. 02) Da leitura dos textos I, II e III, possvel depreender que: I. o trabalho de Orwell pode ser entendido como um conto de fadas ou como uma crtica a regimes polticos totalitrios. II. o filme Animal Farm retrata todos os aspectos apresentados no livro de mesmo nome e, por isso, recebeu elogios da crtica. III. o enredo de Animal Farm aborda, entre outros temas, a influncia do poder no comportamento humano. Ento est(o) correta(s): a) apenas a I b) apenas a II c) apenas a III d) apenas I e II e) apenas I e III RESOLUO: ALTERNATIVA E I Verdadeira. A primeira afirmao est correta e pode ser confirmada com o trecho The novel can be seen as the historical analysis of the causes of the failure of communism, of as a mere fairy-tale. II Falsa. A segunda afirmao est errada, o que pode ser confirmado atravs do ltimo pargrafo do primeiro texto, em que o autor afirma que uma leitura do livro se faz necessria para que se possa captar aquilo que o filme no disse. Alm disso, segundo a crtica apresentada no texto I, o filme no passa a mensagem mais profunda do livro. III Verdadeira. O livro trata de uma fazenda dominada por animais, aps a expulso dos humanos. No incio da dominao da fazenda pelos animais, estes trabalhavem em p de igualdade, porm mais adiante os porcos passaram a comandar o lugar, reescrevendo as regras, de modo a reforar e consolidar o prprio poder. A alegoria faz aluso luta pelo poder entre seres humanos, o que reforado quando os porcos firmam um acordo com seres humanos para a explorao dos demais animais. O desfecho se d quando os outros animais concluem que porcos e humanos so iguais, portanto, podemos dizer que a afirmao de que o poder influncia no comportamento humano verdadeira, pois os porcos nada mais so do que uma stira dos humanos detentores do poder. 03) Assinale a opo que indica possibilidade de substituio de termo ou expresso em negrito no Texto I sem que o sentido do texto seja comprometido. a) has cropped up em has cropped up on scholl reading lists pode ser substitudo por has been imposed. b) fail em Few readers can fail to be touched pode ser substitudo por help. c) since em This familiarity is convenient since, as pode ser substitudo por because. d) Instead na ltima sentena do primeiro pargrafo pode ser substitudo por Furthermore. e) left out em just to know whats been left out pode ser substitudo por included. RESOLUO: ALTERNATIVA C a) Incorreta. O verbo crop up quer dizer acontecer ou aparecer repentinamente, enquanto to impose significa impor; c) Correta. Since pode ser um advrbio de tempo ou uma preposio, mas tambm pode ser uma conjuno, expressando tempo ou razo. Na orao em questo: This familiarity is convenient razo e podendo, portanto, ser substituda por because; d) Incorreta. Instead quer dizer ao invs disso e furthermore quer dizer alm de, alm disso; e) Incorreta. O verbo to be left out quer dizer deixar de for a, enquanto included significa exatamente o oposto. 04) Os termos prevent from (Texto II) e realize (Texto III) significam, respectivamente: a) prevenir-compreender b) impedir-compreender c) prevenir-imaginar d) impedir-idealizar e) preparar-idealizar RESOLUO: ALTERNATIVA B A questo traz famosos falsos cognatos que costumam confundir os estudantes com menor domnio do ingls. Prevent from quer dizer impedir, evitar e realize significa notar, compreender.

64

05) No Texto III o termo which em the seventh and most important of which is that refere-se a a) the new philosophy of Animalism b) the wall of the barn c) commandments d) all animals e) all people RESOLUO: ALTERNATIVA C Nesta questo of which faz referncia a um elemento que for a citado anteriormente, e este elemento os mandamentos, como se pode notar atravs da leitura de todo o perodo: The Seven Commandments () are written on the wall (), the seventh and most important of which, cuja traduo Os Sete Mandamentos () foram escritos na parede (), o stimo e mais importante deles 06) No final do Texto III, o autor afirma que os animais no conseguiam mais diferenciar os homens dos porcos. Qual das opes abaixo, com frases encontradas no Texto II, faz observao semelhante? a) power turned the pigs from simple comrades to ruthless dictators. b) dictators who managed to walk on two legs, and carry whips. c) human nature and diversity prevent people from being equal. d) the creatures outside looked from pig to man. e) but already it was impossible to say which was which. RESOLUO: ALTERNATIVA E A alternativa E tem como traduo mas j era impossvel dizer qual era qual em um contexto em que se enfatiza significativamente a semelhana entre homens e porcos. 07) A expresso break out em destaque no Texto III significa a) to make a sudden, quick advance b) to bring or come to an end c) to develop suddenly d) to force or make a way through e) to cause to separate into pieces suddenly or violently RESOLUO: ALTERNATIVA C To break out quer dizer aparecer repentinamente. As demais alternativas trazem os seguintes significados: a) fazer um repentino, rpido avano b) finalizar, concluir ou chegar a um acordo d) abrir caminho e) rasgar ou espedaar As questes de 8 a 10 referem-se ao seguinte cartaz de divulgao de evento, afixado em um dos murais do ITA. 5th

Brazilian
MEETING
2 0 0 6

SBPMat BRAZIL-MRS
Sociedade Brasileira em Pesquisas de Materiais CALL FOR PAPERS

MRS
FLORIANPOLIS
The annual meeting of the Brazilian Materials Research Society (SBPMat), now being held for the 5th time, is the Largest and most comprehensive Brazillilan national conference in Materials Science and Engineering. The V SBPMat / Brazilian MRS Meeting 2006 will provide a very stimulating environment for the dicussion of relevant themes in materials research. The Meeting will be held together with the X Brazilian Congress for Microscopy of Materials (MICROMAT 2006), promoted by the Brazilian Society for Microscopy and Microanalysis (SBMM). The joint event will take place in the Costo do Santino Resort & Spa in Florianpolis, State of Santa Catarina, Brazil. As in previous meeting, the conference will bring together scientists and engineers, from Brazil and abroad, interested in multidisciplinary topics of materials research. The scientific program consistis of 13 symposia, each featuring Internationally recognized speakers as well as oral and poster Contributions. In addition, leading-edge topics of materiais research Will be highlighted by distinguished lectures in plenary sessions.

Brazilian MRS Meeting 2006 Florianpolis, SC, Brazil October 8 th 12 th, 2006 Costo do Santinho Resort & SPA Abstracts Submission Deadline

June 30, 2006 08) De acordo com o texto do cartaz, considere as seguintes afirmaes:

65

I. o V SBPMat uma conferncia internacional realizada anualmente na rea de Cincias dos Materiais e Engenharia. II. o evento tem por objetivo promover discusses sobre temas ligados pesquisa em Materiais. III. estaro presentes no evento engenheiros e cientistas de vrias partes do mundo, no apenas do Brasil. a) apenas a I b) apenas a II c) apenas I e II d) apenas II e III e) todas RESOLUO: ALTERNATIVA D I. Falsa A primeira afirmao torna-se falsa, por dizer que a conferncia internacional, enquanto o texto diz: () the largest and most comprehensive Brazilian national conference, isto , trata-se de uma conferncia nacional. II. Verdadeira Ao final do primeiro pargrafo lemos: MRS meeting 2006 will provide a very stimulating environment for the discussion of relevant themes in material reserch., ou seja, o encontro ir proporcionar um ambiente muito estimulante para a discusso de relevantes temas em pesquisa de materiais. III. Verdadeira No quarto pargrafo lemos: The conference will bring together scientists and engineers, from Brazil and abroad (), ou seja, a conferncia ir colocar em contato cientistas e engenheiros do Brasil e de outros pases. 09) Ainda de acordo com o texto no cartaz, considere as seguintes afirmaes: I. a programao cientfica do evento composta de simpsios, palestras, comunicaes orais e posters. II. o V SBPMart e o X MICROMAT sero realizados concomitantemente. III. a informao Abstract Submission Deadline June 30, 2006 refere-se ao prazo final para inscries no evento. Ento est(o) correta(s): a) apenas a I b) apenas a II c) apenas a III d) apenas I e II e) todas RESOLUO: ALTERNATIVA D I. Correta Esta afirmao confirmada nas ltimas linhas do ltimo pargrafo. The scientific program consists of 13 symposia, each featuring internationally recognized speakers as well as oral and poster contributions. In addition, leading-edge topics of materials research will be highlighted by distinguished lecturers in plenary sessions., ou seja, na programao h 13 simpsios, apresentaes orais, posters, alm de palestras conduzidas por distintos professores universitrios. II. Correta - A segunda afirmao pode ser confirmada pelo segundo pargrafo: The Meeting will be held together with the X Brazilian Congress for Microscopy of Materiais (MICROMAT 2006). To be held together quer dizer acontecer junto tornando corret a assertiva. III. Incorreta Abstract submission deadline significa prazo final para apresentao de resumos de trabalhos (no caso trabalhos cientficos, pesquisas), no fazendo referncia s inscries propriamente ditas. possvel, por outro lado, que a inscrio de palestrante do evento e apresentao dos resumos seja um processo nico, o que no esclarecido pelo texto. De todo modo, este provavelmente no seria o prazo mximo para se inscrever no evento, por exemplo, como ouvinte. 10) Finalmente considere as seguintes informaes: I. the largest and most comprehensive conference indica que a conferncia um evento de grande porte e de carter abrangente. II. being held em now being held for the 5th time e will be held together em The meeting will be held together with the X Brazilian poderiam ser substitudos, respectivamente, por taking place e will occur. III. leading-edge topics refere-se a temas em destaque na rea de pesquisa de materiais. Ento eest(o) correta(s): a) apenas a I b) apenas I e II c) apenas II e III d) apenas I e III e) todas RESOLUO: ALTERNATIVA E I. Correta Largest significa maior; most comprehensive o mesmo que mais abrangente. II. Correta To be held sinnimo de to take place, will be held together, significa vai ocorrer junto e, na frase, pode ser substitudo sem perda de significado por will occur, ficando a frase com a seguinte traduo: O encontro vai ocorrer com o X III. Correta Leading-edge topics significa tpicos de elevada relevncia e modernos, avanados, novos. 11) A opo que melhor preenche a lacuna da primeira frase no dilogo abaixo : A Well, Mr. Vagner, our point is that Ahn, ___________________ B Not at all

66

A Thanks you know, people here got quite intolerant B I dont mind. C Meu chefinho est tentando parar!!! a) Can I smoke? b) Mind if I smoke? c) Do you smoke? d) Want a cigarrete? e) Are you OK? RESOLUO: ALTERNATIVA B Alm da resposta dada, Not at all, que pode significar de nada ou de modo nenhum, dependendo do contexto, o desenrolar do quadrinho mostra que o autor da lacuna estaria perguntando ao outro se ele se importa de o primeiro fumar, portanto a frase que melhor preenche a lacuna Mind if I smoke?. As questes de 12 a 16 referem-se ao texto a seguir: An ever-improving electrical system If you total up all the wires and cables of all diameters and lengths found in a large airplane, they could stretch for hundreds of miles. The plane also has hundreds of places using alternating and low tension continuous current (115 volts). It is common practice for these generating and distribution systems. To maintain a solid energy reserve for peak usage times and unusual circumstances. The kilowatts used on new models cannot be compared to what will be required on board the megajets A380 (now going through flight testing) and the future A350, plus the Boeing 787 Dreamliner. All of these plan on offering new dimensions in inflight entertainment and a level of connectivity that is comparable to what we already have on the ground-for laptops and individual cell phones, broadband internet, email, Wi-Fi (long distance enlacing) and all the rest. In todays wide-body panes, the biggest consumer is the heating for de-icing and defrosting the windscreens. But, even after adding in the 16 ovens and coffee machines in the gallery, plus the electronic navigation, communications and avionics systems, we will still be well short of the electrical system on board the advance jets that will go into service between 2006 and 2010. Besides the fact that each of those aircraft will have two (not just one) alternators per engine, there will be other brand-new electrical applications. The wiring may come with a variety of tensions and frequencies, which will bring a new level of freedom to designers who are projecting a much larger, more flexible. More efficient, and safer electrical system.
Texto by Ernesto Klotzel, Illustration Clayton Jr. Icaro, April, 2006

12) Assinale a opo que melhor explicita o objetivo principal do texto a) Descrever o sistema eltrico de um avio de grande porte. b) Descrever as partes de um avio que demandam o uso de sistema eltrico. c) Discorrer sobre as inovaes que sero realizadas no sistema eltrico em novos avies de grande porte. d) Comentar que, no futuro, poderemos usar celulares e internet em mega jatos. e) Ressaltar que, no futuro, a fiao de novos avies comportar tenses e freqncias variadas. RESOLUO: ALTERNATIVA C O ttulo An ever-improving electrical system (um sistema eltrico sempre em desenvolvimento), e todo o texto falam sobre como os avies, em pouco tempo, no suportaro a demanda pelo sistema eltrico e quais sero os prximos passos para o desenvolvimento deste sistema. 13) Atualmente, nos grandes avies, a maior parte da energia consumida: a) na utilizao de fornos e mquinas de caf. b) em navegao eletrnica, sistemas de comunicaes e de avinica. c) em sistemas antiembaamento e degelo dos pra-brisas. d) na manuteno da temperatura interna da aeronave. e) na manuteno da energia da aeronave para eventuais emergncias. RESOLUO: ALTERNATIVA C No incio do terceiro pargrafo temos The biggest energy consumer is the heating for de-icing and defrosting the windscreens., sendo windscreen o mesmo que prabrisa em portugus. 14) O termo these em All of these plan on offering est substituindo: a) novos megajatos. b) equipamentos eletrnicos que podero ser usados a bordo. c) nveis de conectividade comparveis ao que temos em solo. d) quilowatts usados nos novos megajatos. e) correntes eltricas que sero usadas nos megajatos.

67

RESOLUO: ALTERNATIVA A O perodo anterior sentena termina falando dos novos megajatos. O seguinte faz referncia contextual these a estes novos megajatos, levando-nos alternativa A. 15) A expresso to be well short of em we will still be well short of the electrical system pode ser substituda por: a) to be far from b) to be adequately suppolied with c) to be below the limit d) to be close e) to be left off RESOLUO: ALTERNATIVA A A expresso to be well short of signifca to be far from. Note o sentido do trecho But, even after adding in we will still be well short of the electrical system on board, o qual afirma que mesmo colocando muitos dos equipamentos que consumam energia eltrica no avio ainda estaremos distantes da capacidade do sistema eltrico a bordo. Este fato justificado pelo constante desenvolvimento na rea. 16) O aspecto que permitir maior liberdade as projetistas que: a) os novos jatos contaro com aplicaes eltricas inovadores. b) os megajatos tero dois alternadores por motor e a fiao comportar tenses e freqncias diversificadas. c) os novos jatos contaro com corrente alternada e contnua. d) os megajatos oferecero novas possibilidades de entretenimento. e) as novas aeronaves contaro com um sistema eltrico mais eficiente e seguro. RESOLUO: ALTERNATIVA B No final do ltimo pargrafo temos uma breve descrio de algumas melhorias que daro mais liberdade aos projetistas do sistema eltrico: The wiring may come with a variety of tensions and frequencies, which will bring a new level of freedom to designers who are projecting (). A variedade de tenses e freqncias que dar aos projetistas maior liberdade citada na alternativa B. As questes de 17 a 20 referem-se a um trecho do prefcio a seguir: () For about 25 years, I have the opportunity to observe the efforts of many individuals applying digital image-processing techniques to problems offered by the real world. A few of these individuals have established and enduring track record solid success on almost every attempt. They have consistently contributed innovate and effective solutions that creatively employ the tools of the discipline. These highly productive individuals demonstrably hold several characteristics in common. One can venture to assume that these characteristics constitute a formula for success, to whatever extent such a thing can exist in this field. Uniformly, these success persons have (1) a genuine interest in even fascination with the technology involved, (2) a thorough understanding of the fundamentals of this highly multidisciplinary technology, (3) a conceptual type of understanding (as opposed to rote memorization of totally abstract theory), and (4) a knack for seeing problems visually, graphically, and from more than one viewpoint. In line with this cast point, they often find themselves hard pressed to explain their ideas without the aid of a graph or drawing. This book is designed to help the reader develp the last three of these traits nd perhaps enhance the first as well. The selection of materials for inclusion (and, equally important, for omission), the example used, the references cited, and the exercises and suggestions for projects are all directed toward this goal. In the field of digital image processing, mathematical analysis forms the stable basics upon which one can make definite predictions regarding the performance of a digital imaging system. In this treatment, however, mathematics is employed more as a faithful servant than as ruthless master. The emphasis is on developing a conceptual understanding, and the analysis used to support this goal.
Castleman, K. R. Digital Image Processing. Prentice Hall, 1996

17) Assinale a assero que NO indica possveis objetivos do livro. a) Auxiliar no desenvolvimento da compreenso conceitual da teoria do processamento de imagens digitais em contraposio memorizao mecnica da teoria. b) Aprofundar a compreenso dos fundamentos da tecnologia de processamento de imagens digitais. c) Desenvolver a capacidade de enxergar problemas relacionados a processamento de imagens sob pontos de vista variados. d) Apresentar exemplos, referncias, exerccios e sugestes para o desenvolvimento de projeteos usando tecnologia de processamento de imagens digitais.

68

e) Aumentar o interesse genuno pela tecnologia de processamento de imagens digitais. RESOLUO: ALTERNATIVA D O propsito do livro ajudar no desenvolvimento de trs das caractersticas de pessoas de sucesso na rea de processamento de imagens digitais. Isto est explicado no quarto pargrafo, onde feita uma referncia ao desenvolvimento das trs ltimas qualidades citadas no pargrafo anterior e melhoramento da primeira qualidade, tambm citada anteriormente: This book is designed to help the reader develop the last three of these traits and perhaps enhance the first as well. O terceiro pargrafo, que enumera as caractersticas das pessoas bem sucedidas na rea, fala das seguintes caractersticas: (1) interesse na tecnologia envolvida. (2) compreenso dos fundamentos da tecnologia de processamento de imagens digitais. (3) compreenso conceitual da teoria do processamento de imagens digitais em contraposio memorizao mecnica da teoria. (4) a capacidade de enxergar problemas relacionados a processamento de imagens sob pontos de vista variados. Uniformly, these success persons have (1) a genuine interest in even fascination with the technology involved, (2) a thorough understanding of the fundamentals of this highly multidisciplinary technology, (3) a conceptual type of understanding (as opposed to rote memorization of totally abstract theory), and (4) a knack for seeing problems visually, graphically, and from more than one viewpoint. 18) Considere as seguintes asseres: I. para elaborar o livro, o autor estudou, por um perodo de vinte e cinco anos, o trabalho de pessoas que utilizam tcnicas de processamento de imagens. II. no processamento de imagens digitais, a anlise matemtica utilizada como apoio ao desenvolvido da compreenso conceitual. III. a anlise matemtica oferece os fundamentos sobre os quais se pode fazer previses precisas em relao ao desempenho de um sistema de imageamento digital. Das afirmaes acima est(o) correta(s): a) apenas a I b) apenas a II c) apenas a III d) apenas I e II e) apenas II e III RESOLUO: ALTERNATIVA E I. Falsa A resposta para a primeira afirmao se encontra logo no comeo do texto, em que o autor afirma: For 25 years, Ive had the opoortunity to observe the efforts of many individuals (). Em nenhum momento este trecho denota estudo, apenas observao de pessoas e seus trabalhos, tornando a afirmao falsa. II. Verdadeira No ltimo pargrafo lemos sobre a matemtica: mathematics is employed more as a faithful servant than as a ruthless master. The emphasis is on developing a conceptual understanding (). Este trecho demonstra que a matemtica utilizada no desenvolvimento de compreenso conceitual, como afirma a assero, tornando-o de fato falso. III. Verdadeira Ainda no ltimo pargrafo, temos o trecho () mathematical analysis forms the stable basis upon which one can make definite predictions regarding the performance of a digital imaging system., que nada mais do que a verso em ingls da assertiva. 19) Assinale a opo que melhor traduz, respectivamente, os termos venture, knack e hard-pressed a) especular; inclinao; no obrigados b) atrever-se; inaptido; inflexveis c) evitar; propenso; sobrecarregados d) ser prudente; idoneidade; obsessivos e) arriscar-se; talento; sob presso RESOLUO: ALTERNATIVA E Venture: arriscar-se, aventurar-se, Knack: aptido, destreza, talento, habilidade, hard-pressed: sob presso, pressionados 20) As expresses as opposed to, in line with e however podem ser substitudas, respectivamente, por: a) in contrast with, in agreement with, although b) in contrast with, in agreement with, though c) in contrast with, in dissension with, though d) in conflict with, in dissension with, although e) in conflict with, in agreement with, although RESOLUO: ALTERNATIVA B As opposed to usado para dizer que duas coisas so muito diferentes, portanto deve ser substitudo por in contrast with, in line with expressa concordncia, portanto deve ser substituda por in agreement with, j however expressa o contraste entre duas idias colocadas no perodo e deve ser substituda por though pois esta palavra pode assumir uma funo adverbial que tem o objetivo de contradizer aquilo que foi dito anteriormente.

69

03. IME INSTITUTO MILITAR DE ENGENHARIA


TEXTO QUESTES 1 e 2 New planet definition sparks furore The new planet definition that relegates Pluto to dwarf planet status is drawing intense criticism from astronomers. It appears likely that the definition will not be widely adopted by astronomers for everyday use, even though it is the International Astronomical Unions (IAU) official position. On Thursday, astronomers at the IAU meeting in Prague approved a resolution that says the solar system has only eight planets, with Pluto excluded. Pluto is considered a dwarf planet instead. Bu the new definition has provoked a backlash. Alan Stern, who heads NASAs New Horizons mission to Pluto and works at the Southwest Research Institute in Boulder, Colorado, US, says the new definition is awful. The definition introduced is fundamentally flawed,, he told New Scientist. As a sicentist, Im embarassed. Four planets He says only four of the eight objects mentioned in the IAU definition actually meet the definitions criteria and Earth, Mars, Jupiter and Neptune do not. That is because the definition stipulates that to be a planet, an object must have cleared the neighborhood around is orbit. But Earths orbital neighborhood is filled with thousands of near-Earth asteroids, Stern says. And Mars, Jupiter and Neptune have so-called Trojan asteroids sharing their orbits. This is a half-baked criterion for planethood, he says. He says the new definition was pushed by people who are unhappy with having large numbers of planets (an earlier proposal, which was abandoned after heavy criticism at the meeting, would have potentially allowed hundreds of new planets into the fold). Its just people that say things like, School kids will have to memorize too many names. Do we limit the number of stars because children have to think of too many names? Or rivers on the Earth? Its just crazy, Stern told New Scientist. Minority report Stern is also critical of the fact that only astronomers present for the vote, which ocurred at the end of the two-week meeting, were allowed to have their say on the matter No email voting was allowed for the decision it was made by a show of hands and that meant that less than 5% of the clearly 9000 IAU members actually voted. Youre going to see a real backlash in the coming weeks, he says. I know there is a petition among planetary scientists thats getting a lot of support. In any case, he says, astronomers are not obligated to use the new definition, since the IAU does not have the power to enforce it. I dont think its going to be very widely followed, he says. David Weintraub, author of the upcoming book Is Pluto a Planet? And a researcher at Vanderbilt University in Nashville, Tennessee, US, says he thinks Pluto is still a planet even under the new definition. Grammar issue As best I can tell, dwarf is an adjective and planet is a noun, he told New Scientist. I think the IAU thinks they defined Pluto to not be a planet But they in fact have defined Pluto to be a planet a particular kind of planet. Astronomers who proposed the new definition respond that the term dwarf planet is meant to be thought of as a single concept. And others pooint out that minor planets asteroids and other small bodies are not considered planets. But he agrees with Stern that the stipulation that a planet clears out its neighbourhood is flawed. A better definition would say a planet is an object that orbits a star and is large enough to be spherical, but is not large enough to be: a brown dwarf a failed star with between about 13 and 75 times the mass of Jupiter or a star, he says. This is crazy Everyone agrees on those criteria, he says, The question is, can we agree on additional criteria to refine the definition further? I think the answer is no. Everybody who has communicated with me is saying. This is crazy and we dont agree with it, he adds Im not convinced that the folks who were at the meeting represented well the larger community.

70

But not everyone is unhappu with the decision. Richard Conn Henry pleased with the outcome. As far as Im concerned, the right decision was made, he told New Scientist. I know a planet when I see it and there are eight of them. He says it makes no sense to call Pluto a planeet because it is just one of huge numbers of objetcs in the Kuiper Belt beyond Neptune.
(Adaptado de New Scientist Space, Augusto 2006)

1.1) Leia o texto New planet definition sparks furoree responda, EM PORTUGUS, as perguntas que se seguem. Considering the content of the text, why is the title New planet definition sparks furore appropriate? RESOLUO: O ttulo do texto se mostra muito apropriado devido ao debate gerado em torno da nova definio de Pluto como um planeta ano. Tal resoluo foi tomada em uma reunio de duas semanas, e votaram para a deciso apenas astrnomos presentes a ela, e no todos os membros da IAU (International Astronomical Union). De acordo com o texto, isso corresponde a menos de 5 por cento dos membros. Alm disso, o critrio utilizado foi falho, de acordo com a nova definio um objeto deve ter limpa a sua vizinhana orbital para ser considerado um planeta. Planetas como Terra, Jpiter, Marte e Netuno no se enquadram nesta categoria, portanto tambm no poderiam ser chamados de planeta. Finalmente o IAU no tem poder de obrigar a nova denominao dada por ele a Pluto, gerando uma tendncia de que poucos astrnomos a adotariam no futuro. 1.2) Leia o texto New planet definition sparks furore e responda, EM PORTUGUS, as perguntas que se seguem. Alan Stern, of Southwest Research Institute, states, The definition is fundamentally flawed. Find in the text one argument that can be used to support his opinion. RESOLUO: De acordo com o cientista a definio falha pois h outros planetas que no se enquadram nele. A nova regra diz que para um corpo ser considerado um planeta ele deve ter sua vizinhana orbital limpa. No entanto Terra, Marte, Netuno e Jpter no se enquadram nesta categoria pois possuem asterides em suas vizinhanas orbitais. 1.3) Leia o texto New planet definition sparks furore e responda, EM PORTUGUS, as perguntas que se seguem. The scientist Alan Stern states that according to the criterion established by the resolution, Earth shouldnt be considered a planet. Why wouldn Earth be considered a planet? RESOLUO: A definio do texto diz que um corpo celeste pode ser considerado um planeta quando ele apresenta a sua vizinhana limpa, ou seja, no h corpos celestes em torno dele. Mas o cientista afirma que h milhares de asterides prximos Terra. Dessa maneira, no poderamos dizer que a Terra um planeta, de acordo com essa nova definio. 1.4) Leia o texto New planet definition sparks furore e responda, EM PORTUGUS, as perguntas que se seguem. Richard Conn Henry, of Johns Hopkins University, says he is pleased with the outcome. Find in the text one argument that supports his position. RESOLUO: Richard Henry afirma que no faz sentido chamar Pluto de planeta pois ele seria apenas mais um dentre muitos outros enormes corpos celestes no cinturo de Kuiper. 2.1) Considere as informaes contidas no texto New planet definition sparks furore What resolution was approved at the IAU meeting in Prage? a) One that redefines what a planet is b) One that states Earth, Mars, Jupiter and Neptune are not planets c) One that sends a mission headed by Alan Stern to Pluto d) One that considers Trojan asteroids in Mars, Jupiter and Neptune orbits RESOLUO: ALTERNATIVA A No sexto pargrafo lemos () the definition stipulates that to be a planet an object must have cleared the neighborhood around its orbit (), ou seja, trata-se de uma definio do que seria um planeta. Isto confirmado pelo ttulo do texto, que indica a nova definio de planeta, levando alternativa A.

71

2.2) Considere as informaes contidas no texto New planet definition sparks furore e marque a alternativa correta em cada item seguinte. According to the Prague resolution a planet a) has a neighborhood of clear objetcts. b) was included in the solar system. c) must have its orbit cleared. d) is a half-backed criterion. RESOLUO: ALTERNATIVA C O texto diz, no sexto pargrafo, que um dos critrios (bastante controverso) para classificar um corpo celeste na categoria de planeta que ele deve apresentar sua rbita limpa. 2.3) Considere as informaes contidas no texto New planet definition sparks furore e marque a alternativa correta em cada item seguinte. According to the text it is correct to affirm that a) hundreds of new planets were included in the Prague resolution list of planets. b) an earlier proposal for adopting a planet definition was rejected. c) the Prague proposal was abandoned after heavy criticism at the meeting. d) school kids will not understand the new definition. RESOLUO: ALTERNATIVA B Segundo o pargrafo 8, temos: () (an earlier proposal, which was abandoned after heavy criticism at the meeting, would have potentially allowed hundreds of new planets into the fold.) (). Temos que este trecho fala de uma proposta anterior, que faria com que o nmero total de planetas fosse elevado casa da centena. Esta proposta foi rejeitada. 2.4) Considere as informaes contidas no texto New planet definition sparks furore e marque a alternativa correta em cada item seguinte. What can be inferred by the statement Yourre going to see a real blackash in the coming weeks. I know there is a petition among planetary scientists thats getting a lot of support. a) Only 5% of the nearly 2000 IAU members will vote in the next meeting. b) Most of the members of IAU dont have access to emails. c) The petition among scientists will be sent by email. d) The resolution will probably be contested by the majority of members because they were not allowed to vote by email. RESOLUO: ALTERNATIVA D Questes de interferncia sempre so perigosas, pois o candidato precisar de conhecimento de mundo ou algum vocabulrio para resolv-la. Nesta questo era necessrio compreender o significado da palavra backlash. O aluno acostumado com o mtodo de decomposio de palavras perceberia que back quer dizer trs, volta. O texto diz que repercusses seriam vistas nas prximas semanas. O texto fala que a resoluo causou descontentamento por parte da comunidade astronmica devido ao fato que apenas os presentes puderam votar na resoluo da reunio de Praga e implica que provavelmente esta ser contestada por este motivo. TEXTO PARA A QUESTO 3 Leia o texto Relaxation e COMPLETE CADA LACUNA NUMERADA DAS FRASES A SEGUIR COM APENAS UMA PALAVRA reetirada desta passagem, mantendo a mesma idia do texto original. Relaxation True relaxation is most certainly not a matter of flopping down in front of the television with a welcome drink. Nor it as about drifting into an exhausted sleep. Useful though these responses to tension and overtiredness may be, we should distinguish between them and conscious relaxation in terms of quality and effect. Regardless of the level of tiredness, real relaxation is a state of alert yet at the same time passive awareness, in which our bodies are at rest while our minds are awake. Moreover, it is natural for a healthy person to be relaxed when moving as resting. Being relaxed in action means we bring the appropriate energy to everything we do, so as to have a feeling of healthy tiredness by the end of the day, rather than one of exhaustion. Unfortunately, as a result of living in todays world, we are under constant strain and have difficulty in coping, let alone nurturing our bodys abilities. What needs to be rediscovered is conscious relaxation. With this in mind we must apply ourselves to understanding stress and the nature of its causes, however deep-seated. 3.1) The text relates two ways of relieving from tension and over-tiredness, reinforcing the power of __3.1__ relaxation over the false one, which is considered only momentary. RESOLUO: TRUE devido relao dicotmica da frase entre true e false.

72

3.2) The difference between these two ways of relaxing lies upon its __3.2__ and __3.3__. RESOLUO: QUALITY O primeiro pargrafo do texto cita que podemos distinguir entre as maneiras de relaxar em termos de suas qualidades e efeitos. 3.3) The difference between these two ways of relaxing lies upon its __3.2__ and __3.3__. RESOLUO: EFFECT O primeiro pargrafo do texto cita que podemos distinguir entre as maneiras de relaxar em termos de suas qualidades e efeitos. 3.4) In order to feel really relaxed, people should keep their __3.4__ alert at the same time their __3.5__ are at rest. RESOLUO: MINDS As questes 3.4 e 3.5 se encontram no final do primeiro pargrafo do texto em que lemos () in which our bodies are at rest while our minds are awake () em que nossos corpor esto em repouso enquanto as nossas mentes esto ativas. 3.5) In order to feel really relaxed, people should keep their __3.4__ alert at the same time their __3.5__ are at rest. RESOLUO: BODIES As questes 3.4 e 3.5 se encontram no final do primeiro pargrafo do texto em que lemos () in which our bodies are at rest while our minds are awake () em que nossos corpor esto em repouso enquanto as nossas mentes esto ativas. 3.6) Someone who is healthy is supposed to feel relaxed either moving or __3.6__. RESOLUO: RESTING Resposta no incio do segundo pargrafo () to be relaxed either moving or resting () 3.7) It is possible to feet relaxed even when you are moving or doing any other activity, it is just a matter of devoting the right amount of __3.7__ to the things you do, not less or more. RESOLUO: ENERGY Resposta no incio do segundo pargrafo () Being relaxed in action means we bring the appropriate energy to everything we do () Estar relaxado em ao significa que ns usamos a quantidade de energia apropriada para tudo o que fazemos. 3.8) If by the time you go to sleep you have a feeling of __3.8__ you are not taking the right relaxation, for by the time your day finishes you should have a feeling of healthy __3.9__. RESOLUO: EXHAUSTION Respostas das questes 3.8 e 3.9 se encontram nas duas ltimas linhas do segundo pargrafo. so as to have a feeling of healthy tiredness by the end of the day, rather than one of exhaustion. para ter uma sensao de cansao saudvel ao fim do dia, ao invs de um sentimento de exausto. 3.9) If by the time you go to sleep you have a feeling of __3.8__ you are not taking the right relaxation, for by the time your day finishes you should have a feeling of healthy __3.9__. RESOLUO: TIREDNESS Respostas das questes 3.8 e 3.9 se encontram nas duas ltimas linhas do segundo pargrafo. so as to have a feeling of healthy tiredness by the end of the day, rather than one of exhaustion. para ter uma sensao de cansao saudvel ao fim do dia, ao invs de um sentimento de exausto. 3.10) The text suggests that practicing __3.10__ relaxation is a way of living in nowadays stressing world paying close ttention to our bodies abilities and needs. RESOLUO: CONSCIOUS Terceira linha do terceiro pargrafo. What needs to be rediscovered is conscious relaxation. O que precisa ser redescoberto o relaxamento consciente. 4.1) Marque com um (X) a nica alternativa correta para cada uma das perguntas abaixo. My discovery of Tillie Olsen was a figt from a friend; years ago she gave me her copy of Tell Me a Riddle because she liked the stories and wanted to share the experience. What do we know of Tillie Olsen? a) She is a friend. b) Shie likes stories. c) She gives gifts. d) She is an author. RESOLUO: ALTERNATIVA D O texto diz que a descoberta de Tillie Olsen foi um presente de uma amiga, que deu a sua cpia do livro Tell Me a Riddle para dividir a experincia. Assim, Tille Olsen a autora do livro. 4.2) Marque com um (X) a nica alternativa correta para cada uma das perguntas abaixo.

73

The medical journal reported that heart attack victims who recover are approximately five times as likely to die within the next five years as those people without a history of heart disease. What did this article say about people who have had a heart attack? a) They are more likely to die in the near future than others. b) They will die in five years. c) They are less likely to die than people without a history of heart disease. d) They are likely to recover. RESOLUO: ALTERNATIVA A A expresso five times as likely comparativa e pode ser traduzida no texto por cinco vezes mais propensas. Assim, temos que as pessoas que sofreram um ataque do corao esto mais propensas (cinco vezes mais que uma pessoa que no teve ataque) a morrer dentro do perodo de cinco anos seguintes. 4.3) Marque com um (X) a nica alternativa correta para cada uma das perguntas abaixo. Thanks to the ubiquitous use of vanilla as a _______ in ice creams and cakes the world over, its taste is more ___________ to the majority of people than the appearance of the plant. Que alternativa contm as palavras que completam correta e respectivamente as lacunas da frase? a) flavor, recognize b) flavoring, recognizable c) flavored, recognized d) flavoring, recognizing RESOLUO: ALTERNATIVA B Por ser precedido de artigo, percebe-se que a primeira lacuna um substantivo (substncia que d o sabor, no caso, de baunilha). O sufixo ing formador de substantivos e portanto chegamos a flavoring. Entretanto o substantivo recognizing no existe, e sim recognizable (reconhecvel, familiar). TEXTO PARA A QUESTO 5 Leia o perodo seguinte. Em cada uma das linhas pode haver uma palavra excedente que torna errada a estrutura da elocuo. Caso a linha esteja correta, escreva no espao indicado a palavra CORRETO; caso haja alguma palavra extra, escreva-a no espao indicado. Training is not a cost. Its an investment. It really doesnt matter what much we pay for an investment. What is really relevant is what we get in return. 5.1) Linha 1 ______________ RESOLUO: MUCH A palavra no faz parte do contexto, tornando-a dispensvel e irrelevante. Uma outra forma que a tornaria correta seria dizer how much (we pay), ou seja, no importa o quanto pagamos. Porm o enunciado fala sobre palavra excedente, portanto elimina-se much, fazendo com que a frase tenha o sentido de que no importa o que pagamos em um investimento. 5.2) Linha 2 ______________ RESOLUO: CORRETO No h nada que desabone a elocuo da segunda linha.

74

You might also like